You are on page 1of 258

DR.

FAHMI KHAN

Education & Research Sharing

Menu

MULTIPLE CHOICE QUESTIONS-1

1. A 33-year-old anthropologist from New York had been doing research in a desert region of Arizona
for about 6 months. After returning home, he visits his physician complaining of an influenza-like
illness with cough, mild chest pain, and occasional fever. He says that the illness started during the last
few weeks of his stay in Arizona. Red, tender nodules are present on his shins. Chest x-rays fail to reveal
evidence of pulmonary infiltrates or pleural effusion. Which of the following is the most appropriate
next step in the management of this patient?

A) Delay treatment until culture results are obtained.

B) Begin treatment with fluconazole.

C) Begin treatment with amphotericin B.

D) Aspirate bone marrow and culture.

E) Institute immediate isolation.

Answer: A

2. A young man is referred by his GP following investigation for recurrent mouth ulceration. Subsequent
blood count shows HB of 13.2, WBC 3.8 (neutrophil 1.2, lymph 1.5) PLT 332, examination reveals a fit
young male with no evidence of organomegaly or lymphadenopathy, further questioning reveals the
history of mouth ulceration occurring over the previous 3-4 years. Select the most likely diagnosis 

A) AML 

B) Post-viral neutropenia 

C) HIV infection 
D) Drug induced neutropenia 

E) Cyclical neutropenia.

Answer: E

3. A 35-year-old woman presented with tender lesion on both legs. She had no respiratory symptoms
and was not on any medications. O/E: she was febrile, pulses 90 regular and blood pressure 136/88. Her
chest was clear. There were bilateral erythematous raised lesions on her shins. Chest x ray shows bi-hilar
lymphadenopathy. The most helpful test that would give a definitive diagnosis would be: 

A) Kveim test 

B) Serum ACE level 

C) Serum calcium 

D) Skin biopsy 

E) Transbronchial needle biopsy

Answer: E

4. A 78-year-old man complains of increasing fatigue and bone pain, especially around the knees and
ankles. He has a long-standing anemia with hemoglobin of 9 to 10 g/dL and MCV of 102. He had not
responded to therapeutic trials of iron and vitamin B12, but had been symptomatically stable until the
past month. Examination reveals pallor and spleen tip just palpable at the left costal margin. CBC reveals
hemoglobin of 8.2 g/dL, but for the first time his platelet count is low (15,000); the white blood cell
count is 14,000. What is the likely cause of his worsening anemia?

A) Folic acid deficiency 

B) Acute myeloid leukemia 

C) Myelofibrosis 

D) Tuberculosis 

E) Viral infection
Answer: B

5.  A 58-year-old man with history of chronic myeloid leukemia was admitted with pneumonia and deep
venous thrombosis. He was started on antibiotics and I.V. heparin. His condition deteriorated and had
acute respiratory distress syndrome and hypotension. Despite large amounts of i.v. fluids and inotropes,
he remained hypotensive. You are considering adrenal insufficiency. What should you do next?

A) Order random cortisol then treat with hydrocortisone

B) Start hydrocortisone then do the ACTH stimulation test

C) Start dexamethasone then do the ACTH stimulation test

D) Start hydrocortisone

E) Do the ACTH stimulation test; treat according to the results

Answer: C

6. A 68-year-old diabetic woman with chronic renal insufficiency is admitted to the hospital with
urosepsis. She has a history of anaphylactic reaction to penicillin that required intubation for severe
bronchospasm. Urinalysis shows > 100 WBC, and Gram's stain shows 3+ plump gram-negative rods.

Which of the following antibiotics would be most appropriate for this patient with a known severe
penicillin allergy?

A) Ampicillin

B) Ceftazidime

C) Vancomycin

D) Aztreonam

E) Imipenem

Answer: D
7. 65-year-old man with history of chronic AF presents to ER with acute memory impairment (recent
amnesia), confusion and impaired vision. Physical examination confirms the confusion state and right
homonymous hemianopsia. No paresis was noted. The current status is most probably related to
occlusion of: 

A) Left posterior cerebral artery 

B) Basilar artery 

C) Left middle cerebral artery (posterior branch) 

D) Left common carotid artery 

E) Right vertebral artery

Answer: A

8. A 24-year-old man is in the operating room for a massive liver injury sustained when his motorcycle
hit a truck. After one hour of surgery he has received 15 units of packed cells and has developed diffuse
oozing from the surface of his liver. Clots are no longer forming. His body temperature is 34oC.

Coagulation abnormalities expected in this patient include all of the following EXCEPT 

A) Prolonged PT

B) Prolonged aPTT

C) Prolonged bleeding time

D) Low fibrinogen level

E) Viscoelastic test (point of care coagulation testing) is usually normal.

Answer: E

9. A 42-year-old female with a recent diagnosis of systemic sclerosis, is referred to hospital with a
complaint of headaches and blurred vision. She has a past medical history of asthma. On examination,
her blood pressure is 230/120, and there is bilateral papilloedema. Which of the following medications
should be prescribed immediately?

A) IV Furosemide
B) IV Labetolol

C) Oral angiotensin II receptor blockers plus IV Sodium Nitroprusside

D) Oral captopril plus IV Sodium Nitroprusside

E) Sublingual Nimodipine

Answer: D

10. A 55 year old patient with acute necrotizing pancreatitis has been in the Intensive Care unit with
multiple organ failure for approximately 4.5 weeks. Although his clinical situation improved over the last
few days, the patient now deteriorates. There is fever up to 104 °F, a rising CRP of 398 and white blood
cell count of 27 x 109/L. A contrast enhanced CT demonstrates a large, heterogeneous, walled off,
peripancreatic collection with gas inside. What is the preferred treatment strategy at this point?

A) Conservative treatment

B) Image-guided percutaneous or endoscopic catheter drainage

C) Primary open necrosectomy

D) Primary minimally invasive surgical necrosectomy

E) Video assisted retroperitoneal debridement.

Answer: B

11. A 25-year-old female is admitted with acute dyspnea and chest pain. A diagnosis of pulmonary
embolism is confirmed and her investigations reveal urine dipstick protein ++ but no blood, anti-double
stranded DNA antibodies of 200 U/mL (0 - 73), with a 24 hour urinary protein concentration of 5g (<
0.2). Which one of the following diagnoses is most likely to be found on renal biopsy?

A) AA amyloid

B) Focal segmental glomerulonephritis

C) IgA nephropathy

D) Membranous nephropathy
E) Minimal change nephropathy.

Answer: D

12. A 46-year-old woman is referred for preoperative evaluation before undergoing resection of newly
diagnosed glioblastoma. Her HB is 12 gm/dl, leukocyte count 6.7 x109/L, and PLT 198 x109/L, the
surgeon is particularly interested in an opinion concerning her risk of bleeding. Which of the following
will provide the best estimate of her surgical risk of bleeding? 

A) Bleeding time 

B) PLT function analyzer-100 analysis 

C) PLT aggregation studies 

D) Medical history, including outcomes of previous surgical procedures 

E) PT and partial thromboplastin time.

Answer: D

13. A 42-year-old woman presents with acute onset of headache and neck pain vertigo, nausea and
vomiting. Neurological examination reveals left nystagmus, left Horner syndrome and absent left gag
reflex. Left appendicular ataxia and anesthesia to pin prick in the left face and right arm and leg. Which
of the following is the most likely diagnosis? 

A) MCA 

B) PCA 

C) ACA 

D) dissection of aorta 

E) Vertebral artery dissection

Answer: E
14. A 30-year-old man is evaluated for a thyroid nodule. The patient reports that his father died from
thyroid cancer and that a brother had a history of recurrent renal stones. Blood calcitonin concentration
is 2000 pg/mL (normal is less than 100); serum calcium and phosphate levels are normal. Before
referring the patient to a surgeon, the physician should do which of the following?

A) Obtain a liver scan 

B) Perform a calcium infusion test 

C) Measure urinary catecholamines 

D) Administer suppressive doses of thyroxine and measure levels of thyroid stimulating hormone 

E) Treat the patient with radioactive iodine

Answer: C

15.  All of the following vaccine can be given to pregnant lady except: 

A) Flu vaccine 

B) H1N1 

C) Hepatitis B vaccine 

D) Rubella vaccine 

E) Tetanus vaccine

Answer: D

16. A 78-year-old man is evaluated in the hospital for poor glycemic control before undergoing femoral
popliteal bypass surgery. He has been on the vascular surgery ward for 3 weeks with a non-healing foot
ulcer. The patient has an extensive history of arteriosclerotic CV disease, including PVD, and a 20 year
history of DM 2, his most recent Hb A1c obtained 2 months before admission was 8.9%, his diabetes
regimen consists of glibenclamide 15 mg/day. While in the hospital his plasma glucose levels have
generally been in the 200 to 250 mg/dl range he is eating well. In addition to stopping glibenclamide,
which of the following is the most appropriate treatment for this patient? 

A) Basal insulin and rapid acting insulin before meals. 


B) Insulin infusion 

C) NPH insulin twice daily 

D) Sliding scale regular insulin 

E) Insulin glargine once daily

Answer: A

17. One of flowing statement regarding COPD is true: 

A) Most smokers develop COPD during their life 

B) Tiotropium is as effective as smoking cessation in reduction the rate of decline in FEV1. 

C) Long term O2 therapy improves survival in all COPD treatment with FEV1 less than 50% of predicted. 

D) Rehabilitation may improve survival in patients with severe COPD 

E) Inhaled corticosteroids decreases exacerbation rate of COPD patients

Answer: E

18. What is the best laboratory goal to use to guide fluid resuscitation in acute pancreatitis?

A) Hematocrit

B) Blood urea nitrogen

C) Serum creatinine

D) Amylase

E) Lipase

Answer: A
19. A 62-year-old female with a history of a recent pulmonary embolus presents to your office for
follow-up on anticoagulation treatment. She takes warfarin on a daily basis. She reports that for the last
week she has noticed mild rectal bleeding and multiple bruises over the extremities with minimal
trauma. She is comfortable appearing with normal vital signs and is not orthostatic. You ordered a stat
CBC and PT/INR which revealed a mildly decreased Hgb at 11 g/dL and an elevated INR of 7. Which of
the following would be the most appropriate intervention? 

A) Fresh frozen plasma 

B) Withhold warfarin

C) Intravenous vitamin K 

D) Reduce warfarin dose 

E) Oral vitamin K

Answer: E

20. Which of the following produces the greatest increase in bone mineral density (BMD) in patients
with osteoporosis? 

A) Estrogen 

B) Calcitonin 

C) Alendronate 

D) Teriparatide 

E) Raloxifene

Answer: D

21. A 34-year-old woman was found to have a 2-cm right thyroid nodule at the time of a well woman
examination. The remainder of the thyroid was palpably normal and there were no lymph nodes
palpable. There was no history of thyroid disease or radiation therapy to her head or neck. She was
clinically euthyroid. Thyroid-stimulating hormone (TSH) was normal. Which of the following tests would
be the most useful in establishing a specific diagnosis? 
A) Ultrasound of the thyroid 

B) Nuclear scan of the thyroid 

C) Thyroid antibody studies 

D) Fine needle aspiration of the nodule 

E) CT of the neck

Answer: D

22.  A slim, healthy 30-year-old woman is scheduled for a dental prosthodontic procedure and was sent
for medical evaluation of a known history of mitral valve prolapse (MVP). The patient is a highly active
individual and denies palpitations, chest pain, or shortness of breath. She admits to having a family
history of heart disease, notably her father, who had died of a heart attack in his forties, and her
mother, who had mitral valve prolapse. On physical examination, the patient is comfortable and has
normal vital signs. Auscultation of the heart reveals a normal S1 and S2 and a prominent midsystolic
click, which is accentuated in the standing position. No systolic murmur is appreciated. What is your
overall assessment and plan for this patient?

A) Get an echocardiogram to evaluate mitral valve motion and blood flow prior to clearing her for the
procedure

B) Prescribe empiric antibiotics for endocarditis prophylaxis and clear her for the procedure

C) Get a cardiology consultation prior to medical clearance because the patient has a significant family
history of heart disease

D) Clear her for the procedure without endocarditis prophylaxis

E) Clear her for the procedure with endocarditis prophylaxis

Answer: D 

23. A 35-year-old man complains of chest pain, which began following the use of cocaine 30 minutes
prior to arrival. The patient describes severe substernal chest pressure, radiating to the left arm and jaw.
It is associated with shortness of breath. Initial vital signs: temperature 100.4°F, heart rate 120/min,
respiratory rate 20/min, blood pressure 185/100 mm Hg, pulse oximetry 98% on room air. An ECG is
consistent with acute myocardial infarction. A drug which is contraindicated is:
A) Oxygen.

B) Aspirin.

C) Lorazepam.

D) Bisoprolol.

E) Morphine

Answer: D

24. A 55-year-old man with a history of alcoholism complains of more than a month of malaise, low-
grade fever, and a productive cough with greenish sputum tinged with blood. Examination shows
periodontal disease with bad breath and clubbing of fingers. On chest x-ray, there is a 2 cm cavity with
an air–fluid level in the posterior segment of the right upper lobe. Sputum smear shows many
neutrophils and a variety of bacteria. Appropriate treatment includes:

A) Isolate the patient and initiate a four-drug antituberculosis treatment.

B) Start intravenous clindamycin.

C) Refer the patient to a dentist for periodontal care.

D) Schedule a bronchoscopy for the next day.

E) Start administration of methicillin and tobramycin

Answer: B

25. A previously healthy 54-year-old man presents to the emergency department complaining of chest
pain. His ECG shows an acute inferior wall myocardial infarction. His blood pressure is 90/60 mm Hg. On
physical examination, he has jugular vein distention and clear lungs. You should treat him immediately
with which of the following:

A) Intravenous fluids.

B) Norepinephrine.

C) Dopamine.
D) Nesiritide.

E) Nitroprusside

Answer: A

26. Inhalant abusers (“huffers” and “baggers”) are at risk for:

A) Ventricular dysrhythmias.

B) Acute lung injury.

C) Hypoglycemia.

D) Acidosis.

E) Kidney failure

Answer: A

27. A 37-year-old woman has a 2-week history of intermittent headache and general malaise. Over the
last 24 hours, she has developed back pain, hematuria, vomiting, fever, and confusion. She denies
recent travel or insect bites. There is no history of dysuria, urgency, frequency, or kidney stones. Her
oral temperature is 38°C and her heart rate is 100/min. Physical findings include pale conjunctivae,
borderline tachycardia, bilateral costovertebral tenderness, and several purpuric skin lesions. Her urine
dipstick is strongly positive for hemoglobin, but negative for nitrites and leukocyte esterase. The test
that will most likely reveal the correct diagnosis is:

A) Intravenous pyelogram.

B) CBC with differential and peripheral smear.

C) Complete urinalysis with microscopic examination.

D) Blood cultures.

E) Liver function tests

Answer: B
28. A 52-year-old man complains of severe right knee pain. He recalls no trauma, but attended a wine
tasting party last weekend. He denies past medical history and is afebrile with normal vital signs.
Physical examination reveals a swollen, red, painful right knee. Radiograph shows an effusion, but no
bony erosions. You perform arthrocentesis and obtain 27 mL of cloudy straw-colored fluid, which you
send to the laboratory for studies. The cell count is 50,000 WBCs/mm3 with 85% PMNs, glucose 120
mg/dL (serum 130), and protein 3.5 g/dL. The Gram stain shows numerous WBCs, but is negative for
organisms. Evaluation for crystals shows numerous needle-shaped crystals with negative birefringence.
A good treatment plan would be:

A) Oral colchicine 0.5 mg every hour until symptoms abate, GI toxicity develops or the maximum dose of
6 mg in 24 hours is reached. 

B) Subcutaneous colchicine 1 mg plus oral probenecid 250 mg bid for 1 week.

C) Sublingual colchicine 2 mg followed by oral steroids tapered over 5 days.

D) Intra-articular triamcinolone injection.

E) Oral allopurinol 500 mg tid for 1 week

Answer: A

29. A 35-year-old woman presents to her gynecologist with complaints of burning on urination for the
past 2 days. Dipstick test of her urine demonstrates marked positivity for leukocyte esterase, but no
reactivity for nitrite. Urine culture later grows out large numbers of organisms. Which of the following
bacteria are most likely to be responsible for this patient's infection?

A) Enterobacter sp.

B) Enterococcus faecalis

C) Escherichia coli

D) Klebsiella pneumoniae

E) Pseudomonas aeruginosa

Answer: B
30. A 23 year old single woman referred with 3-month history of weight loss and heat intolerance. On
examination pulse is 120 beat/min regular, blood pressure 120/72 mmHg, weight 58 kg, height 165 cm.
she had diffuse goiter with bilateral exophthalmos. Investigations: serum free T4 3.9 ng/dl (N: 0.9-2.0),
serum TSH 0.001 mu/l (N: 0.4-4.0), TSH receptor antibody 8 U/L(N:<2). She was started on methimazole
30 mg/day. Six weeks later she developed severe migratory arthralgia associated with the presence of
palpable purpura over both legs and dermal infarcts in the finger tips. Laboratory studies shows ESR 54
mm/h, urine showed microscopic hematuria, ANCA titer was 1:640, antimyeloperoxidase antibodies
112U/l (N:0-9), antiproteinase 3 antibodies 6.4 U/l (N:0-3.5).

The best action is:

A) Continue methimazole and add NSAID.

B) Continue methimazole and add prednisolone 40 mg per day.

C) Substitute propylthiouracil for methimazole.

D) Refer the patient for definitive therapy with radioactive iodine.

E) No intervention

Answer: D

31. A 16-year-old girl consults you because of progressive pallor and poor exercise tolerance. She has
had dysfunctional uterine bleeding for several months. Her haemoglobin is 6.5 g/dl. Which one of the
following would be inconsistent with iron-deficiency anaemia in this girl?

A) Thrombocytosis

B) Low serum iron

C) Low serum ferritin

D) Reticulocyte percentage 10%

E) Transferrin saturation 9%

Answer: D
32. A 48-year-old auto mechanic presents to the clinic with complaints of many years of “pins and
needles” in his left hand that initially occurred only while working but have worsened substantially. He
claims the pain wakes him almost every night. On examination, marked weakness and wasting of the left
hand muscles are evident. What is the patient’s most likely diagnosis?

A) Amyotrophic lateral sclerosis

B) Angina

C) Carpal tunnel syndrome

D) Multiple sclerosis

E) Myasthenia gravis

Answer: D

33. Three months following laparoscopic cholecystectomy, a 50-yearold woman presents with right
upper abdominal quadrant pain, excessive flatulence and intolerance to fatty food. Abdominal
ultrasound and ERCP fail to detect any residual stone or biliary stricture. What is the cause of this
woman’s presentation?

A) Small common bile duct stones

B) Cholangiocarcinoma

C) Peri-ampullary adenocarcinoma

D) Post-cholecystectomy syndrome

E) Choledochal cyst

Answer: D

34. All of the following clinical findings are consistent with severe mitral stenosis except

A) atrial fibrillation

B) opening snap late after S2

C) pulmonary vascular congestion


D) pulsatile liver

E) right-ventricular heave

Answer: B

35. A 50-year-old man has a blood pressure of 160/105 mm Hg, which is repeatedly confirmed to be
high. He has migraine without aura for which he takes sumatriptan during pain episodes. Which is the
best antihypertensive medication for this patient to be given initially?

A) Amlodipin 

B) Propranolol

C) Hydrochlorothiazide

D) Losartan

E) Alpha methyldopa

Answer: B

36. All of the following represent examples of hypothalamic-pituitary negative feedback EXCEPT:

A) Cortisol on the CRH-ACTH axis

B) Gonadal steroids on the GnRH-LH/FSH axis

C) IGF-1 on the growth hormone–releasing hormone (GHRH)-GH axis

D) Renin-angiotensin-aldosterone axis

E) Thyroid hormones on TRH-TSH axis

Answer: D

37. You are investigating the cause for a patient’s anemia. He is a 50-year-old man who was found to
have a hematocrit of 25% on routine evaluation. His hematocrit was 47% 1 year ago. Mean corpuscular
volume is 80, mean corpuscular hemoglobin concentration is 25, mean corpuscular hemoglobin is 25.
Reticulocyte count is 5%. Review of the peripheral blood smear shows marked numbers of
polychromatophilic macrocytes. Ferritin is 340 μg/L. What is the cause of this patient’s anemia?

A) Defective erythroid marrow proliferation

B) Extravascular hemolysis

C) Intravascular hemolysis

D) Iron-deficiency anemia

E) Occult gastrointestinal bleeding

Answer: A

38. A 23-year-old Japanese man attends a party where he drinks three glasses of wine. In a short period
of time, he develops facial erythema and experiences severe facial flushing. Which of the following is the
most likely diagnosis?

A) Alcohol dehydrogenase deficiency

B) Glucoronyl tranferase deficiency

C) Aldehyde dehydrogenase deficiency

D) Angioedema

E) Photosensitivity reaction

Answer: C

39. A 46-year-old woman with mitral stenosis presents with dense right-sided hemiplegia and global
aphasia. Her radial pulse is rapid and irregular. Brain CT scan shows cerebral infarction and she is
admitted to the neurology ward to receive treatment for embolic stroke. Which one of the following
arteries is likely to be occluded?

A) Main stem of the left middle cerebral artery

B) Lower posterior branch of the right middle cerebral artery


C) Left anterior cerebral artery

D) Top of the basilar artery

E Left vertebral artery

Answer: A

40. Which of the following is consistent with a diagnosis of subacute thyroiditis?

A) A 38-year-old female with a 2-week history of a painful thyroid, elevated T4, elevated T3, low TSH,
and an elevated radioactive iodine uptake scan

B) A 42-year-old male with a history of a painful thyroid 4 months ago, fatigue, malaise, low free T4, low
T3, and elevated TSH

C) A 31-year-old female with a painless enlarged thyroid, low TSH, elevated T4, elevated free T4, and an
elevated radioiodine uptake scan

D) A 50-year-old male with a painful thyroid, slightly elevated T4, normal TSH, and an ultrasound
showing a mass

Answer: B

41. A 65-year-old obese man presents with recurrent episodes of cough. His wife said his coughing
attacks disturb her sleep almost every night. He noticed that his voice is hoarse in the morning. He has
never smoked and has no history of cardiac or respiratory problems. There is no history of allergy and
the chest X-ray is normal. The procedure most likely to yield crucial diagnostic information is 

A) Pulmonary function tests 

B) 24-hour pH monitoring of the lower oesophagus 

C) High-resolution CT scan of the chest 

D) Barium swallow 

E Indirect laryngoscopy
Answer: B

42. A 38-year-old man has a six-month history of diarrhoea, abdominal pain and tenesmus. Flexible
sigmoidoscopy reveals proctitis. A biopsy specimen shows acute and chronic inflammation. Each of the
following statements is true about this disorder, except 

A) There is an increased risk of gastrointestinal malignancy 

B) Other family members are likely to develop the same illness 

C) Steatorrhoea is likely to develop as a result of pancreatic insufficiency 

D) The likelihood of renal stones will increase 

E) The likelihood of gallstones will increase

Answer: C

43. A 56-year-old man is referred with dyspnoea and cough productive of putrid green sputum of four
weeks duration. The chest X-ray shows cavity formation in the left lower lobe. He has advanced motor
neurone disease. This clinical picture is most likely to be caused by infection with 

A) Legionella pneumophila 

B) Mycoplasma pneumoniae 

C) Streptococcus pneumoniae 

D) Anaerobic bacteria

E) Common cold virus

Answer: D

44. A 50-year-old publican is referred with anuria. The initial renal function test showed the urea at 45
mmol/I and the creatinine at 480 μmoI/l. A urine sample from a catheter was strongly positive for
myoglobin. This clinical picture can develop as a result of each of the following clinical situations except 

A) Alcohol ingestion 
B) Hypokalaemia 

C) Barbiturate overdose 

D) Hypophosphataemia 

E) Hypocalcaemia

Answer: E

45. A 75-year-old woman is admitted with left lower lobe pneumonia. She was given a course of
cephalosporin antibiotics and she improved. However, at the end of the course she started to have
diarrhoea, passing a watery stool. Investigations confirmed the diagnosis of pseudomembranous colitis.
Each of the following oral medications is beneficial in treating this condition, except 

A) Vancomycin 

B) Bacitracin 

C) Colestyramine 

D) Metronidazole  

E) Gentamicin

Answer: E

46. A 38-year-old woman has dyspnoea on exertion and orthopnoea. Examination reveals a loud first
heart sound, a diastolic rumble, and a large v wave in the jugular pulse. The most likely diagnosis is

A) Mitral stenosis and tricuspid regurgitation 

B) Isolated mitral stenosis 

C) Aortic regurgitation 

D) Mitral regurgitation and tricuspid regurgitation 

E) Tricuspid regurgitation
Answer: A

47. A 40-year-old man has a six-month history of increasing shortness of breath and wheeze. The full
blood count shows an increased absolute eosinophil count. Chest X-ray and a CT scan of the chest show
right apical bronchiectasis. Which one of the following is the most likely diagnosis?

A) Loeffler's syndrome 

B) Allergic bronchopulmonary aspergillosis 

C) Polyarteritis nodosa 

D) Nitrofurantoin hypersensitivity 

E) Strongyloides stercoralis infection

Answer: B

48. A 22-year-old woman has a three-month history of fatigue of insidious onset, poor appetite and
athralgia. She has no previous history of liver disease. She takes thyroxine replacement therapy, Physical
examination reveals spider naevi and hepatomegaly, The ALT is five-times higher than the normal. Her
disease is most likely be associated with 

A) Antimitochondrial antibodies 

B) Low serum caeruloplasmin levels 

C) Anti-parietal cell antibodies 

D) Anti-smooth muscle antibodies 

E) Hepatitis B surface antigen (1-HbsAg )

Answer: D

49. A 35-year-old woman has slurred speech and a progressive gait disturbance. Magnetic resonance
imaging (MRI) shows several focal abnormalities in the periventricular areas, normal sized ventricles and
no space occupying lesion. The cerebellum was also normal. The most likely diagnosis is 
A) Motor neurone disease 

B) Friedreich's ataxia 

C) Multiple sclerosis (MS) 

D) Syphilis 

E) Huntington's disease

Answer: C

50. An 85-year-old man is referred with a three-month history of dysphagia. Barium swallow showed a
filling defect in the oesophagus. Gastro-oesophagoscopy and biopsy confirmed the presence of
squamous cell carcinoma of the oesophagus. Each of the following conditions might be associated with
this tumour, except 

A) Achalasia 

B) Smoking 

C) Tylosis 

D) Barrett's oesophagus 

E) Head and neck cancer

Answer: D

51. An 84-year-old woman presented with tiredness. On examination, she was anaemic but had no
palpable splenomegaly. Investigations revealed a haemoglobin of 9.7 g/dL (11.5 - 16.5). She was
commenced on oral iron therapy for one month and her haemoglobin remained unchanged. Further
investigations revealed:

MCV 102 fL (80 - 96)

blood film marked anisopoikilocytosis

serum ferritin 70 ug/L(15 - 300)

Vitamin B12 280 ng/L (160 - 760)


red cell folate 230 ug/L (160 - 640)

serum urea 9.1 mmol/L (2.5-7.5)

serum creatinine 150 umol/L (60 - 110)

What is the most likely diagnosis?

A ) aplastic anaemia

B ) anaemia due to renal disease

C ) hypothyroidism

D ) iron deficiency anaemia

E ) sideroblastic anaemia

Answer: E

52. A 29-year-old male presents with symptoms of severe gastro-oesophageal reflux. Which one of the
following is most useful in assessing the role of surgery?

A ) cardiac sphincter manometry

B ) gastric emptying study

C ) intragastric pH monitoring off therapy

D ) oesophageal motility study

E ) oesophageal pH monitoring on therapy

Answer: D

53. An 80 year old man with a 5 year history of diet controlled type 2 diabetes mellitus presents with a
one month history of cough and weight loss. He was a non-smoker and had difficulty expectorating.
Investigation revealed a HbA1c of 7% but his chest X-ray showed a cavitating left apical shadow. Which
of the following investigations would be most useful in establishing the cause of this lesion?

A ) bronchoscopy
B ) CT scan of the chest

C ) Gastric aspirate for acid-fast bacilli

D ) Percutaneous lung biopsy 

E ) Sputum for acid-fast bacilli

Answer: A

54. A 68 year old male diagnosed with nephritic syndrome receives steroid therapy without benefit. His
investigations show an albumin of 20 g/L (37 - 49), Total cholesterol of 12 mmol/l, dipstick urinanalysis
reveals +++ protein and a renal biopsy shows focal segmental glomerulosclerosis. Which one of the
following is most likely to preserve renal function?

A ) dietary salt restriction

B ) low dietary protein intake

C ) ramipril

D ) simvastatin

E ) warfarin

Answer: C

55. Which of the following is characteristically inherited in an autosomal recessive manner?

A ) Achondroplasia

B ) Adult polycystic kidney disease

C ) C1 esterase deficiency

D ) Familial hypercholesterolaemia

E ) Friedreich's ataxia

Answer: E
56. A clinical trial assessing a new lipid lowering therapy for stroke allocates 1000 patients to active
treatment and another 1000 patients to placebo. Results demonstrate that number needed to treat
(NNT) is 20 for the prevention of the primary end-point. Which of the following best describes the
results?

A ) 20 patients in the treatment group were protected from stroke.

B ) 20 extra patients in the placebo group had a stroke

C ) For 1000 patients treated with active therapy, there would be 20 fewer strokes

D ) For 1000 patients treated with active therapy, there would be 50 fewer strokes.

E ) For every 1000 patients treated with active therapy there would be 100 fewer strokes

Answer: D

57. A 70 year old male presents with haemoptysis. Bronchoscopy reveals a tumour in the proximal right
main bronchus. Which of the following is a contraindication to radical radiotherapy?

A) Adenocarcinoma

B) FEV1 of 25% predicted

C) Involvement of the pulmonary artery

D) Ischaemic heart disease

E) Superior vena caval obstruction

Answer: B

58. Which of the following statements concerning iron metabolism is correct?

A) Approximately 0.1% of body iron circulates in the plasma

B) Approximately 90% of dietary iron is absorbed in the intestine

C) The main route of excretion is the liver


D) The serum ferritin concentration is reduced characteristically following surgery

E) The transferrin content of intestinal mucosal cells is high when body iron stores are high

Answer: A

59. A 35 year-old woman with type 1 diabetes mellitus presents for annual assessment. Which one of
the following features on fundoscopy would require urgent referral to anophthalmologist?

A) asteroid bodies

B) hard exudates in the macular region

C) intraretinal microvascular abnormalities

D) scattered microaneurysms

E) soft exudates

Answer: B

60. A 34-year-old man presented for an insurance medical. He was symptom free, but clinical
examination suggested a small ventricular septal defect. Which one of the following findings was most
likely to have been present?

1) An early diastolic murmur

B) A short systolic murmur at the left sternal edge

C) A systolic murmur maximal at the apex

D) A systolic murmur with thrill at the left sternal edge

E) Fixed splitting of the second heart sound

Answer: D

 
MULTIPLE CHOICE QUESTIONS-2

1. A 60-year-old woman presented with a small right pupil, right ptosis and impaired sweating over the
ipsilateral forehead. Sweating on the rest of the face was unaffected. Where is the most likely site of this
lesion?

A) Cervical spinal cord

B) Common carotid artery

C) Hypothalamus

D) Internal carotid artery 

E) Lateral medulla

Answer: D

2. A 53-year-old man presented with hypertension of 150/110 mmHg. He is generally asymptomatic and
has no previous medical history of note. He is a smoker of 5 cigarettes daily and drinks modest
quantities of alcohol. He takes no prescribed medications. Examination reveals a BMI of 33.5 kg/cm2 but
nil else. Investigations:

Serum sodium 146 mmol/l (NR 133-145); Serum potassium 3.2 mmol/l (NR 3.5 - 5); Urinary potassium
excretion 42 mmol/l (NR less than 30)

What is the likely diagnosis?

A) adrenocortical adenoma

B) Bartter's syndrome

C) Liddle's syndrome

D) liquorice ingestion

E) pheochromocytoma

Answer: A
3. A previously healthy 54-year-old man presents to the emergency department complaining of chest
pain. His ECG shows an acute inferior wall myocardial infarction. His blood pressure is 90/60 mm Hg. On
physical examination, he has jugular vein distention and clear lungs. You should treat him immediately
with which of the following:

A) Intravenous fluids.

B) Norepinephrine.

C) Dopamine.

D) Nesiritide.

E) Nitroprusside.

Answer: A

4. While caring for a patient with suspected pericardial tamponade, you would expect to find: 

A) Equalization of right and left ventricular pressures. 

B) Isolated systolic hypertension. 

C) A hyperdynamic precordium. 

D) Poor R wave progression on the 12-lead electrocardiogram. 

E) Mitral regurgitation.

Answer: A

5. A female patient aged 30 has a 5 years history of difficulty getting upstairs and out of a low chair and
mild upper limb weakness but no pain. There is no family history. She presented with severe type 2
respiratory failure. EMG showed evidence of myopathy. 

The most likely diagnosis is:

A) Polymyositis
B) Inclusion body myositis

C) Acid maltase deficiency

D) Miller-Fisher Syndrome

E) Lambert-Eaton Myasthenic syndrome

Answer: C

6. A teenage girl presents with Guillain-Barre syndrome. Her weakness continues to worsen after
admission to hospital. Which of the following should be used to monitor her? 

A) Arterial blood gases

B) Chest expansion size

C) FEV1/FVC ratio

D) PEFR

E) Vital capacity

Answer: E

7. A 19-year-old woman presents to the clinic having had 5 blackouts over the last year, all while she is
standing up. She gets warnings of blurred vision, nausea, feeling hot. She had been witnessed twice to
have jerking of all limbs while she is unconscious. The attacks last 30-60 seconds. She recovers quickly
after the attacks. She has never bitten her tongue or sustained any injuries. Physical examination and an
ECG are normal. Her grandmother and sister suffer from epilepsy. Which of the following investigations
is the most appropriate?

A) EEG

B) 24 hour ECG recording

C) CT brain

D) ECHO

E) Tilt table test


Answer: E

8. A 21-year-old female presented with a sudden onset of left sided head and neck pain. 24 hours later
she presents with sudden onset of right hemiparesis, facial weakness and homonymous hemianopia and
left Horner's syndrome. A CT brain showed a left middle cerebral artery territory infarction. The most
likely diagnosis is: 

A) Cardiac embolism

B) Migrane 

C) Left carotid artery dissection

D) Antiphospholipid syndrome

E) Systemic vasculitis

Answer: C

9. A female presents with headache, lethargy and weight loss. Which of the following would make the
diagnosis of giant cell arteritis unlikely?

A) A normal ESR

B) Bilateral headache

C) Non-tender temporal arteries

D) Papilloedema on fundoscopy

E) The patient is 50 years old

Answer: D

10. A 60-year-old man presents with a 2 month history of progressive confusion, gait disturbance, and
urinary incontinence. Examination reveals mild global cognitive dysfunction and gait ataxia. CT brain
shows enlarged ventricles with no evidence of obstruction to CSF outflow. Lumbar puncture reveals
normal CSF pressure and constituents. Which one of the following management steps is likely to be
most helpful? 

A) CSF drainage via repeated lumbar puncture

B) EEG

C) Intracranial pressure monitoring

D) MRI brainstem

E) Serum B12 and folate levels

Answer: A

11. A 72-year-old man presents with an acutely painful right knee. On examination, he had a
temperature of 37°C with a hot, swollen right knee. Of relevance amongst his investigations, was his
white cell count which was 12.6 x109/l and a knee X-ray revealed reduced joint space and calcification of
the articular cartilage. Culture of aspirated fluid revealed no growth. What is the most likely diagnosis?

A) Gout

B) Psoriatic monoarthropathy

C) Pseudogout

D) Rheumatoid arthritis

E) Septic arthritis

Answer: C

12. A 40-year-old female who has been prescribed thyroid replacement therapy has routine thyroid
function tests. On examination she appeared clinically euthyroid with no abnormal findings. Her TFTs
revealed: TSH 3.2 mU/L (0. 35 - 5.0) Total T4 20 nmol/L (55 - 144) free T4 2.6 pmol/L (9 - 24) Total T3 2.5
nmol/L (0. 9 - 2.8).

Which one of the following statements is correct?

A) Her thyroid hormone replacement is adequate.


B) Investigation of pituitary function is required.

C) She has tertiary hypothyroidism.

D) She has thyroiditis.

E) She has sick euthyroid syndrome.

Answer: A

13. Which of the following viral causes of acute hepatitis is most likely to cause fulminant hepatitis in a
pregnant woman?

A) Hepatitis A

B) Hepatitis B

C) Hepatitis C

D) Hepatitis D

E) Hepatitis E

Answer: E

14. A 35-year-old healthy woman has a faint systolic murmur on physical examination. An
echocardiogram is performed, and she is found to have a bicuspid aortic valve. In explaining the
meaning of this finding to her, the most appropriate statement is that?

A) An aortic valve replacement is eventually likely to be required

B) Other family members are likely to have the same condition

C) She should be treated with a cholesterol lowering agent

D) The problem resulted from past injection drug usage

E) This is one manifestation of an underlying autoimmune condition.

Answer: A
15. A 54 year old man presents with central crushing chest pain. Examination is normal. 12-lead ECG
shows ST segment elevation in leads II, III, aVF and ST depression in V1, V2 and V3. Which coronary
artery is occluded?

A) Circumflex.

B) Left anterior descending.

C) Left main stem.

D) Obtuse marginal.

E) Right coronary artery.

Answer: B

16. Primary hyperparathyroidism may occur in association with the following conditions 

A) Chronic renal failure

B) Vitamin D deficiency

C) Gastrinoma

D) Autoimmune polyendocrine syndrome

E) Sjogren's syndrome

Answer: C

17. A 57-year-old man with a history of diabetes mellitus and chronic kidney disease with a baseline
creatinine of 1.8 mg/dL undergoes cardiac catheterization for acute myocardial infarction. He is
subsequently diagnosed with acute kidney injury related to iodinated contrast. All of the following
statements are true regarding his kidney injury EXCEPT:

A) Fractional excretion of sodium will be low.

B) His creatinine is likely to peak within 3–5 days.


C) His diabetes mellitus predisposed him to develop contrast nephropathy.

D) Transient tubule obstruction with precipitated iodinated contrast contributed to the development of
his acute kidney injury.

E) White blood cell casts are likely on microscopic examination of urinary sediment.

Answer: E

18. A 25 year-old male presents with an eight week history of difficulty walking. On examination he had
increased tone and pyramidal weakness of the right leg. There was impairment of pinprick sensation in
the left leg up to the groin. Which one of the following is the cause of these signs? 

A) A central cauda equina lesion.

B) A cervical spinal cord lesion.

C) A lesion at the foramen magnum.

D) A right-sided thoracic spinal cord lesion.

E) Bilateral cerebral hemisphere lesions.

Answer: D

19. An 81 -year-old man admitted with a stroke becomes increasingly drowsy after receiving nasogastric
feeding for five days. Which biochemical abnormality is the most likely cause of his drowsiness?

A) Hyperglycemia.

B) Hypermagnesemia.

C) Hypernatremia.

D) Hypocalcemia.

E) Hypophosphatemia

Answer: C
20. One of the nurses working on the Care of the Elderly ward sustains a needlestick injury while taking
blood from a patient. What is the most appropriate immediate management?

A) Administer prophylactic hepatitis B immunoglobulin regardless of vaccine status.

B) Exclude the nurse from performing exposure-prone procedures for three months until a negative HIV
antibody test has been obtained.

C) Immediately take the nurse’s blood to test for antibodies to hepatitis B, hepatitis C and human
immunodeficiency viruses.

D) Promptly administration of antiretroviral therapy.

E) Wash the wound with soap under running water.

Answer: E

21. A 65-year-old male with left ventricular systolic dysfunction was dyspneic on climbing stairs but not
at rest. The patient was commenced on Ramipril and Furosemide.

Which one of the following drugs would improve the patient’s prognosis further?

A) Amiodarone

B) Digoxin

C) Diltiazem

D) Metoprolol

E) Isosorbide dinitrate.

Answer: D

22. All of the following are recognized feature of pulmonary embolism except?

A) S1Q3T3

B) An increase in serum troponin levels


C) An arterial pH greater than 7.2

D) Increased PCO2 on room airk,m

E) Positive D-dimer levels

Answer: D

23. Which of the following concerning diabetic retinopathy is correct?

A) Is unusual in Type 2 diabetic patients.

B) Improved glycemic control is more effective than hypertensive control in reducing progression of
disease.

C) Normal visual acuity is seen in proliferative retinopathy.

D) Progression may be reduced with statin therapy.

E) Soft exudates are a feature of background retinopathy.

Answer: C

24. A 52 year old female presents with tiredness. There are no specific abnormalities noted on
examination, but investigations reveal a T4 of 21.1 (NR 9.8 - 23), a T3 of 5.2 pmol/l (NR 3.3 - 5.5) and a
TSH of 0.05 mU/l (NR 0.1 - 5 mU/l). Thyroid autoantibody titers are all undetectable. These results
suggest a diagnosis of: 

A) De Quervain's thyroiditis.

B) Sick euthyroid syndrome.

C) Solitary toxic nodule.

D) Grave's disease.

E) Hashimoto's thyroiditis.

Answer: C
25. A 46-year-old woman with a 10-year history of primary progressive multiple sclerosis is admitted to
the hospital for surgical debridement and wound management of a sacral pressure ulcer. The patient is
bedbound and is cared for at home by her husband. Her medications include sertraline, baclofen, and
oxybutynin. The remainder of the medical history is noncontributory. On physical examination, she is
thin, in no distress, and has normal vital signs. There are contraction deformities of the lower
extremities, and a 6- × 8-cm sacral ulcer that extends to the fascia, with minimal purulent exudates and
no evidence of cellulitis. Laboratory studies include a hematocrit of 34%, leukocyte count of 15,000/µL
(15 × 109/L) with 80% neutrophils, and a platelet count of 425,000/µL (425 × 109/L). Subcutaneous
prophylactic unfractionated heparin is administered. Six days after initiation of heparin, her platelet
count decreases to 210,000/µL (210 × 109/L), and on the ninth day after therapy, the platelet count has
decreased to 95,000/µL (95 × 109/L). Which of the following is the most appropriate next step in the
management of this patient?

A) Discontinue unfractionated heparin.

B) Discontinue sertraline, baclofen, and oxybutynin.

C) Discontinue unfractionated heparin and begin therapy with a direct thrombin inhibitor.

D) Discontinue unfractionated heparin and begin low-molecular-weight heparin.

E) Discontinue unfractionated heparin and begin novel oral anticoagulants.

Answer: C

26. A 75 year old man has a history of Chronic Lymphocytic Leukemia. He has had treatment with
several courses of chemotherapy and has now been admitted to hospital with pneumonia. His past
medical history revealed that he had suffered several previous upper respiratory tract infections over
the previous six months.

Which of the following components of his immune system is likely to be deficient?

A) complement

B) immunoglobulin G

C) macrophages

D) mast cells

E) T lymphocytes
Answer: B

27. A 42-year-old man presented to the hospital with right upper quadrant pain. He was found to have
multiple masses in the liver that were found to be malignant on H&E staining of a biopsy sample. Your
initial history, physical examination, and laboratory tests, including prostate-specific antigen, are
unrevealing. Lung, abdominal, and pelvic CT scans are unremarkable. He is an otherwise healthy
individual with no chronic medical problems. Which immunohistochemical markers should be obtained
from the biopsy tissue?

A) α-Fetoprotein

B) Cytokeratin

C) Leukocyte common antigen

D) Thyroglobulin

E) Thyroid transcription factor 1

Answer: B

28. A 47-year-old man is evaluated for 1 year of recurrent episodes of bilateral ear swelling. The ear is
painful during these events, and the right ear has become floppy. He is otherwise healthy and reports no
illicit habits. He works in an office and his only sport is tennis. On examination, the left ear has a beefy
red color, and the pinna is tender and swollen; the earlobe appears minimally swollen but is neither red
nor tender. Which of the following is the most likely explanation for this finding?

A) Behçet's syndrome.

B) Cogan's syndrome.

C) Hemoglobinopathy.

D) Recurrent trauma.

E) Relapsing polychondritis.

Answer: E
29. One of the following pulmonary function test results is with the respiratory disorder chronic
obstructive pulmonary disease.

A) Increased total lung capacity (TLC), decreased vital capacity (VC), decreased FEV1/FVC ratio.

B) Decreased TLC, decreased VC, decreased residual volume (RV), increased FEV1/FVC ratio, normal
maximum inspiratory pressure (MIP).

C) Decreased TLC, increased RV, normal FEV1/FVC ratio, decreased MIP.

D) Normal TLC, normal RV, normal FEV1/FVC ratio, normal MIP.

E) Decreased TLC, normal RV, normal FEV1/FVC ratio, increased MIP.

Answer:A

30. A 23 year old single woman referred with 3-month history of weight loss and heat intolerance. On
examination pulse is 120 beat/min regular, blood pressure 120/72 mmHg, weight 58 kg, height 165 cm.
she had diffuse goiter with bilateral exophthalmos. Investigations: serum free T4 3.9 ng/dl (N: 0.9-2.0),
serum TSH 0.001 mu/l (N: 0.4-4.0), TSH receptor antibody 8 U/L(N:<2). She was started on methimazole
30 mg/day. Six weeks later she developed severe migratory arthralgia associated with the presence of
palpable purpura over both legs and dermal infarcts in the finger tips. Laboratory studies shows ESR 54
mm/h, urine showed microscopic hematuria, ANCA titer was 1:640, antimyeloperoxidase antibodies
112U/l (N:0-9), antiproteinase 3 antibodies 6.4 U/l (N:0-3.5). The best action is:

A) Continue methimazole and add NSAID.

B) Continue methimazole and add prednisolone 40 mg per day.

C) Substitute propylthiouracil for methimazole.

D) Refer the patient for definitive therapy with radioactive iodine.

E) No intervention.

Answer: E
31. A 34-year-old man comes for neurologic consultation because of paroxysmal episodes of speech
difficulty that have occurred recently. The best way to distinguish whether these are seizures or other
types of events is:

A) History

B) Neurologic examination

C) Brain MRI

D) Routine EEG

E) Empiric anticonvulsant trial.

Answer: A

32. A 23-year-old woman is diagnosed with a lower extremity deep venous thrombosis. Which of the
following medical conditions represents a contraindication to therapy with low-molecular-weight
heparin (LMWH)?

A) Pregnancy

B) Obesity

C) Dialysis-dependent renal failure

D) Uncontrolled diabetes mellitus

E) Jaundice.

Answer: C

33. All the following forms of glomerulonephritis (GN) have associated normal serum complement C4
levels EXCEPT:

A) lupus nephritis stage IV

B) poststreptococcal GN

C) hemolytic-uremic syndrome
D) membranoproliferative GN type II

E) endocarditis-associated GN

Answer: A

34. A 60-year-old man attends clinic because of hypertension. His BP in clinic is 70/90 mmHg and his
echocardiogram shows mild LVH and mild LA dilatation. He is not diabetic and has no other medical
history of note.Which one of the following medications is most effective in preventing AF?

A) ACE inhibitors

B) Beta-blockers

C) Calcium-channel antagonists

D) Diuretics

E) Alpha-blocker

Answer: A

35. A 35-year-old male is seen in the clinic for evaluation of infertility. He has never fathered any
children, and after 2 years of unprotected intercourse his wife has not achieved pregnancy. Sperm
analysis shows a normal number of sperm, but they are immotile. Past medical history is notable for
recurrent sinopulmonary infections, and the patient recently was told that he has bronchiectasis. Chest
radiography is likely to show which of the following?

A) Bihilar lymphadenopathy

B) Bilateral upper lobe infiltrates

C) Normal findings

D) Situs inversus

E) Water balloon–shaped heart

Answer: D
36. All of these findings are consistent with a chronic unilateral urinary tract obstruction EXCEPT:

A) anemia

B) dysuria

C) hypertension

D) pain with micturition

E) pyuria

Answer: D

37. Which of the following is more likely to present as a myopathic disorder of gastrointestinal motility
rather than as a neuropathic disorder?

A) Parkinson disease

B) Diabetes mellitus

C) Multiple sclerosis

D) Anticholinergic medication use

E) Dermatomyositis

Answer: E

38. Regarding ophthalmic complications in a patient with diabetes, which ONE of the following
statements is INCORRECT?

A. There is a higher risk of vitreous haemorrhage than in the general population

B. Ocular haemorrhage is common in patients with diabetic retinopathy, following thrombolysis for
acute myocardial infarction

C. Diabetic retinopathy may cause retinal detachment

D. Measurement of intraocular pressure is important because glaucoma is more common in diabetics


Answer: B

39. Carcinoid tumours of the foregut may be associated with which of the following 

A) Cushing's syndrome

B) hypercalcemia

C) pellagra

D) pulmonary hypertension

E) carcinoid syndrome

Answer: A

40. A 69-year-old woman with rheumatoid arthritis presented with swollen ankles. She was diagnosed
as having rheumatoid arthritis over 18 years ago and had been relatively well controlled on non-
steroidal antiinflammatory drugs until six months ago, when her joint pains and swelling required the
addition of penicillamine to control her symptoms. The patient had a past history of hypertension, for
which she took bendroflumethiazide. On examination she had symmetrical joint deformities consistent
with rheumatoid arthritis. The heart rate was 90 beats/min and irregular. Her blood pressure measured
140/90 mmHg. The JVP was not raised. Both heart sounds were normal and the chest was clear.
Abdominal examination was normal. Inspection of the lower limbs revealed pitting oedema.

Investigations are shown.

Hb 11 g/dl

WCC 5 X 109/l

Platelets 190 X109/l

Sodium 134 mmol/l

Potassium 4.5 mmol/l

Urea 6 mmol/l

Creatinine 70 micromol/l
Bilirubin 11micromol/l

Alkaline phosphatase 100 iu/l

Albumin 26 g/l

Urinalysis Protein +++

What is the management?

A) Stop penicillamine.

B) Start prednisolone.

C) Start ACE inhibitor therapy.

D) Arrange renal biopsy.

E) Arrange IVU.

Answer: A

41. You have received a blood test report with the following results: serum calcium 3 mmol/l, serum
phosphate 0.6 mmol/l (normal range 0.8-1.4 mmol/l); plasma parathyroid hormone (PTH) 5.8 pmol/l
(normal range 0.9-5.4 pmol/l). Which one of the following disorders could be associated with this
abnormal blood test? 

A) Addison's disease 

B) DiGeorge syndrome 

C) Mucocutaneous candidiasis 

D) Medullary-cell carcinoma 

E) Magnesium deficiency

Answer: D

42. Vitiligo is associated with which one of the following disorders? 

A) Nelson's syndrome 
B) Hyperparathyroidism 

C) Amiodarone therapy 

D) Addison's disease 

E) Chronic renal failure

Answer: D

43. A 50-year-old woman is admitted with acute diarrhoea and dehydration. Just prior to her admission,
she had a bout of lower urinary tract infection and is on antibiotics. Stool culture is negative and
sigmoidoscopy reveals pseudomembranous colitis. Which one of the following statements is most
characteristic of this patient's condition? 

A) Bloody diarrhoea, abdominal pain and tenderness is the most common presentation 

B) The detection of Clostridium difficile bacilli in the stool is diagnostic 

C) A severe form of the disease is often associated with gentamicin therapy 

D) It is caused by a Gram-positive anaerobic bacterium 

E) Intravenous vancomycin for 2 weeks is an effective treatment

Answer: D

44. A 45-year-old man is being investigated for persistent dyspepsia and heartburn. Acid secretion
studies show gastric acid hypersecretion. Which one of the following conditions is the most likely cause
of this patient's symptoms? 

A) Pernicious anaemia 

B) Large-bowel resection 

C) Vasoactive intestinal polypeptide- (VIP-) secreting tumour 

D) Systemic mastocytosis 

E) Cushing's syndrome
Answer: D

45. A 19-year-old student presents with acute shortness of breath and haemoptysis. She reported left
loin pain and was found to have haematuria 2 weeks ago. She is under regular review by the renal
physicians for nephrotic syndrome. Ultrasound of the kidney shows left renal vein thrombosis and a
computed tomography pulmonary angiogram confirms acute pulmonary embolism. Which one of the
following is the most likely cause of this patient's increased tendency for developing venous
thrombosis?

A) Factor V Leiden mutation 

B) High levels of anticardiolipin antibodies 

C) Protein S deficiency 

D) Protein C deficiency 

E) Antithrombin III deficiency

Answer: E

46. Which of the following statements is most accurate with regard to Wilson's disease? 

A) The primary defect is believed to be enhanced intestinal absorption of copper 

B) An alternative diagnosis should be considered if chorea occurs in the absence of Kayser-Fleischer


rings

C) Chronic liver disease and autoimmune haemolytic anaemia are recognised features 

D) Raised serum copper levels are evident at birth 

E) A sibling with biochemical evidence of the disease is treated only when he or she becomes
symptomatic

Answer: B
47. Which one of the following conditions is most likely to be associated with pyoderma gangrenosum? 

A) Tuberculosis 

B) Leprosy 

C) Chronic myeloid leukaemia 

D) Sulphonamide therapy 

E) Cushing's syndrome

Answer: C

48. A retired 69-year-old man presents to the Emergency Department with a temperature of 39 °C. The
white cell count (WCC) is 45 x 109/l with 90% neutrophils. Which one of the following findings would be
most useful for differentiating chronic myeloid leukaemia (CML) from a leukaemoid reaction?

A) Philadelphia chromosome 

B) Splenic enlargement 

C) Low leucocyte alkaline phosphatase score 

D) Hypercellular bone marrow 

E) Elevated platelet count

Answer: A

49. The blood gas analysis in a 40-year-old woman who has presented with fatigue shows: pH 7.51, Pao2
11 kPa, Paco2 6 Pa, bicarbonate 32 mmol/l. Which one of the following is the most likely cause of this
patient's clinical presentation? 

A) Spironolactone therapy 

B) Acetazolamide therapy 

C) Conn's syndrome 

D) Addison's disease 
E) Chronic diarrhoea

Answer: C

50. A 30-year-old woman presents to the Emergency Department at 9 pm with severe headache. Two
hours earlier she had felt as if she was hit on the back of the head and she then experienced severe
occipital headache and vomiting. A diagnosis of subarachnoid haemorrhage is suspected. Urgent
computed tomography (CT) scan of the brain is normal. Which one of the following is the most
appropriate next step in the management? 

A) Reassure and discharge after prescribing strong painkillers 

B) Observe in hospital and repeat the CT brain scan the next morning 

C) Perform a lumbar puncture and cerebrospinal fluid (CSF) analysis immediately 

D) Observe in hospital and delay lumbar puncture and CSF analysis until the next morning 

E) Arrange urgent magnetic resonance imaging (MRI) of the brain

Answer: D

51. A 43-year-old woman with rheumatoid arthritis visits for a scheduled follow-up. Plain X-ray of the
hands reveals marginal erosions at the metacarpal heads. Why are these erosions marginal?

A) Random localization

B) Plain X-rays fail to show central erosions

C) Rotation of the film

D) Marginal joint area is devoid of overlying cartilage

E) Presence of sesamoid bones

Answer: D
52. A 33-year-old homosexual man presents with fever, cough and prostration. His sputum is positive for
acid-fast bacilli. Which one of the following antituberculous medications is contraindicated in a patient
with AIDS?

A) Isoniazid

B) Thiacetazone

C) Rifabutin

D) Ethionamide

E) Cycloserine

Answer: B

53. A 15-year-old male patient with ventricular septal defect presents to A&E in a dyspnoeic state. After
careful examination the senior house officer (SHO) suggests the development of Eisenmenger’s
syndrome. You reexamine the patient and find something that points away from the SHO’s diagnosis.
What have you found?

A) Clubbing

B) Central cyanosis

C) Soft P2

D) Single S2

E) Graham–Steell murmur

Answer: C

54. A 65 year old female who has a history of longstanding psoriasis and heavy alcohol intake, presents
with a severe exacerbation of psoriasis. She was admitted and received topical therapy and over the
month of her admission, her gamma-GT concentration had fallen from 400 U/L to 150 U/L (4 - 35). Six
weeks after discharge she was seen in outpatients where her psoriasis remained under control, but she
complained of generalized hair loss. What is the most likely cause for her hair loss?

A ) Alopecia areata
B ) iron deficiency

C ) telogen effluvium

D ) thiamine deficiency

E ) trichotillomania

Answer: C

55. A 17 year-old girl presents with vomiting and her investigations show:

sodium 120 mmol/L (137 - 144)

potassium 3.0 mmol/L (3.5-4.9)

urea 2.2 mmol/L (2.5 - 7.5)

urine sodium 2 mmol/L

urine osmolality 700 mosmol/kg (350 - 1000)

What is the most likely diagnosis?

A ) Addison's disease

B ) bulimia nervosa

C ) diuretic abuse

D ) syndrome of inappropriate antidiuretic hormone secretion

E ) water intoxication

Answer: B

56. A 70-year-old female is diagnosed with anaplastic thyroid cancer. What is the most likely
consequence of this cancer?

A ) Brain metastases
B ) Hypercalcaemia from bony metastases

C ) Liver metastases

D ) Lung metastases

E ) Upper airways obstruction

Answer: E

57. Three months following laparoscopic cholecystectomy, a 50-yearold woman presents with right
upper abdominal quadrant pain, excessive flatulence and intolerance to fatty food. Abdominal
ultrasound and ERCP fail to detect any residual stone or biliary stricture. What is the cause of this
woman’s presentation?

A) Small common bile duct stones

B) Cholangiocarcinoma

C) Peri-ampullary adenocarcinoma

D) Post-cholecystectomy syndrome

E) Choledochal cyst

Answer: D

58. A 23-year-old woman has postpartum haemorrhage following

vaginal delivery of a macrosomic baby. Three hours after starting blood transfusion, she develops fever,
breathlessness and diffuse pulmonary infiltrates. Pulmonary capillary wedge pressure is 8 mmHg. You
start high-flow oxygen. What has developed in this woman?

A) Over-transfusion

B) Major ABO incompatibility

C) Pulmonary aspiration

D) Re-expansion pulmonary oedema


E) Transfusion-related acute lung injury

Answer: E

59. A 49-year-old man is referred to you for further evaluation of right iliac fossa mass. His abdominal
ultrasound shows many small rounded liver masses in addition to the right iliac fossa mass. His 24-hour
urinary 5-HIAA is prominently raised. Which one of the following could be part of this man’s illness?

A) Unilateral rhonchus

B) Episodic facial pallor

C) Giant JVP n-wave

D) Fingers telangiectasia

E) Episodic constipation

Answer: C

60. A 23-year-old recently married woman presents with dysuria and suprapubic discomfort. She is
afebrile with no loin pain. Her past medical history is unremarkable. How would you treat her
empirically?

A) Intravenous ciprofloxacin

B) Oral rifampicin

C) Oral co-amoxiclav

D) Intramuscular gentamicin

E) Oral vancomycin

Answer: C

MULTIPLE CHOICE QUESTIONS-3


1. A 42-year-old woman has a12-year history of ulcerative colitis that has responded well to mesalamine
and occasional corticosteroid enemas. Recent surveillance colonoscopy with biopsies showed low-grade
dysplasia. Which of the following would be the most appropriate next step in the management of this
lady?

A) No intervention, repeat colonoscopy in 6 months

B) No intervention, repeat colonoscopy in 1 year

C) Administer continuous corticosteroid enemas

D) Refer to the surgeon for colectomy

E) Administer a high-dose corticosteroid intravenously

Answer: D

2. A 28-year-old man undergoes an endoscopic transthoracic sympathectomy for palmar hyperhidrosis.
Postoperatively, he has developed ptosis of his eye. This is due to iatrogenic damage to which one of the
following nerves?

A) 8th cervical nerve

B) 1st thoracic nerve

C) Lower cord of brachial plexus

D) 2nd thoracic ganglion

E) 7th cervical nerve

Answer: B

3. A 51-year-old woman undergoes a successful bone marrow transplant from a matched unrelated
donor for refractory Hodgkin’s disease. She is discharged from the hospital on no medications and is
feeling well. At an appointment 6 months posttransplant, she is well with no evidence of malignancy.
Three weeks later, she travels to Florida with her family. She is cautious of the sun but develops sunburn
on her face, despite wearing sunscreen and a protective hat. When she returns from her trip 5 days
later, she presents with persistent erythema of her face. She also states that her wrists and hands have
been sore for the past 2–3 weeks. On examination, her face is mildly tender to touch, and a rash is
present. Her hands are diffusely swollen. She is afebrile, and the remainder of her physical examination
is benign. What is the most likely diagnosis?

A) Graft-versus-host disease

B) Hypersensitivity to sunlight due to the patient’s antirejection regimen

C) New-onset systemic lupus erythematosus

D) Rosacea

E) Staphylococcal skin infection acquired during travel

Answer: A

4. A 35-year-old woman arrives on the floor after an uneventful hysteroscopy to evaluate her long
history of uterine fibroids. About 30 minutes after her arrival, she begins to complain of nausea and has
two episodes of vomiting. The physician administers 0.625 mg of droperidol and 400 mg of
acetaminophen by mouth. On follow-up evaluation, the patient's neck is involuntarily flexed to one side.
She is alert, oriented, and conversant and has an otherwise normal neurologic examination. Which of
the following is the most likely diagnosis?

A) Cerebral vascular accident 

B)Conversion disorder 

C) Dystonic reaction to droperidol

D) Munchausen syndrome 

E) Seizure

Answer: C

5. A 50-year-old woman with diabetes mellitus presents with backache and inability to walk unaided.
This came on suddenly, while she was trying to lift her shopping bag from the car boot. Which of the
following is most suggestive of a lesion of the sciatic nerve?
A) Absent knee tendon jerk

B) Foot drop

C) Inability to flex the hip

D) Decreased sensation on the anterior thigh and medial leg

E) Intervertebral disc prolapse at L2/L3 level

Answer: B

6. A 59-year-old man has a body mass index (BMI) of 42. As might be expected, his fasting blood glucose
level is high, 210 mg/dL, as was his HbA1c level, 9.8%. In addition, he suffers from hypertension and
dyslipidemia. His physician advised him to lose weight both for his general well-being and to help control
his diabetes. Consequently, for the past 4 years, he has desperately tried to lose weight. He worked his
way through an alphabet of popular and fad diets, from the Atkins diet to the Weight Watchers diet. If
he lost a few pounds while on a particular diet, he gained back the pounds lost, plus a few more within a
few months of terminating the diet. Finally, he and his physician decided he should try bariatric surgery,
the Roux-en-Y gastric bypass procedure. A week after leaving the hospital, his fasting blood glucose level
was 100 mg/dL. Three months later, it was 96 mg/dL and his HbA1c was 6.0%. Which of the following
choices is most likely to explain this remarkable improvement is his diabetes?

A) The reduction in level of his circulating glucagonlike peptide-1 (GLP-1)

B) The reduction in level of his circulating peptide YY (PYY)

C) The reduction in his mass of adipose tissue

D) The reduction in the level of his circulating ghrelin

E) The reduction in the level of his circulating leptin

Answer: D

7. A 51-year-old man has recurrent bouts of lightheadedness and mild confusion. Episodes of
hypoglycemia were recently documented that are improved with ingestion of food. The patient has no
headache, blurred vision, or double vision. He has gained approximately 4.5 kg (10 lb) in the past 2
months. Medical and family histories are noncontributory. Physical examination is normal. The following
laboratory data are obtained after an overnight fast: plasma glucose 30 mg/dL (1.67 mmol/L), serum
insulin 30 mlU/L (215.25 pmol/L), and an elevated serum C-peptide level. Screening for sulfonylurea is
negative and CT scan of the abdomen is normal. 

Which of the following diagnostic studies should be done next?

A) Endoscopic retrograde pancreatography.

B) Transabdominal (Endoscopic) ultrasonography.

C) MRI of the abdomen. 

D) Positron emission tomography.

E) Somatostatin receptor scintigraphy.

Answer: B

8. A 23-year-old married woman comes to the office after recent exposure to a person with active
hepatitis A. She has a long history of recurrent sinopulmonary infections and bronchial asthma. In
addition, after her last pregnancy, she received a blood transfusion for severe postpartum hemorrhage.
After receiving an intramuscular dose of immune serum globulin as prophylaxis against hepatitis A, she
develops an anaphylactic reaction. Which of the following is the most likely cause of this patient’s
reaction?

A) Immunoglobulin A (IgA) deficiency with anti-IgA antibodies

B) A hemolytic transfusion reaction

C) Contaminated immune serum globulin

D) A type IV hypersensitivity reaction against a protein in the immune serum globulin

E) A febrile reaction

Answer: A

9. A 47-year-old man recently consulted a physician about developing weakness, particularly in his
right hand. Upon providing a history, the man explained that he does house repair and has been working
on a neighborhood rehabilitation project for the past several months. In doing this, he sandblasts and
sands and scrapes by hand to remove the old paint. These
homes were first constructed in the 1920s and since have been covered with several layers of paint.
He also revealed that he habitually ate his lunch at the work site, which he described as being dusted
with old paint particles. In addition to the weakness in his arm, he admitted to sporadic stomachaches,
constipation, and said his wife had complained that he is always irritable. He also states that, until
recently, he had been in good health. Upon examination, he was found to be 6 feet (19.7 m) tall and to
weigh 170 lb (77.1 kg). His heart, lungs, and abdomen were normal, as were most analytical values, but
he did show signs of right wristdrop consistent with radial nerve palsy and his complete blood count
(CBC) showed a microcytic anemia; his serum iron levels were found to be normal. Which of the
following diagnostic tests would provide the most useful information regarding the appropriate
treatment?

A) Nerve conduction velocity (NCV) study of the right arm

B) Radiography of the right arm and wrist

C) Magnetic resonance imaging (MRI) scans of the right arm and wrist

D) Urine screen for heavy metals (lead, mercury, arsenic)

E) Screening for diabetes mellitus.

Answer: D

10. Which of the following clinical findings is least likely in a patient with infarction in the distribution of
the left anterior choroidal artery? 

A) Right hemiparesis

B) Right hemi-sensory loss

C) Right hemianopia

D) aphasia.

Answer: D

11. A 76-year-old man presenting with dysphagia was found to have inoperable esophageal
adenocarcinoma. He has an endoscopically placed esophageal stent for palliation of his symptoms, but
unfortunately found it very painful, and it was removed a few days later. There is no perforation. He asks
whether there are any other treatment options to help with his symptoms. Which of the following
modalities is an appropriate first-line treatment option to discuss?

A) Band ligation

B) Botulinum toxin injection

C) Brachytherapy

D) Local ethanol injection

E) Photodynamic therapy.

Answer: C

12. An AIDS patient develops symptoms suggestive of a severe, persistent pneumonia with cough, fever,
chills, chest pain, weakness, and weight loss. The patient does not respond to penicillin therapy, but
goes on to develop very severe headaches. The presence of focal neurologic abnormalities leads the
clinician to order a CT scan of the head. This demonstrates several metastatic brain abscesses. Biopsy of
one of these lesions demonstrates beaded, branching, filamentous gram-positive bacteria that are
weakly acid fast. Which of the following is the most likely causative organism? 

A) Actinomyces

B) Aspergillus

C) Burkholderia

D) Francisella

E) Nocardia.

Answer: E

13. You are a hospitalist called to admit a 70-year-old man to the ICU. His wife states that he was sitting
at the table eating breakfast with her when he dropped his fork and had difficulty speaking. Within a
couple of minutes he was unable to move his right side. She called the paramedics, who brought him to
the hospital. Now in the ICU, his vital signs are as follows: BP 200/98 mm Hg, HR 100, RR 10, O2
saturation 94% on room air. He is afebrile. On physical examination he is lethargic and unable to speak.
His pupils are equal and round but sluggish. He has flaccid paralysis of the right arm and leg with a
Babinski sign present on the right. His heart is irregularly irregular, and an ECG confirms atrial fibrillation.
A CT of the head shows a large bleed in the left frontoparietal area with mass effect and midline shift.
You decide to intubate the patient to protect his airway. What is the next most appropriate step in the
treatment of this patient while you are awaiting your urgent neurosurgical consult?

A) Hyperventilate the patient to a goal pCO2 of 20 mm Hg

B) Give a bolus of IV mannitol

C) Give a bolus of IV dexamethasone

D) Give sublingual nifedipine to decrease the BP

E) Anticoagulate with IV heparin because of the atrial fibrillation.

Answer: B

14. A 73-year-old woman is brought in by paramedics after fainting in the mall and hitting her face. She
does not remember any preceding symptoms, and she did not lose control of her bowel or bladder.
Witnesses at the scene say that she was down for less than a minute, then woke up and was fairly alert.
She was bleeding from a laceration on her chin and paramedics were called. When she arrived at the
hospital, her initial laboratory

values were normal and an EEG did not show epileptiform activity. She is placed on a cardiac monitor.
The following day, she becomes lightheaded and loses consciousness while lying in bed, and her monitor
shows tachycardia with the QRS complexes being uniformly longer than 120 ms.

What is the most common cause of this rhythm disturbance?

A) Uncontrolled hypertension

B) Distention of the pulmonary veins

C) Accessory pathway

D) Ischemic heart disease.

Answer: D
15. A 47-year-old woman presents at night to the Emergency Department with chest pain. She states
that the pain started that evening and has progressively been getting worse. She is concerned that she is
having a heart attack. The pain is described as a burning sensation associated with a sour taste in her
mouth, and it started shortly after she ate dinner; it has occurred on previous occasions, but never as
bad as it is now.

Previously, she used calcium carbonate tablets that were effective for the pain. She has no history of
heart disease or other medical problems, and she takes no regular medications. She does not smoke
cigarettes or use cocaine. Her vitals are normal, and her physical examination is unremarkable. Initial
laboratory tests and an ECG are normal. Which of the following is the most likely cause of this patient’s
chest pain?

A) Unstable angina

B) Myocarditis

C) Pulmonary embolism

D) Gastroesophageal reflux disease

E) Costochondritis.

Answer: D

16. A 28-year-old patient with end-stage renal disease (ESRD) on continuous ambulatory peritoneal
dialysis (CAPD) for two months presents with fever, abdominal pain and cloudy dialysis fluid. There is no
diarrhea or vomiting and the pain has been present for about 12 hours. The patient has ESRD secondary
to chronic glomerulonephritis, there is no history of diabetes, urinary infections or antibiotic use.
Examination reveals a temperature of 38.9 C (102 F), and blood pressure of 110/70 mm Hg. The throat is
clear, as are the lungs. Cardiac examination reveals a grade 2/6 systolic murmur. Abdominal
examination reveals decreased bowel sounds with diffuse tenderness. There is mild rebound. There is no
edema or skin rash. A complete blood count shows a leukocyte count of 14,200/mm3, hemoglobin is
12.5 g/dL. Peritoneal fluid is cloudy with 1,000 white blood cells, 85% of which are polymorphonuclear
leukocytes. Gram's stain of fluid is negative. Cultures of blood and peritoneal dialysis fluid are taken.
Which of the following is the most appropriate initial step in management? 

A) Fluconazole

B) Immediate removal of dialysis catheter.

C) Intravenous vancomycin

D) Intravenous gentamicin 
E) Oral ciprofloxacin

Answer: C

17.A 59-year-old lady is admitted with a 30 minute history of heavy central chest pain associated with
nausea and sweating. Her ECG shows ST elevation in leads V1, V2, V3 and V4.

Which of the following coronary arteries is most likely to be occluded?

A) Circumflex artery

B) Left anterior descending artery

C) Obtuse marginal artery

D) Posterior descending artery

E) Right coronary artery

Answer: B

18. A family physician cares for a family consisting of a 45-year old husband, 43-year-old wife and a 12-
year-old daughter. The family reports that recently the 77-year-old maternal grandmother who lived
with them died after a prolonged respiratory infection. Autopsy subsequently confirms that she had
active pulmonary tuberculosis at the time of death. The organism tested sensitive to all anti-tuberculosis
drugs. In responding to the grandmother's illness, which of the following is the most appropriate step in
management? 

A) Obtain leukocyte counts on all family members

B)Obtain sputum cultures for acid fast bacilli 

C)Obtain chest computerized tomograms on all members 

D)Place protein purified derivative (PPD) test on all members

E)Schedule bronchoscopy lavage for the adults.

Answer: D
19. Which of the following extraintestinal manifestations is associated with Crohn disease but not
ulcerative colitis? 

A) Ankylosing spondylitis 

B) Erythema nodosum 

C) Nephrolithiasis 

D) Thromboembolic disease 

E) Uveitis

Answer: C

20. A 30 year old woman presents with unprovoked left popliteal deep vein thrombosis. Her family
history is negative for venous thromboembolism. She has a history of SLE and takes prednisone 10mg
daily, hydroxychloroquine and supplements of calcium and vitamin D. She has one child but has had two
miscarriages. An aPTT test performed before starting anticoagulation in this patient is prolonged, but
her PT is normal.

Which statement is most likely to be correct?

A) Because her APTT is increased already, Heparin should be withheld and warfarin treatment initiated
at INR 2.5 to 3.5, perhaps forever 

B) The patient should be encouraged to use oral contraceptive to prevent pregnancy during the time she
is anticoagulated

C) She will need Heparin and then anticoagulation with Warfarin at INR 2 to 3, perhaps indefinitely

D) Start anti-coagulation with Heparin and then Warfarin at INR 2.5 to 3.5 for 3 to 6 months

E) She will need lifelong new oral anticoagulants.

Answer: C

21. A 65-year-old man has been stable on the general medical ward following an admission with acute
coronary syndrome several days previously. His drug history consists of aspirin, enalapril and glyceryl
trinitrate (GTN) spray. He has developed dyspnea over the last few hours. On examination he has a
raised jugular venous pressure (JVP) and crackles to his mid zones. His electrocardiogram (ECG) shows a
rate of 140 beats per minute in atrial fibrillation. Which of the following is the most appropriate
management?

A) Intravenous amiodarone

B) Intravenous digoxin

C) Intravenous flecainide

D) Observe and screen for MI 

E) Synchronized DC cardioversion.

Answer: E

22. A 48-year-old man is admitted through the ER with a possible left lower extremity cellulitis. The
patient says that he has been having fever, swelling, and redness of his left lower extremities that comes
and goes spontaneously over the past couple of months. He also tells you about an episode in which he
lost vision in his left eye for several minutes a couple of weeks ago, but the vision returned without
incident. He had a urologic evaluation for penile trauma 3 months ago. On physical examination, his BP
is 125/80, HR 70, RR 14, and he is currently afebrile. His examination is significant for a 3/6 systolic
murmur heard at the left lower sternal border without radiation while lying supine. Blood cultures
return positive results for Enterococcus species, and an echocardiogram reveals large mitral vegetation.
You review treatment for enterococcal endocarditis. Which of the following antibiotics always misses
enterococcal infections?

A) Ampicillin-sulbactam

B) Nitrofurantoin

C) Cefipime

D) Vancomycin

E) Linezolid.

Answer: C
23. A 70-year-old woman with a history of type2 DM and CAD with a known left bundle branch block on
ECG is admitted to the ICU with sepsis from a urinary source. She is fluid resuscitated, and empiric
broad-spectrum antibiotics are begun after the appropriate cultures are obtained. Despite what appears
to be adequate resuscitative efforts with volume replacement, the patient has had minimal urine output
over the past couple of hours. You decide to place a pulmonary artery catheter to help to determine the
patient’s hemodynamic situation. Which of the following complications of pulmonary artery catheter
placement is the patient at increased risk for because of her past medical history?

A) Pulmonary artery perforation

B) Pulmonary infarction

C) Complete heart block

D) Pneumothorax

E) Ventricular tachycardia.

Answer: C

24. The first four tests that should be ordered in the initial evaluation of patients with a suspected
coagulopathy are all of the following except

A) Platelets count

B) Bleeding time

C) Platelet aggregation studies

D) PT

E) aPTT

Answer: C

25. Which of the following is not true about tetanus?

A) The toxin affects inhibitory GABA and glycine receptors, leading to unopposed contraction and spasm
of skeletal muscle.

B) It is characterized by acute onset of skeletal muscle rigidity and convulsive spasm. 


C) Initial symptoms involve lockjaw and risus sardonicus.

D) Fractures, dislocations, and rhabdomyolysis may occur due to forceful sustained muscle 

E) Tetanus disease usually leads to long-lasting immunity.

Answer: E

26. A 55-year-old woman, who has never smoked, presents to you on the ward with a history of weight
loss, decreased appetite and finger clubbing. You are told that her chest x-ray revealed opacity in the
hilar region of the right lung suggesting a bronchogenic carcinoma. She is currently awaiting a CT-chest
with bronchoscopy to follow. From the list below, select the most likely diagnosis: 

A) Squamous cell carcinoma of the lung 

B) Adenocarcinoma of the lung 

C) Small cell carcinoma of the lung 

D) Large cell carcinoma of lung 

E) Carcinoid tumour of the lung.

Answer: B

27. A 68-year-old woman recently diagnosed with multiple myeloma presents to her GP with
progressively increasing breathlessness, exercise intolerance and ankle swelling. On examination, there
is bilateral pitting leg edema to her thighs, ascites and raised JVP. The apical impulse is impalpable. An
ECG shows diffusely diminished voltage. Chest X-ray is normal and the echocardiogram shows small
thick ventricles and dilated atria with a thickened interatrial septum. The ventricular myocardium has a
granular sparkling texture on echo, and minimal fluid in the pericardial space is noted. What is the most
likely diagnosis? 

A) Chronic pericardial effusion with tamponade 

B) Chronic pericardial effusion without tamponade 

C) Constrictive pericarditis 

D) Restrictive cardiomyopathy 
E) Congestive heart failure.

Answer: D

28.A 65-year-old male patient is admitted with renal failure and is diagnosed with acute tubular
necrosis. Which of the following is least likely to be the cause of acute tubular necrosis?

A) Rhabdomyolysis

B) Paracetamol poisoning

C) Hypovolemia

D) Hypertension

E) Corticosteroid therapy

Answer: E

29. A 65-year-old woman with long-standing diabetes mellitus visits your clinic for follow-up 2 weeks
after the initial visit. Her fasting home glucose monitoring shows elevated blood sugar levels ranging
between 200-250 mg/dl. Two weeks ago, her HbA1c was 7.2% and the fasting plasma glucose was 212
mg/dl. You recommend the following to evaluate the discrepancy between the fasting values and the
HbA1c:

A) Order a CBC 

B) Prescribe a new glucometer

C) Order a fructosamine

D) Repeat the HbA1c

E) Repeat the fasting plasma glucose.

Answer: A
30. A 43-year-old man presents to the clinic with complaints of fever, night sweats, anorexia, cough, and
chest pain. The chest x-ray reveals infiltrates in both the lower and upper lobes, with possible cavitations
in the apices. A presumptive diagnosis of tuberculosis is made on the basis of finding acid-fast bacilli
(AFB) on microscopic examination of sputum. The patient is started initially on isoniazid, rifampin,
pyrazinamide, and ethambutol. 

What is the best way to monitor this patient?

A) Sputum acid-fast stains every month for 6 months

B) Sputum cultures every month until cultures become negative

C) Serial chest x-rays

D) Blood testing for drug toxicity

E) Observe for clinical deterioration.

Answer: B

31. A 49-year-old man with untreated hepatitis C infection develops persistent proteinuria. Which of the
following diagnoses is not typically associated with hepatitis C?

A) Membranous nephropathy

B) Focal segmental glomerulosclerosis

C) Membranoproliferative glomerulonephritis

D) Cryoglobulinemia

E) Polyarteritis nodosa

Answer: B

32. A 59-year-old woman is admitted with central abdominal pain. Serum amylase is 1800 IU/L. Her
initial Glasgow Coma Scale score is 4. You are asked to review her the next day as the nurses have
noticed that her urine output has been just 15 mL in the past 3 hours. The rest of her observations are
as follows:

• Blood pressure = 105/45 mmHg


• Pulse = 113 beats/min

• Respiratory rate = 28 breaths/min

• Saturation 93% on 8 L of oxygen

On auscultation of her chest you hear widespread crepitations. What is the most appropriate next
course of action?

A) Fluid restriction

B) Colloid bolus

C) Furosemide

D) Transthoracic echocardiogram

E) Central line insertion

Answer: E

33. Features of type 1 renal tubular acidosis include all of the following EXCEPT:

A) Normal AG acidosis

B) Hypokalemia

C) Ca phosphate stone formation

D) Urine pH decrease below 4, following oral ammonium chloride loading test

E) Raised serum PTH hormone

Answer: D

35. All of the following statements regarding non-alcoholic steatohepatitis (NASH) are correct EXCEPT:

A) Liver biopsy is the only means of accurately diagnosing NASH.

B) Weight loss has been shown to result in improvement of elevated liver enzymes.

C) Patients with NASH can develop liver cirrhosis.


D) Histologically it is characterized by severe portal or peri-portal inflammation.

E) Laboratory studies cannot identify patients at risk of progression.

Answer: D

36. A 30-year-old male patient complains of fever and sore throat for several days. T he patient presents
to you today with additional complaints of hoarseness, difficulty breathing, and drooling. On
examination, the patient is febrile and has inspiratory stridor. Which of the following is the best course
of action?

A) Begin outpatient treatment with ampicillin.

B) Culture throat for β-hemolytic streptococci.

C) Admit to intensive care unit and obtain otolaryngology consultation.

D) Schedule for chest x-ray.

E) Obtain Epstein-Barr serology.

Answer: C

37. Regarding management of an episode of hypoglycaemia in a 65-year-old who is on sulfonylurea


therapy, which ONE of the following is TRUE?

A) Initial oral or intravenous glucose therapy can be omitted because it is likely to fail

B) Octreotide should be considered for recurrent or persistent hypoglycaemia

C) Hypoglycaemia in a stable diabetic on a regular sulfonylurea dose is not usually due to a precipitating
event

D) There is a more sustained response to intravenous glucose therapy in sulfonylurea-induced


hypoglycaemia than in insulin-induced hypoglycaemia

Answer: B
38. A 59-year-old man presents to the ED with left-sided chest pain and shortness of breath that began 2
hours prior to arrival. He states the pain is pressure-like and radiates down his left arm. He is
diaphoretic. His BP is 160/80 mm Hg, HR 86 beats per minute, and RR 15 breaths per minute.
ECG reveals 2-mm ST-segment elevation in leads I, aVL, V3 to V6. Which of the following is an absolute
contraindication to receiving thrombolytic therapy?

A) Systolic BP greater than 180 mm Hg

B) Patient on Coumadin and aspirin

C) Total hip replacement 3 months ago

D) Peptic ulcer disease

E) Previous hemorrhagic stroke

Answer: E

39. The best pharmacologic treatment for hypotension related to the spinal anesthetic is

A) atropine

B) epinephrine

C) phenylephrine

D) calcium

E) labetalol

Answer: C

40. An ischemic stroke involving the right side of the pons could lead to which of the following patterns
of weakness?

A) Left facial weakness and right body weakness

B) Right facial weakness and left body weakness

C) Right facial weakness and right body weakness


D) Left arm weakness and right leg weakness

E) Right arm weakness and left leg weakness

Answer: B

41. A 49-year-old man who smokes two packs of cigarettes a day presents with a lung mass on x-ray and
recent weight gain. Laboratory examination shows hyponatremia with hyperosmolar urine. The patient
probably has

A) Renal failure

B) Pituitary failure

C) Conn’s syndrome

D) Cardiac failure

E) Inappropriate ADH

Answer: E

42. A 75-year-old woman is referred for assessment of her mental state. Her carer states that she
noticed general decline in her physical and mental state. She is more forgetful and her concentration is
poor. Sometimes her speech is incoherent. She trips frequently and had many falls. On rare occasions
she has imagined seeing people sitting in her room. Each of the following features would help to
distinguish between organic and functional disorders, except

A) Nystagmus 

B) Visual hallucinations 

C) Poor performance of the serial sevens test 

D) Dysphasia 

E) Hemianopia

Answer: C
43. A 40-year-old man with occasional dysphagia and who otherwise feels well undergoes esophageal
motility studies that show an LES amplitude of approximately 60 mmHg. The esophagus relaxes
completely when he swallows. The most likely diagnosis is

A) GERD (gastroesophageal reflux disease)

B) Achalasia

C) Hypertensive LES

D) Barrett’s esophagus

E) Esophageal spasm

Answer: C

44. A 60-year-old accountant complains of recurrent attacks of exquisite pain and swelling in the left big
toe. Each of the following conditions is likely to be associated with this disorder, except 

A) Chronic alcoholism

B) Obesity

C) Rheumatoid arthritis 

D) Diabetes mellitus 

E) Diuretic therapy

Answer: C

45. A 60-year-old man was referred for further assessment of a cardiac murmur. On examination it was
noted that he has a weak and slow-rising carotid pulse. The most likely underlying cardiac abnormality is

A) Aortic regurgitation 

B) Alcoholic cardiomyopathy

C) Dissecting aneurysm
D) Hypertrophic obstructive cardiomyopathy (HOCM)

E) Aortic stenosis

Answer: E

46. A 50-year-old woman has a history of gastrinoma and pituitary tumour. She reports increasing
lethargy, drowsiness and constipation. The laboratory studies include raised calcium (2.9 mmol/I), low
phosphate and raised PTH levels. The most likely diagnosis is? 

A) Carcinoma of the bronchus 

B) Pseudohypoparathyroidism 

C) Acromegaly 

D) Sarcoidosis 

E) Multiple endocrine neoplasia syndrome (MEN I)

Answer: E

47. A 60-year-old teacher is being investigated for increasing shortness of breath and diffuse fibrotic
changes found on plain chest X-rays. He was taking diuretics and anti-arrhythmic treatment. The 12-lead
ECG shows a prolonged QT interval. Each of the following clinical situations could be responsible for this
ECG abnormality, except

A) Heart failure 

B) Hypokalaemia 

C) Furosemide therapy 

D) Hypercalcaemia 

E) Amiodarone therapy

Answer: D
48. A 30-year-old mechanic presented with central chest pain, worse on lying flat. He claims that he has
had a flu-like illness for a week. Which one of the following ECG changes is most characteristic of this
disorder? 

A) PR prolongation 

B) ST depression 

C) Peaked, tall T wave 

D) Prominent U wave 

E) PR-segment depression

Answer: E

49. A 16-year-old male treated for acute lymphocytic leukemia develops severe lumbar and abdominal
pain. His serum amylase is markedly elevated. Which of the following agents most likely caused these
findings?

A) 6-MP

B) Asparaginase

C) Doxorubicin

D) Methotrexate

E) Vincristine

Answer: B

50. A 58-year-old man was admitted with a three-weeks history of shortness of breath. The chest X-ray
demonstrates a right pleural effusion. Pleural fluid analysis shows 6600 mm3 WBC,40% eosinophils. The
condition least likely to be responsible this clinical presentation is 

A) Pneumothorax 

B) Haemothorax 
C) Pulmonary infarction 

D) Benign asbestos pleural effusion 

E) Tuberculous pleural effusion

Answer: E

51. A 33-year-old woman presents for evaluation of chronic hepatitis C virus (HCV) infection discovered
during a life insurance physical examination. Her risk factor for HCV infection was a blood transfusion
after a motor vehicle accident at the age of nine years. Her husband of seven years tests negative for
anti-HCV antibody. The couple wishes to conceive a child. Which of the following recommendations is
most appropriate?

A) Avoid breast-feeding the baby.

B) Plan delivery of the baby by cesarean section.

C) Administer HCV immunoglobulin to the infant at the time of delivery.

D) Test the infant for HCV after the age of 18 months.

E) Use latex condoms.

Answer: D

52. A general practitioner is in a dilemma over a case he has referred to you. He says that the referred
31-year-old man has no myocardial infarction or ischaemic heart disease, but his 12-lead-ECG shows
many Q waves. Which one of the following is not responsible for this man’s ECG Q wave?

A) Hyperkalaemia

B) Hypertrophic cardiomyopathy

C) Wolff–Parkinson–White syndrome

D) Limb lead reversal

E) True posterior wall myocardial infarction


Answer: E

53. A 55-year-old man presents with gross haematuria. He has recurrent urinary bladder stones. Rapid
bedside ultrasound examination fails to demonstrate any stone, but there is a fungating bladder mass.
What type of bladder cancer is likely?

A) Transitional cell cancer

B) Clear cell cancer

C) Squamous cell cancer

D) Leiomyosarcoma

E) Bladder lymphoma

Answer: C

54. A 45-year-old woman is found to have gallstones on abdominal ultrasonography that forms part of a
pre-medical insurance examination. She is reasonably well and healthy and denies abdominal
symptoms. She neither smokes nor drinks alcohol and there is no family history of note. Which of the
following statements is true regarding this woman’s gallbladder disease?

A) Emergency open cholecystectomy is required

B) Prophylactic laparoscopic gallbladder removal should be considered

C) Advise against cholecystectomy

D) Evaluate for surgery fitness

E) Refer to ERCP

Answer: C

55. A 22-year-old man develops a severe attack of acute intermittent porphyria, two days after starting a
medication prescribed by his GP. Which one of the following medications has not been prescribed by his
GP?
A) Atenolol

B) Tolbutamide

C) Theophylline

D) Valproic acid

E) Dapsone

Answer: A

56. A 48-year-old man with acute leukaemia develops fever, chills, headache, flushing, tachycardia and
chest tightness one hour after starting blood transfusion because of anaemia. He reports ‘similar things’
whenever he receives blood transfusion. What is the best action now?

A) Continue the transfusion

B) Call the resuscitation team

C) Stop the transfusion

D) Give paracetamol

E) Epinephrine (adrenaline) injection

Answer: C

57. A 59-year-old man with long-standing Type 2 diabetes presents for his annual check-up. Fundoscopy
reveals scattered dot haemorrhages and a few hard exudates in the macular area. He states that his
vision is not that good but he is coping with it and that his eye glasses are not helpful. Which one of the
following is true?

A) He has proliferative retinopathy

B) He has pre-proliferative retinopathy

C) He should be referred to the ophthalmologist

D) He should be referred for pars plana vitrectomy


E) Reassure and ask him to return next year

Answer: C

58. A 50-year-old man with septicaemia develops severe shortness of breath, cough and irritability.
Which one of the following is inconsistent with the diagnosis of adult respiratory distress syndrome?

A) PaO2/FiO2 150 mmHg

B) Reduced lung compliance

C) Bilateral interstitial infiltrate

D) Bilateral alveolar infiltrate

E) Pulmonary artery capillary tracing of 25 mmHg

Answer: E

59. A 60 year old male is brought to casualty in the early hours of the morning after being found
unconscious in the street. On examination, he was drowsy but localised to painful stimuli. There was no
evidence of head injury or meningism. Investigations revealed:

sodium 134 mmol/L (137-144)

potassium 4.0 mmol/L (3.5-4.9)

urea 4.0 mmol/L (2.5-7.5)

creatinine 80 micromol/L(60-110)

glucose 4.5 mmol/L (3.0-6.0)

chloride 100 mmol/L (95 - 107)

bicarbonate 25 mmol/L (20 - 28)

plasma osmolality 385 mosmol/Kg (278 - 305)

What is the most likely explanation for his presentation

A ) diazepam poisoning
B ) ethanol poisoning

C ) methanol poisoning

D ) phenobarbitone poisoning

E ) Phenytoin poisoning

Answer: C

60. A 46-year-old male was seen for an insurance medical examination. He was entirely asymptomatic,
but his serum urate concentration was noted to be 0.5 mmol/L (0.23 - 0.46). What is the most
appropriate management for this patient?

A ) Allopurinol

B ) Colchicine

C ) Ibuprofen

D ) Lifestyle intervention

E ) Sulphinpyrazone

Answer: D

MULTIPLE CHOICE QUESTIONS-4

MULTIPLE CHOICE QUESTIONS-4

1. A 92-year-old man is referred from his nursing home for evaluation of lethargy. Examination is
unrevealing, but laboratory results are significant for a serum sodium level of 118 meq/L (normal, 135
148). Serum osmolality is 260, urine osmolality is 450, and urine sodium is 80. Which of the following is
the most likely cause of this patient's lethargy?

A) hyperglycemia
B) hyperlipidemia

C) hyperproteinemia

D) SIADH

E) diabetes insipidus

Answer: D

2. Which one of the following is a recognized feature of polymyalgia rheumatica?

A) Weakness of distal muscle groups

B) Elevated serum creatine phosphokinase activity

C) An association with bronchial carcinoma

D) Weight loss

E) A peak incidence in the fourth decade of life

Answer: D

3. A 17-year-old girl notes an enlarging lump in her neck. On examination, her thyroid gland is twice the
normal size, firm to rubbery, multilobular, nontender, and freely mobile. There is no adenopathy. Family
history is positive for both hypo- and hyperthyroidism. Her serum triiodothyronine (T3) and thyroxine
(T4) levels are low normal, and serum thyroid-stimulating hormone (TSH) is high normal. Technetium
scan shows nonuniform uptake. Serum and antithyroglobulin titer is strongly

positive. What will thyroid biopsy of this patient most likely disclose?

A) giant cell granulomas and necrosis

B) polymorphonuclear cells and bacteria

C) diffuse fibrous replacement

D) lymphocytic infiltration

E) parafollicular cells
Answer: D

4. A 60-year-old man presents with a nonproductive cough for a week and generalized malaise. He also
has noted some abdominal pain associated with diarrhea for the past few days. His temperature is
101.5°F and c linical examination is unremarkable. ACXR shows a left lower lobe infiltrate. His urinalysis
shows 50 RBCs, and his BUN (30) and creatinine (1.6) are both mildly elevated. In light of the
extrapulmonary symptoms and signs, which of the following is the most likely cause of his pneumonia?

A) Pseudomonas aeruginosa

B) S. aureus

C) H. influenzae

D) S. pneumoniae

E) Legionella

Aswer: E

5. A 78-year-old woman comes to your primary care office practice with her son who is concerned about
changes in her mood. She is less interested in going out to dinner and does not want to visit family or
friends. Her language skills seem to have deteriorated over the last few years and her memory is not as
sharp. Her gait and motor strength are normal. Which of the following is the most likely diagnosis?

A) Parkinson's disease

B) anxiety disorder

C) meningioma

D) Alzheimer's disease

E) dysthymia

Answer: D
6. A 30-year-old woman who has been human immunodeficiency virus (HIV) positive for 4 years was
recently diagnosed with AIDS. Which of the following immunologic abnormalities would be expected?

A) increased numbers of CD4+ (helper) T cells

B) decreased number of CD8+ (suppressor) T cells

C) cutaneous anergy to usual skin test antigens

D) normal B-cell function

E) increased natural killer cell function

Answer: C

7. A 60-year-old previously healthy man presents with massive rectal bleeding. Which of the following is
the most likely diagnosis?

A) diverticulosis of the colon

B) ulcerative colitis

C) external hemorrhoid

D) ischemic colitis

E) carcinoma of the colon

Answer: A

8. A 59-year-old woman who lives independently and had been healthy, presents to the emergency
department with cough and fever. She related she was well until 2 days before when she noted onset of
fever, chills, and cough productive of yellow sputum. On examination, you note a tired appearing
woman with BP of 160/90, pulse of 105, and respiratory rate of 32. You start her on ceftriaxone and
azithromycin and admit her to the hospital. Which of the following factors is a poor prognostic sign in
community acquired pneumonia?

A) age less than 60

B) systolic BP = 160 mmHg


C) leukocytosis = 15,000

D) respiratory rate = 32

E) mycoplasma pneumonia infected

Answer: D

9. You see a 19-year-old Caucasian man in your clinic who presents with a history of transient jaundice.
On direct questioning, you ascertain that the jaundice is noticeable after periods of increased physical
activity and subsides after a few days. The patient has no other symptoms and physical examination is
unremarkable. Full blood count is normal (with a normal reticulocyte count) and liver function tests
reveal a bilirubin of 37 μmol/L. The most appropriate management is:

A) Reassure and discharge

B) Start on a course of oral steroids

C) Request abdominal ultrasound

D) Request MRCP

E) Refer to Haematology

Answer: A

10. You see a 56-year-old man who was admitted for an elective upper GI endoscopy due to
longstanding GORD which has failed to improve on antacids and PPIs. Your registrar suspects that this
patient may have Barrett’s oesophagus and asks you to define what this is. The most appropriate
description of Barrett’s oesophagus is:

A) Metaplasia of the squamous epithelium of the lower third of the oesophagus to columnar epithelium

B) Metaplasia of the columnar epithelium of the upper third of the oesophagus to squamous epithelium

C) Metaplasia of the columnar epithelium of the lower third of the oesophagus to squamous epithelium

D) Metaplasia of the squamous epithelium of the upper third of the oesophagus to columnar epithelium

E) Metaplasia of the squamous epithelium of the middle third of the oesophagus to columnar
epithelium
Answer: A

11. You see a 48-year-old lorry driver, who presents to you with a three-month history of heartburn
after meals which has not been settling with antacids and PPIs. You suspect that the patient has a hiatus
hernia. The most appropriate investigation for diagnosing a hiatus hernia is:

A) Computer tomography (CT) scan

B) Chest x-ray

C) Upper GI endoscopy

D) Barium meal

E) Ultrasound

Answer: D

12. You see a 47-year-old man in clinic with a three-month history of epigastric dull abdominal pain. He
states that the pain is worse in the mornings and is relieved after meals. On direct questioning, there is
no history of weight loss and the patient’s bowel habits are normal. On examination, his abdomen is soft
and

experiences moderate discomfort on palpation of the epigastric region. The most likely diagnosis is:

A) Gastric ulcer

B) Gastro-oesophageal reflux disease (GORD)

C) Duodenal ulcer

D) Gastric carcinoma

E) Gastritis

Answer: C
13. A 60-year-old farmer presented with non specific symptoms of chronic cough and progressive
dyspnea for the last six years. The patient had interstitial pneumonitis. Chest x-ray revealed
reticulonodular infiltrate associated with honey-combing of the lung. Pulmonary function studies
showed a restrictive pattern with loss of lung volumes, impaired diffusion capacity, decreased
compliance and exercise induced hypoxia. This farmer’s symptoms are most likely secondary to the
following environmental hazard: 

A) The farmers cheese processing plant

B) The presence of spores of bacillus anthracis in the farmer’s farm

C) The harvesting of crops in rainy weather

D) The presence of asbestos in the water tanks.

E) The presence of antigenic detergents

Answer: C

14. Which of the following antiarrhythmic agents may promote AF?

A) Adenosine

B) Quinidine

C) Propafenone

D) Amiodarone

E) Atenolol

Answer: A

15. Torsades de pointes is characterized by all of the following except:

A) Results from triggered activity (early afterdepolarizations) that occurs during

phase 2 or 3 of the cardiac action potential

B) Prolonged QT interval
C) Exacerbation by bradycardia with short-long coupling intervals

D) Polymorphic VT

E) Often provoked during amiodarone administration

Answer: E

16. A 68-year-old patient is seen for a general examination. Current

recommendations for immunizations include

A) Tetanus booster every 5 years

B) Influenza vaccination yearly

C) Pneumococcal vaccination yearly

D) Hepatitis booster every 5 years

E) Meningococcal vaccination

Answer: B

17. Which of the following statements about familial periodic paralysis is true?

A) It is an autosomal-recessive transmitted disorder.

B) It involves disturbances of potassium regulation.

C) It is associated with permanent muscle weakness.

D) It is aggravated by administration of acetazolamide.

E) It most commonly affects the elderly

Answer: B
18. A 52-year-old man complains of severe right knee pain. He recalls no trauma, but attended a wine
tasting party last weekend. He denies past medical history and is afebrile with normal vital signs.
Physical examination reveals a swollen, red, painful right knee. Radiograph shows an effusion, but no
bony erosions. You perform arthrocentesis and obtain 27 mL of cloudy straw-colored fluid, which you
send to the laboratory for studies. The cell count is 50,000 WBCs/mm3 with 85% PMNs, glucose 120
mg/dL (serum 130), and protein 3.5 g/dL. The Gram stain shows numerous WBCs, but is negative for
organisms. Evaluation for crystals shows numerous needle-shaped crystals with negative birefringence.
A good treatment plan would be:

A) Oral colchicine 0.5 mg every hour until symptoms abate, GI toxicity develops or the maximum dose of
6 mg in 24 hours is reached. 

B) Subcutaneous colchicine 1 mg plus oral probenecid 250 mg bid for 1 week.

C) Sublingual colchicine 2 mg followed by oral steroids tapered over 5 days.

D) Intra-articular triamcinolone injection.

E) Oral allopurinol 500 mg tid for 1 week

Answer: A

19. A 38-year-old woman is brought to the emergency department by her spouse because of decreased
mental status. She had knee surgery 2 days ago and was prescribed oral oxycodone for pain. Her spouse
notes that she finished the entire 7-day supply during that day. He denies any seizure activity.They have
no other drugs or medications in the house. She is afebrile with blood pressure of 130/75 mmHg, heart
rate of 70 bpm, respiratory rate of 4 breaths/min, and SaO2 of 85% on room air. She barely responds to
painful stimuli but moves all four extremities equally. Which of the following medications is most likely
to improve her mental status?

A) Albuterol

B) Alvimopan

C) Flumazenil

D) N-Acetylcysteine

E) Naloxone

Answer: E
20. A 45-year-man with longstanding type 1 diabetes mellitus complains of pain in his feet and ankles
that has been present for over a year. All of the following are consistent with neuropathic pain due to
diabetes EXCEPT:

A) Burning pain

B) Electric shock quality

C) Exacerbated by light touch

D) Pain referred to scrotum

E) Tingling

Answer: D

21. Which of the following sets of drug–drug interaction and mechanism is accurately described?

A) Ibuprofen and warfarin: increased risk of GI bleeding; ibuprofen inhibition of CYP2C9

B) Sotalol and furosemide: increased risk of QT prolongation and torsades de pointes;


furosemideinduced inhibition of CYP3A4

C) Sildenafil and sublingual nitroglycerin: increased risk of hypotension; sildenafil inhibition of the
phosphodiesterase type 5 isoform that inactivates cyclic guanosine monophosphate

D) Ritonavir and lovastatin: increased risk of myotoxicity; ritonavir inhibition of CYP2C19

E) Allopurinol and azathioprine: increased risk of blood dyscrasias; allopurinol inhibition of Pglycoprotein

Answer: C

22. Mr. Jonas is a 47-year-old truck driver with a history of HIV, hypertension, coronary artery disease,
atrial fibrillation, and ischemic cardiomyopathy. He is on antiretroviral therapy. He presents today
complaining of a new rash on his chest and axilla, which you astutely diagnose as tinea corporis. You

would like to prescribe a course of oral ketoconazole for therapy. You should consider dose adjustment
for all of the following medicines that he is already taking EXCEPT:
A) Carvedilol

B) Lovastatin

C) Mexiletine

D) Ritonavir

E) Saquinavir

Answer: A

23. A 24-year-old woman who has tested positive for human immunodefi ciency virus (HIV) presents to
your officewith mild fever and symptoms of an upper respiratory

tract infection. She is currently in her 26th week of pregnancy and has a recent positive tuberculin skin
test (TST) conversion with a 22-mm induration. Additionally, she has genital ulcerative lesions that test
positive for herpes simplex virus. Which antimicrobial agent should you avoid prescribing for this
patient?

A) Acyclovir

B) Amoxicillin

C) Tetracycline

D) Azithromycin

E) Isoniazid

Answer: C

24. Which of the following characteristics of Mycobacterium tuberculosis can be used to distinguish it


from Mycobacterium bovis ?

A) Nonchromogenic pigmentation

B) Positive pyrazinamidase test

C) Negative 68°C catalase test


D) Positive nucleic acid probe for M tuberculosis complex

E) Negative arylsulfatase test

Answer: B

25. A 65-year-old man presents with a left footdrop. On examination,

the left anterior tibialis, extensor hallucis longus, and posterior tibialis muscles are weak. The strength of
the quadriceps, gastrocnemius, and gluteus maximus muscles is normal. Stretch reflexes of the
quadriceps and ankle muscle are normal. To which structure does the lesion best localize?

A) Peroneal nerve

B) L4 spinal root

C) L5 spinal root

D) Sciatic nerve

E) Thoracic spinal cord

Answer: C

26. A 55-year-old male who has a long history of marginally-controlled hypertension presents with
gradually increasing shortness of breath and reduced exercise tolerance. His physical examination is
normal except for a blood pressure of 140/90 mm Hg, bilateral basilar rales, and trace pitting edema.
Which one of the following ancillary studies would be the preferred diagnostic tool for evaluating this
patient? (check one)

A) 12-lead electrocardiography

B) Posteroanterior and lateral chest radiographs

C) 2-dimensional echocardiography with Doppler

D) Radionuclide ventriculography

E) Cardiac MRI
Answer: C

27. Which one of the following extraintestinal manifestations of inflammatory bowel disease do not
parallel the course of intestinal inflammation and do not improve in parallel with improvement in
intestinal symptoms?

A) peripheral arthritis

B) apthous ulcers

C) spondylitis and sacroiliitis

D) erythema nodosum

E) uveitis and iritis

Answer: C

28. An 18-year-old Asian woman is being treated for hepatitis B. Prior to therapy she was found to have
ALT 198 U/L, AST 91 U/L, normal bilirubin, albumin, and prothrombin time. Liver biopsy results showed
chronic hepatitis B, grade 3, stage 3. After 12 weeks of therapy, serum ALT is found to have increased to
1,082 U/L, bilirubin 2.1 mg/dL but albumin and prothrombin time remain normal. Apart from some
fatigue, the patient is tolerating interferon well. The best course of action is:

A) check for antinuclear antibodies and total immunoglobulin level in serum and consider instituting
corticosteroid therapy

B) stop interferon

C) recheck lab work again in 2 weeks time

D) add lamivudine to the regimen

E) ask the patient to skip three scheduled doses of interferon

Answer: C

29. A 53-year-old man presented with hypertension of 150/110 mmHg. He is generally asymptomatic
and has no previous medical history of note. He is a smoker of 5 cigarettes daily and drinks modest
quantities of alcohol. He takes no prescribed medications. Examination reveals a BMI of 33.5 kg/cm2 but
nil else. Investigations: Serum sodium 146 mmol/l (NR 133-145); Serum potassium 3.2 mmol/l (NR 3.5 -
5); Urinary potassium excretion 42 mmol/l (NR less than 30). What is the likely diagnosis?

A) adrenocortical adenoma

B) Bartter's syndrome

C) Liddle's syndrome

D) liquorice ingestion

E) pheochromocytoma

Answer: A

30. A female patient aged 30 has a 5 years history of difficulty getting upstairs and out of a low chair and
mild upper limb weakness but no pain. There is no family history. She presented with severe type 2
respiratory failure. EMG showed evidence of myopathy. 

The most likely diagnosis is:

A) Polymyositis

B) Inclusion body myositis

C) Acid maltase deficiency

D) Miller-Fisher Syndrome

E) Lambert-Eaton Myasthenic syndrome

Answer: C

31. A 28-year-man presents with a 1-week history of fever, headache, malaise, limb aching and
constipation. Examination reveals a pulse rate of 80 beats/minute and high fever. His fever is gradually
rising in a stepladder fashion. The anti-O and anti-H titres are high in the Widal test. Which one of the
following is uncommon during the second week of this man’s illness?

A) Cough
B) Diarrhoea

C) Erythematous spots on the upper abdomen

D) Melaena

E) Splenomegaly

Answer: D

32. Which one of the following diseases shares many clinical characteristics with sarcoidosis?

A) Lymphangioleiomyomatosis

B) Berylliosis

C) Asbestosis

D) Alpha-1 antiprotease deficiency

E) Pulmonary alveolar proteinosis

Answer: B

33. A 19-year-old man presents to the emergency department with 2 days of ascending weakness. He
had diarrhea 3 weeks earlier. He looks comfortable. On examination, you find moderate weakness in all
limbs, with normal strength in facial muscles. You suspect Guillain-Barré syndrome and recommend
admission to Neurology. What should be done first?

A) Spinal tap to look for albuminocytologic dissociation

B) Call for an emergency EMG to verify diagnosis

C) Send the patient for a whole spine MRI to rule out cord compression

D) Obtain pulmonary function tests, including FVC

E) Start high-dose steroids and then move to the floor

Answer: D
34. A 54-year-old woman is started on medical treatment following a diagnosis of acromegaly. Which
one of the following is indicative of active disease?

A) Excessive sweating

B) Polyuria

C) Abdominal pain

D) Arthropathy

E) Goitre

Answer: A

35. Which of the following statements regarding hepatitis B infection in pregnancy is true?

A) Most women of childbearing age with chronic hepatitis B have a high risk of the development of
complications of their disease during gestation.

B) Maternal-fetal transmission is responsible for most cases of hepatitis B worldwide.

C) Mothers who test negative for the hepatitis B e-antigen cannot transmit the virus to their fetuses.

D) Women with hepatitis B can be treated with interferon during pregnancy.

E) Women with hepatitis B should not be treated with lamivudine during pregnancy.

Answer: B

36. All of the following conditions or drugs decrease theophylline clearance, resulting in increased serum
level of this drug EXCEPT:

A) Cimetidine

B) Cirrhosis

C) Allopurinol

D) Rifampin
E) Erythromycin

Answer: D

37. A 39-year-old man presents after tripping over his child’s toy and subsequently falling down the
stairs. The patient reports landing on his chest and now complains of pain over certain regions of his
right chest wall. He endorses some mild pain on deep inspiration. The patient is otherwise healthy and
denies alcohol or smoking. Physical examination demonstrates pain on palpation over the sternum. A
chest x-ray is ordered which confirms two right rib fractures. The radiologist notes an incidental 2.25-cm
coin-shaped lesion on the left upper lung. There is no associated adenopathy or atelectasis noted. Which
of the following is the most appropriate next step with respect to the solitary lung lesion?

A) CT scan of the chest

B) Lung biopsy

C) Review of an old chest x-ray

D) Reassurance

E) C-guided biopsy

Answer: C

38. Which of the following renal pathologies is most likely to recur in a renal transplant patient?

A) Diabetic renal disease

B) Focal segmental glomerulosclerosis 

C) Minimal change glomerulonephritis 

D) Membranous glomerulonephritis 

E) Mesangioproliferative glomerulonephritis

Answer: E
39. In which of the following patients is it appropriate to administer the vaccination against herpes
zoster?

A) A 35-year-old woman who has never had varicella-zoster infection who is 12 weeks pregnant with her
first child

B) A 54-year-old man who has never had varicella-zoster infection and is otherwise healthy

C) A 62-year-old man who had a car accident resulting in splenectomy

D) A 64-year-old woman with dermatomyositis-associated interstitial lung disease treated with


prednisone 20 mg daily and azathioprine 150 mg daily

E) A 66-year-old woman who was recently diagnosed with non-Hodgkin lymphoma

Answer: C

40. A 23-year-old pregnant woman in her first trimester develops hyperthyroidism. Which of the
following treatments is contraindicated?

A) thyroid surgery

B) propylthiouracil (PTU)

C) drugs that cross the placenta

D) radioactive iodine

E) glucocorticoids

Answer: D

41. An elderly patient who becomes acutely short of breath presents with the combination of
hypotension, elevated jugular venous pressure, and muffled heart sounds. This triad of symptoms is
most suggestive of

A) Chronic pericarditis

B) Chronic pericardial effusion

C) Cardiac tamponade
D) Dissecting aortic aneurysm

E) Right heart failure

Answer: C

42. A patient becomes markedly tetanic following a recent thyroidectomy. This symptom can be rapidly
reversed by the administration of

A) Vitamin D

B) Calcitonin

C) PTH

D) Plicamycin (mithramycin)

E) Calcium gluconate (CaG)

Answer: E

43. A 52-year-old female presents with nausea, fatigue, muscle weakness, and intermittent pain in her
left flank. Laboratory examination reveals an increased serum calcium and a decreased serum
phosphorus. The patient’s plasma parathyroid hormone levels are increased, but parathyroid hormone–
related peptide levels are within normal limits. Urinary calcium is increased, and microhematuria is
present. The patient’s abnormality is most likely caused by

A) Primary hyperparathyroidism

B) Primary hypoparathyroidism

C) Pseudohypoparathyroidism

D) Secondary hyperparathyroidism

E) Secondary hypoparathyroidism

Answer: A
44. A 55-year-old female given a general anesthetic for a surgical procedure develops hyperthermia,
hypertension, hyperkalemia, tachycardia, muscle rigidity, and metabolic acidosis. Which of the following
general anesthetics did she receive?

A) Ketamine

B) Midazolam

C) Thiopental

D) Propofol

E) Halothane

Answer: E

45. A patient with reduced VC, FRC, and RV is found to have a normal pH. A tentative diagnosis of diffuse
interstitial fibrosis is made. Which of the following characteristics are consistent with this disease?

A) An increase in lung compliance

B) A decrease in respiratory rate

C) An increase in the V/Q ratio

D) A decrease in PaCO2

E) An increase in the FEV1/FVC ratio

Answer: E

46. An HIV-positive patient asks if you can tell him the chances of him progressing to symptomatic AIDS.
Which one of the following tests would be most useful?

A) CD4 lymphocyte count

B) HIV antibody test

C) HIV RT PCR
D) Neopterin

E) HIV p24 antigen

Answer: C

47. A 33-year-old female patient treated with haloperidol for a history of schizophrenia is seen in the ED
because of complaints of fever, stiffness, and tremor. Her temperature is 104°F, and her serum creatine
kinase (CK) level is elevated. What has occurred?

A) Overdose

B) Allergy

C) Neuroleptic malignant syndrome (NMS)

D) Tardive Dyskinesia

E) Parkinsonism

Answer: C

48. A 43-year-old male is recovering from an infectious disease and experiences a marked instability in
his blood pressure with episodes of spiking of blood pressure. After a series of extensive examinations, it
was concluded that this disorder was due to the effects of the infectious agent upon a component of the
peripheral nervous system. Logical sites where an infectious agent could produce such an effect include
the

A) Superior ganglia of cranial nerves IX and X

B) Geniculate and trigeminal ganglia

C) Otic and superior salivatory ganglia

D) Carotid sinus and aortic arch

E) Carotid and aortic bodies

Answer: D
49. A 25-year-old female suspected of having vitamin D–resistant rickets has decreased blood phosphate
levels. Aside from high-dose vitamin D and oral phosphate, an alternative therapeutic approach might
be the use of which of the following?

A) Estrogen

B) Pamidronate

C) Hydrochlorothiazide

D) Prednisone

E) Calcitrol

Answer: E

50. A 30-year-old female stored her contact lenses in tap water. She noticed deterioration of vision and
visited an ophthalmologist who diagnosed her with severe retinitis. Culture of the water as well as
vitreous fluid would most likely reveal

A) Naegleria

B) Pneumocystis

C) Acanthamoeba

D) Babesia

E) Entamoeba coli

Answer: C

MULTIPLE CHOICE QUESTIONS-5

MULTIPLE CHOICE QUESTIONS-5


1. A 38-year-old woman is referred to you by a neurologist who was seeing her for severe headaches.
His final diagnosis was stress headaches, but because a head MRI showed an empty sella, the
neurologist wanted you to see her. On questioning, you learn that she has 7 children and all are healthy.
When the first child was born, she experienced a difficult delivery and considerable bleeding that
required a transfusion. All subsequent deliveries were uneventful. The review of systems was otherwise
negative. Physical examination shows normally pigmented abdominal striae. The blood pressure is
100/60. You order some blood tests and find that the electrolytes, glucose, and TSH are normal. Which
of the following is the most correct thing for you to do next?

A) Diagnose Sheehan syndrome and begin replacement therapy.

B) Evaluate for pituitary hypofunctioning.

C) Evaluate for pituitary hyperfunctioning.

D) Refer her to a neurosurgeon for exploration of her pituitary gland.

E) Tell her that everything is normal and discharge her from your clinic

Answer: E

2. The condition that characteristically results in recurrent axillary and groin abscesses is:

A) Regional enteritis.

B) Diabetes mellitus.

C) Ulcerative colitis.

D) Hidradenitis suppurativa.

E) Acne conglobata

Answer: D

3. The most important initial therapy for a patient with toxic epidermal necrolysis (TEN) is:

A) Corticosteroids.

B) Antibiotics.
C) Anticoagulants.

D) Analgesics.

E) Crystalloids

Answer: E

4. In evaluating the skin lesions of patients with AIDS:

A) Kaposi sarcoma is common but seldom more than a cosmetic problem.

B) Candidiasis may occur but is easily treated with standard medications.

C) Tinea corporis is no more likely in these patients than in the general population.

D) Lichen planus of the oral mucosa is associated with the disease.

E) Seborrheic dermatitis-like eruptions are frequently found

Answer: A 

5. A 57-year-old man comes to the emergency department because of excruciating pain in his right big
toe. He describes the pain as so severe that it woke him from a deep sleep. He has no chronic medical
conditions, does not take any medications, and denies any similar episodes in the past. He admits to a
few "drinking binges" over the past 2 weeks. His temperature is 38.1 C (100.5 F), blood pressure is
130/90 mm Hg, and pulse is 80/min. Examination shows an erythematous, warm, swollen, and
exquisitely tender right great toe. The skin overlying the first metatarsophalangeal joint is dark red,
tense, and shiny. Synovial fluid analysis reveals negatively birefringent, needle-shaped crystals within
polymorphonuclear leukocytes (PMNs). Laboratory studies show: 

Serum Leukocytes........16,000/mm3; Uric acid...........15 mg/dL; Calcium.............9 mg/dL 

Which of the following is the most appropriate pharmacotherapy? 

A. Allopurinol 

B. Ceftriaxone 

C. Cyclooxygenase-2 inhibitors but in higher dosages than are typically used. 


D. Probenecid 

E. Sulfinpyrazone

Answer: C

6. A 28-year-old woman is diagnosed with lupus nephritis, World Health Organization (WHO) type IV.
She has a malar rash, diffuse arthritis, and edema. Her blood pressure is 190/110 mm Hg. Her creatinine
is 2.1 mg/dL with a blood urea nitrogen of 28 mg/dL. Her urine reveals 25 red blood cells per hpf, and 3+
protein. One red blood cell cast is seen. A 24-hour urine collection reveals a protein of 11 grams with a
creatinine of 1 gram. Which of the following would be the most appropriate management? 

A) Oral azathioprine 

B) Oral cyclophosphamide 

C) Oral gold 

D) Oral prednisone 

E) Pulse IV cyclophosphamide

Answer: E

7. A 50-year-old man is evaluated for a routine physical examination. He was diagnosed with rheumatoid
arthritis 5 years ago, and his condition is well controlled with methotrexate and hydroxychloroquine. He
does not have fatigue, morning stiffness, or systemic symptoms. His weight is stable. On physical
examination, there are no joint deformities. There is minimal soft-tissue swelling around the second and
third metacarpophalangeal joints. Grip strength is normal, but squeezing the forefeet elicits mild
discomfort. On laboratory studies, hemoglobin is 14.2 g/dL (142 g/L), alanine and aspartate
aminotransferase and alkaline phosphatase are normal, and erythrocyte sedimentation rate is 65 mm/h.
Which of the following is the most appropriate management for this patient?

A. Prednisone, 10 mg daily

B. Prednisone, 40 mg daily

C. Anti–tumor necrosis factor therapy

D. CT scan of the chest, abdomen, and pelvis


E. No additional therapy

Answer: D

8. A family physician cares for a family consisting of a 45-year old husband, 43-year-old wife and a 12-
year-old daughter. The family reports that recently the 77-year-old maternal grandmother who lived
with them died after a prolonged respiratory infection. Autopsy subsequently confirms that she had
active pulmonary tuberculosis at the time of death. The organism tested sensitive to all anti-tuberculosis
drugs. In responding to the grandmother's illness, which of the following is the most appropriate step in
management?

A) Obtain leukocyte counts on all family members 

B) Obtain sputum cultures for acid fast bacilli 

C) Obtain chest computerized tomograms on all members 

D) Place protein purified derivative (PPD) test on all members 

E) Schedule bronchoscopy lavage for the adults

Answer: D

9. A patient with a history of papillary thyroid cancer is noted to

have an enlarged lateral neck lymph node on routine

examination. Which one of the following is the most sensitive and specific indicator of metastatic
disease within the enlarged lymph

node?

A. Neck ultrasound

B. Fine needle aspiration cytology

C. Stimulated serum thyroglobulin

D. PET‐CT scanning

E. Thyroglobulin wash‐out testing


Answer: D

10. A 60-year-old woman is establishing care in your clinic. She has occasional numbness and tingling in
her fingers and toes and has noted some numbness around her mouth, especially when she is stressed
or anxious. She had thyroid surgery for Graves’ disease about 2 years ago and takes 100 µg of
levothyroxine and 1 tablet of calcium daily. On exam, her blood pressure is 130/80, pulse 80, and she
has cramping in her right forearm and fingers when the blood pressure cuff is attached. Based on this
history and exam, which of the following is most likely?

A) Calcium 8.5, PTH 65, PO4 4.5

B) Calcium 9.5, PTH 35, PO4 4.0

C) Calcium 6.0 (8.5-10.5), PTH 2 (10-65), PO4 6.0 (2.7-4.5)

D) Calcium 10.8, PTH 108, PO4 2.3

E) Calcium 8.0, PTH 98, PO4 2.1

Answer: C

11. Which of the following sets of drug–drug interaction and mechanism is accurately described?

A. Ibuprofen and warfarin: increased risk of GI bleeding; ibuprofen inhibition of CYP2C9

B. Sotalol and furosemide: increased risk of QT prolongation and torsades de pointes;


furosemideinduced inhibition of CYP3A4

C. Sildenafil and sublingual nitroglycerin: increased risk of hypotension; sildenafil inhibition of the
phosphodiesterase type 5 isoform that inactivates cyclic guanosine monophosphate

D. Ritonavir and lovastatin: increased risk of myotoxicity; ritonavir inhibition of CYP2C19

E. Allopurinol and azathioprine: increased risk of blood dyscrasias; allopurinol inhibition of Pglycoprotein

Answer: C
12. A 47-year-old truck driver with a history of HIV, hypertension, coronary artery disease, atrial
fibrillation, and ischemic cardiomyopathy. He is on antiretroviral therapy. He presents today
complaining of a new rash on his chest and axilla, which you astutely diagnose as tinea corporis. You

would like to prescribe a course of oral ketoconazole for therapy. You should consider dose adjustment
for all of the following medicines that he is already taking EXCEPT:

A. Carvedilol

B. Lovastatin

C. Mexiletine

D. Ritonavir

E. Saquinavir

Answer: A

13. A 17-year-old patient who is known to have Wolff-Parkinson-White syndrome presents with a
regular narrow complex tachycardia with a cycle length of 375 milliseconds (160 bpm) that occurred
with a sudden onset. You note that there is a 1:1 atrial-to-ventricular relationship and that the RP
interval is 100 milliseconds. The best initial treatment is

A) IV procainamide.

B) atropine.

C) vagal maneuvers.

D) catheter ablation

Answer: C

14. A 65-year-old man presents after an arrest while eating at a local restaurant. On arrival, paramedics
documented ventricular fibrillation (VF), and he was successfully resuscitated. He has a history of
myocardial infarction (MI) and congestive heart failure (CHF). Serum electrolytes are remarkable only for
mild hypokalemia. MI is ruled out by ECG and serial blood tests of myocardial enzymes. Subsequent
evaluation includes cardiac catheterization, which shows severe three-vessel coronary artery disease
(CAD) and severe left ventricular (LV) systolic dysfunction. A nuclear myocardial perfusion scan shows a
large area of myocardial scar without significant viability in the territory of the left anterior descending
coronary artery. The decision is made to treat the CAD medically. Which of the following is the best
management strategy for his arrhythmia?

A) PO amiodarone

B) Implantable cardioverter defibrillator (ICD) implantation if an electrophysiologic (EP) study shows


inducible VT or VF

C) ICD implantation

D) β-Blocker medication

Answer: C

15. A 56-year-old woman is undergoing a cadaveric renal transplant. After revascularization of the
transplanted kidney the transplanted renal parenchyma becomes swollen and blue. Which of the
following statements is most accurate regarding her transplanted kidney?

A) The donor had pre-formed antibodies against the recipient’s HLA antigens.

B) It is characterized pathologically by fibrin and platelet thrombosis of renal arterioles and small
arteries and necrosis of the glomerular tufts.

C) Biopsies should not be obtained intraoperatively.

D) This form of rejection is associated with disseminated intravascular coagulation (DIC).

E) The rejection process can be treated with a steroid bolus and OKT3

Answer: C

16. A 55-year-old man comes to the physician with the chief complaint of weight loss and a depressed
mood. He feels tired all the time and is no longer interested in the normal activities he previously
enjoyed. He feels quite apathetic overall. He has also noticed that he has frequent, nonspecific
abdominal pain. Which of the following diagnoses needs to be ruled out for this man?

A) Pheochromocytoma

B) Pancreatic carcinoma
C) Adrenocortical insufficiency

D) Cushing syndrome

E) Huntington disease

Answer: B

17. You are seeing a 78-year-old man who was brought to the office by his daughter. The daughter says
her father is becoming increasingly forgetful. His medical history is significant for a 20-year history of
type 2 diabetes and well-controlled hypertension. On examination, he is mildly hypertensive with
otherwise normal vital signs. He is oriented to time, place, and person, but is unable to complete “serial
sevens” on a mini-mental status examination. Which of the historical features make this diagnosis more
consistent with dementia as opposed to delirium?

A) His history of hypertension

B) His history of diabetes

C) His current level of orientation

D) His inability to complete serial sevens

E) The recent onset of his symptoms

Answer: C

18. A 45-year-old woman with Crohn’s disease and a small intestinal fistula develops tetany during the
second week of parenteral nutrition. The laboratory findings include Ca 8.2 meq/L; Na 135 meq/L; K 3.2
meq/L; Cl 103 meq/L; PO4 2.4 meq/L; albumin 2.4; pH 7.48; 38 kPa; P 84 kPa; bicarbonate 25 meq/L. The
most likely cause of the tetany is

A) Hyperventilation

B) Hypocalcemia

C) Hypomagnesemia

D) Essential fatty acid deficiency

E) Focal seizure
Answer: C

19. A 60 year-old woman has had a mitral valve replacement for chronic mitral stenosis and is on cardiac
ITU post-operatively. She is awake and self ventilating. SPO2 91%, FiO2= 0.6. Monitoring shows atrial
fibrillation at 90/min, blood pressure 88/60 and pulmonary artery pressures of 45/15. Which single drug
is most appropriate first line treatment to reduce the pulmonary artery pressure? 

A) Inhaled nitric oxide 

B) Inhaled prostacyclin 

C) Oral sildenafil 

D) Intravenous isoprenaline 

E) Intravenous milrinone

Answer: E

20. A 62-year-old man is admitted to the emergency department with abdominal pain. The patient has a
past history of ischaemic heart disease and atrial fibrillation. Computed tomography scan features are
highly suggestive of ischaemic bowel. The patient's blood gases are as follows: pH = 7.25; paO2 = 10;
paCO2 = 2.8; HCO3 = 18; Base excess = –8 

Which of the following best describes the patient's acid–base status? 

A) Metabolic acidosis 

B) Metabolic acidosis with respiratory compensation 

C) Respiratory acidosis with metabolic compensation 

D) Metabolic acidosis with inadequate respiratory compensation 

E) Cannot be sure without a serum lactate level

Answer: D
21. Two weeks ago, a 54-year-old man developed a cold sore, which resolved uneventfully. He now
complains of a nonpruritic skin rash. Examination reveals raised red lesions resembling hives, some with
clear fluid bullae. They are located on his hands, including the palms, and his forearms and anterior tibia.
The best way to confirm your suspected diagnosis is by:

A) Viral culture of blister fluid.

B) Smear of blister fluid for Gram stain.

C) Tzanck preparation of blister fluid to look for multinucleate giant cells.

D) Full thickness skin biopsy of involved area.

E) Wood’s light examination of involved areas

Answer: D

22. A 54-year-old man presents for a periodic health examination. His family history is significant for his
mother who died of a cerebrovascular accident at age 72, his father who died of a myocardial infarction
at age 68, and a brother who developed sigmoid cancer at age 60. The patient is on no medications
except for aspirin, 81 mg daily. His physical examination is unremarkable. The patient asks for a
recommendation regarding current cancer screening. Which of the following is the most appropriate
screening test for this patient? 

A) Annual digital rectal examination and fecal occult blood testing 

B) Flexible sigmoidoscopy 

C) Flexible sigmoidoscopy and barium enema 

D) Colonoscopy 

E) Genetic testing for the p53 gene

Answer: D

23. Which of the following treatment options has been most consistently shown to be effective for the
primary prevention of sudden cardiac death in patients with CAD and recent MI?

a. D-Sotalol
b. β-Blocker medications

c. Amiodarone

d. Dofetilide

Answer: B

24. A 72-year-old woman is about to undergo an elective total hip replacement for osteoarthritis. She
has a history of hypertension and type 2 diabetes mellitus but no ischaemic heart disease or peripheral
vascular disease. Which of the following is the most appropriate thromboembolic prophylaxis? 

A) Intermittent pneumatic calf compression 

B) Calf-length thromboembolic deterrent elastic stockings and early ambulation 

C) Full-dose unfractionated heparin to increase the activated partial thromboplastin time to two time
control 

D) Insertion of an inferior vena cava filter 

E) Subcutaneous low-molecular-weight heparin

Answer: E

25. An AIDS patient under treatment with a nucleoside analog and a protease inhibitor comes to medical
attention with complaints of leg weakness and incontinence. His vital signs are within normal limits.
Physical examination reveals reduced strength in the lower extremities with accompanying mild
spasticity. There is also diminished sensation in the feet and legs bilaterally. Lumbar puncture shows:

Opening pressure.....100 mm H20

Cell count................5 lymphocytes/mm3

Glucose...................48 mg/dL

Proteins, total..........33 mg/dL

Gamma globulin.......8% total protein


Additional laboratory investigations show normal hematologic parameters, vitamin B12 within normal
values, and negative serology for syphilis. MRI of the head fails to reveal any focal abnormality. Which of
the following is the most likely diagnosis?

A) AIDS dementia complex 

B) CMV polyradiculopathy 

C) Cryptococcal meningoencephalitis 

D) Vacuolar (HIV) myelopathy 

E) Zidovudine-related toxicity

Answer: D

26. A 59-year-old man presents to the ED with left-sided chest pain and shortness of breath that began 2
hours prior to arrival. He states the pain is pressure-like and radiates down his left arm. He is
diaphoretic. His BP is 160/80 mm Hg, HR 86 beats per minute, and RR 15 breaths per minute. ECG
reveals 2-mm ST-segment elevation in leads I, aVL, V3 to V6. Which of the following is an absolute
contraindication to receiving thrombolytic therapy?

A) Systolic BP greater than 180 mm Hg

B) Patient on Coumadin and aspirin

C) Total hip replacement 3 months ago

D) Peptic ulcer disease

E) Previous hemorrhagic stroke

Answer: E

27. Which of the following statements is true about the patient with a bleeding disorder?

A) Factor deficiencies generally present with mucosal bleeding/petechiae.

B) Hemarthrosis generally reflects a platelet deficiency.

C) Hematomas are usually the result of a factor deficiency.


D) All of the above are true

Answer: C

28. Flu season is right around the corner and you are preparing your clinic for the onslaught. First things
first … you need to know how much vaccine to order and who will be receiving it. The Centers for
Disease Control and Prevention (CDC) annually publishes recommendations for administering influenza
vaccine to the American public. The CDC recommends vaccination for all of the following groups,
EXCEPT:

A) Health-care workers.

B) Nursing home residents.

C) Egg-allergic, febrile neonates.

D) Diabetics.

E) The elderly

Answer: C

29. During an outbreak, what intervention(s) is/are most appropriate for all your unvaccinated, frail
nursing home patients who have no symptoms of febrile respiratory illness?

A) Antiviral prophylaxis with oseltamivir.

B) Antiviral prophylaxis with amantadine.

C) Influenza immunization.

D) A and C given together.

E) B and C given together

Answer: D
30. Which of the following is indicated when evaluating for a suspected inherited coagulopathy?

A) CBC.

B) PT and PTT.

C) Platelet count.

D) PFA-100.

E) All of the above

Answer: E

31. A 43-year-old woman with rheumatoid arthritis visits for a scheduled follow-up. Plain X-ray of the
hands reveals marginal erosions at the metacarpal heads. Why are these erosions marginal?

A) Random localization

B) Plain X-rays fail to show central erosions

C) Rotation of the film

D) Marginal joint area is devoid of overlying cartilage

E) Presence of sesamoid bones

Answer: D

32. A 23-year-oldman attends his GP complaining of reduced exercise tolerance, nocturia and frequent
hiccups over the course of the past 4 months. He also has generalised bony pain. His BP is 165/90 mmHg
and his BMI is stable at 25.

The results of routine blood testsshow:

Creatinine 250 μmol/l

Haemoglobin 14g/dl

Phosphate 2.0 mmol/l

Calcium 2.1 mmol/l


What is the most likely cause of these results?

A) Adult polycystic kidney disease 

B) Diabetes

C) Contrast nephropathy 

D) Hypertensive nephropathy

E) Diffuse proliferative glomerulonephritis

Answer: A

33. A 41-year-old man with a history of nasal congestion, breathlessness, cough and wheeze presents
with a left foot drop. Which one of the following is the most likely diagnosis?

A) Diabetes mellitus

B) Wegener’s granulomatosis

C) Churg–Strauss syndrome

D) Pulmonary eosinophilia

E) Polyarteritis nodosa

Answer: C

34. The preferred initial therapy for elderly patients with arthralgia due to osteoarthritis is which of the
following?

A) NSAIDs.

B) COX-2 inhibitors.

C) Acetaminophen.

D) Combination narcotic analgesics.

E) Early joint replacement.


Answer: C

35. According to the Eighth Joint National Committee (JNC-8) guideline on managing hypertension in 60
years old patient or more (with no DM or chronic kidney disease) the BP lowering goal is:

A) <150/90 mm Hg

B) <140/90 mm Hg

C) <160/90 mm Hg

D) <120/90 mm Hg

E) <130/90 mm Hg

Answer: A

36. In general, all of the following are risk factors for gout EXCEPT:

A) Tobacco use.

B) Alcohol use.

C) Obesity.

D) Diuretic use.

E) Family history.

Answer: A

37. A 59-year-old woman had three episodes of right renal colic over the last year. Review of system was
remarkable for joint pain and fatigability. Past and family history were unremarkable. She is 7 years
postmenopause and takes no medication. On physical examination BMI 28.5 kg/m2, pulse 72 beat per
minute, BP 150/95 mmHg. Laboratory data revealed. Fasting plasma glucose 89 mg/dl: Serum creatinine
1.1 mg/dl: Serum total calcium 11.9 mg/dl: Serum ionized calcium 6.8 mg/dl: Ultrasound of the
abdomen revealed 2 stones at the right renal pelvis: 24 hour urinary calcium 365 mg (N < 250 mg) The
most appropriate next diagnostic test is:

A) Measurement of serum 25 hydroxyvitamin D.

B) Measurement of serum PTH.

C) Meseurement of serum parathyroid related protein.

D) Genetic testing for mutation in parathyroid calcium sensing receptor.

E) 24 hour urinary uric acid.

Answer: B

38. A woman presents with headache, lethargy and weight loss. Which one of the following would make
the diagnosis of giant cell arteritis unlikely?

A) A raised ESR

B) Bilateral headache

C) Non-tender temporal arteries

D) Papilloedema on fundoscopy

E) Patient is 60 years old

Answer: D

39. Which of the following statements regarding polycythemia vera is correct?

A) An elevated plasma erythropoietin level excludes the diagnosis.

B) Transformation to acute leukemia is common.

C) Thrombocytosis correlates strongly with thrombotic risk.

D) Aspirin should be prescribed to all these patients to reduce thrombotic risk.

E) Phlebotomy is used only after hydroxyurea and interferon have been tried.
Answer: A

40. A 49-year-old female with a 5-year history of diabetes mellitus type 2 presents for an initial visit. She
has no known complications of diabetes. She takes metformin, glyburide, andaspirin. On examination,
you find a pleasant, obese female in no distress. Her blood pressure is 136/86 mm Hg, pulse 86,
respirations 14, and temperature 37°C. As you discuss monitoring her diabetes, you recommend
screening for early kidney disease. Which of the following approaches is the recommended way to
screen for diabetic kidney disease?

A) Obtain a 24-hour urine collection for albumin now and again in 3 years.

B) Obtain a spot urine microalbumin every year.

C) Obtain a spot urine microalbumin/creatinine ratio every year.

D) Obtain a urinalysis every year.

E) Obtain a serum creatinine every year.

Answer: C

41.  A 34-year-old female presents with recurrent episodes of severe headaches, palpitations,
tachycardia, and sweating. A physical examination reveals her blood pressure to be within normal limits;
however, during one of these episodes of headaches, palpitations, and tachycardia, her blood pressure
is found to be markedly elevated. Workup finds a small tumor of the right adrenal gland. Which one of
the following is most likely to be increased in the urine of this individual?

A) Acetone

B) Aminolevulinic acid (ALA)

C) Hydroxy-indoleacetic acid (HIAA)

D) N-formiminoglutamate (FIGlu)

E) Vanillylmandelic acid (VMA)

Answer: E
42. One of your 50-year-old male patients with CKD 5 is considering PD as one of his choices for renal
replacement therapy. He asks you about indications and contraindications about PD. Which one of the
following choices is a contraindication for PD?

A) Abdominal hernia not amenable to surgery

B) Adhesions from previous surgery

C) Severe inflammatory bowl disease

D) Diaphragmatic fluid leak

E) All of the above

Answer: E

43. Foreign bodies and/or food boluses can lodge in the esophagus in any of the following four areas of
narrowing except:

A) Hiatal hernia

B) Upper esophageal sphincter

C) Level of the aortic arch

D) Level of the mainstem bronchus

E) Gastroesophageal junction

Answer: A

44. A 62-year-old man with CKD 5 (eGFR 11 mL/min) due to HTN comes to the clinic for routine follow-
up with no complaints of nausea, vomiting, fatigue, or poor appetite. An arterio-venous fistula (AVF) was
placed 1 year ago when his eGFR was 12 mL/min and the fistula is ready for use. His BP is 134/80 mmHg.
He walks 2 miles everyday without shortness of breath, or chest pain, or fatigue. Pertinent labs include:
Na+ 139 mEq/L, K+ 4.4 mEq/L, HCO3 22 mEq/L, BUN 68 mg/dL, Ca2+ 8.8 mg/dL, phosphate 4.2 mg/dL,
and albumin 4.1 g/dL. He expresses hemodialysis (HD) as his choice of renal replacement therapy (RRT).
According to the KDIGO guideline, which one of the following is the MOST appropriate management in
this patient?

A) Start HD in 2 weeks at the outpatient dialysis unit

B) Start peritoneal dialysis (PD) in 4 weeks

C) Convince for preemptive kidney transplantation

D) Start HD when signs and symptoms of kidney failure are present

E) Suggest no RRT at any time, as he may do well with conservative management

Answer: D

45. A 75-year-old man underwent surgery to correct a large abdominal aortic aneurysm. The procedure
appeared to go well, but you are called a few hours later to evaluate the patient who states that he
cannot move or feel his legs. On the way to the ICU, you consider the possible causes of his symptoms
and plan your physical examination. What is the most important test to help localize the lesion?

A) MRI of the spine

B) Sensory level

C) Reflexes in lower extremities

D) Plantar flexion reflex

E) Toe position sense

Answer: E

46. Which one of the following statements in CKD 5 patients regarding early (eGFR 10–14 mL/min)
versus late (eGFR 5–7 mL/min) initiation of HD is CORRECT?

A) Early initiation of HD improves mortality and morbidity of patients

B) Late initiation of HD improves mortality and morbidity of patients

C) No difference between early and late initiation of HD either in survival or other outcomes such as
hospitalizations or quality of life
D) Compared to early initiation, late initiation is better in controlling mortality only

E) None of the above

Answer: D

47. A 69-year-old man was successfully defibrillated after an episode of ventricular fibrillation secondary
to an ST elevation myocardial infarction (STEMI) and transferred to the cardiac catheter laboratory for
primary coronary intervention. After the procedure began he had a further episode of ventricular
fibrillation. Regarding defibrillation, which is true?

A) A single direct current shock of 360 joules with a biphasic waveform is the most likely to restore
spontaneous circulation

B) Defibrillation is no more likely to be successful than a properly delivered praecordial thump

C) It is safe to continue with the coronary angiogram while the shock is delivered to the patient

D) Three shocks delivered with minimal interruptions should be given before any other intervention

E) Two minutes of chest compressions before defibrillation is recommended to optimize coronary


perfusion

Answer: D

48. Which of the following statements regarding pillinduced esophagitis is true?

A) A bronchoesophageal fi stula is a common complication of NSAID-induced esophageal ulceration.

B) Tetracycline or its derivatives cause pillinduced esophagitis by production of a caustic alkaline


solution.

C) Antibiotics as a class are uncommon causes of medication-induced esophagitis.

D) Esophageal damage from a bisphosphonate medication, such as alendronate, can be minimized by


ingestion of a full 8-oz glass of water taken in the upright position.

E) Chemotherapeutic agents are unlikely causes of pill-induced esophagitis.


Answer: D

49. A 32-year-old nurse presents with symptoms of dizziness, jittery behavior, and headaches before
meals. Which of the following supports the diagnosis of factitious hypoglycemia?

A) Elevated sulfonylurea levels

B) Normal proinsulin levels

C) Normal C-peptide levels

D) Plasma insulin-to-glucose ratio <0.3

E) All of the above

Answer: E

50. All of the following paraneoplastic syndromes are associated with gastric cancer except:

A) Acanthosis nigrans

B) Disseminated intravascular coagulation

C) Thrombophlebitis (Trosseau’s sign)

D) Pyoderma gangrenosum

E) Nephrotic syndrome

Answer: D

MULTIPLE CHOICE QUESTIONS-6

1. A broncho-pleural fistula is an abnormal communication or a passage between the bronchial tree and
the pleural space, causing a persistent leak. If these patients are mechanically ventilated, the
management strategy should be:

A) Low tidal volumes and high respiratory rate


B) Reduced inspiratory pressures

C) High tidal volumes and low respiratory rate

D) Low inspiratory times and high PEEP

E) High inspiratory times and low PEEP

Answer: B

2. Which of the following is a recognized feature of pulmonary embolism except?

A) S1,Q3,T3

B) an increase in serum troponin levels

C) an arterial pH greater than 7.2

D) increased PCO2 on air

E) positive D-dimer levels

Answer: D

3. A 45-year-old surgeon sustained a needle stick injury from a known HIV-positive individual during a
routine gall bladder operation and subsequently commenced a course of post-exposure prophylaxis
(PEP). Which of the following statements regarding his management is CORRECT?

A) He can discontinue PEP if it is established that the source patient has an undetectable HIV viral load

B) He must refrain from operating until he has proven to be HIV-negative after completion of PEP

C) If he has successfully completed a course of PEP, there is no requirement for him to have a
subsequent HIV antibody test

D) He cannot be considered HIV-negative until he tests HIV antibody-negative at three months post-PEP

E) He cannot be considered HIV-negative until he tests HIV antibody-negative at six months post-PEP

Answer: E
4. A patient presents with gradually worsening weakness of the proximal arm and leg muscles
symmetrically over several months. On examination, neck flexors and extensors are found to be weak
also. There is no muscle pain or tenderness. What is the most likely site of dysfunction in the nervous
system?

A) Peripheral nerve

B) Brachial plexus

C) Spinal nerve root

D) Internal capsule

E) Muscle

Answer: E

5. A 67 year old lady is referred to the bone clinic following a fractured wrist. A DEXA scan is performed
which shows a T-score of −2.0. She has no other osteoporotic risk factors. Which one of the following
treatments would you offer?

A) Lifestyle advice and vitamin D supplementation

B) Alendronic acid

C) HRT

D) Risedronate

E) No treatment

Answer: A

6. A 56-year-old man is admitted to the intensive care unit for acute respiratory distress syndrome
(ARDS). The patient was transferred from an outside hospital today after a 2-week hospitalization for
pneumonia. During that time, the patient's pulmonary status continued to deteriorate. One week ago he
was intubated and placed on mechanical ventilation and over the past week, his oxygenation has
worsened with a PaO2 of 66 on an inspired concentration of 100% oxygen. The patient has no other
medical history except for rheumatoid arthritis. On transfer to the ICU, the patient is intubated and
sedated on a mechanical ventilator. His chest radiograph shows patchy, bilateral, diffuse interstitial
infiltrates. The most important intervention that will most benefit this patient is to

A) keep the patient in a prone position during mechanical ventilation

B) keep tidal volumes greater than 15 cc/kg

C) limit peak inspiratory pressure to 45 cm H2O or less

D) limit PEEP levels to less than 10 cm H2O

E) limit tidal volumes to 6cc/kg

Answer: E

7. A previously fit and well 50-year-old male underwent a decompressive craniectomy for an acute
subdural haematoma following a traumatic head injury 24 hours ago. He is now on the neurosurgical
intensive care unit and remains intubated and ventilated. You have been asked to review his urine
output, which is recorded as 800 ml over the last 2 hours, despite appropriate intravenous fluid
administration. You suspect neurogenic diabetes insipidus.

Which of the following features would support your diagnosis?

A) Urine specific gravity >1.005

B) Serum Na+ <135 mmol/l

C) Urine osmolality <350 mmol/kg

D) Serum osmolality <295mmol/kg

E) Serum creatinine 200 mmol/l

Answer: B

8. A 35 year old man presents to his GP with a 2-week history of painful, swollen knees and a painful left
heel. He also reveals that he has been experiencing a burning pain when he urinates and has a red, itchy
eye. Which one of the following is the most likely diagnosis?

A) Gout
B) Osteoarthritis

C) Enteropathic arthritis

D) Ankylosing spondylitis

E) Reactive arthritis

Answer: E

9. A 40-year-old female who is intubated and ventilated following a subarachnoid hemorrhage (SAH) 7
days previously has a serum sodium concentration of 128mmol/l and serum osmolality of 270 mOsm/kg.
Which of the following statements is true?

A) Cerebral salt-wasting syndrome (CSWS) is rarely associated with SAH

B) Cerebral salt-wasting syndrome is associated with a reduced serum osmolality

C) To diagnose SIADH, the patient must be clinically dehydrated

D) SIADH almost always requires pharmacological treatment

E) To diagnose SIADH urine osmolality must be greater than serum osmolality

Answer: E

10. A 47-year-old woman who is 2 weeks post triple bypass surgery presents to the emergency
department with a chief complaint of sudden onset, sharp chest pain for several hours. She is fatigued
and short of breath. On physical examination she has distended neck veins that grow more distended on
inspiration. Muffled heart sounds are heard. Her temperature is 37.0°C (98.6°F), pulse is 133/min, blood
pressure is 70/50 mm Hg, respiratory rate is 30/min, and oxygen saturation is 100% on room air. An
echocardiogram shows a large pericardial effusion and chamber collapse; therefore, pericardiocentesis
is performed. Although a large amount of blood is aspirated, the patient’s clinical picture acutely
worsens. Her pain level increases substantially; pulse is 150/min, blood pressure is 60/41 mm Hg,
respiratory rate is 30/min, and oxygen saturation is 100%. Repeat echocardiography shows an even
larger pericardial effusion with chamber collapse. Which complication of pericardiocentesis is most likely
in this patient?

(A) Acute left ventricular failure with pulmonary edema


(B) Aspiration of 10 mL air into the pericardium

(C) Laceration of a coronary vessel

(D) Pneumothorax

(E) Puncture of the left ventricle

Answer: C

11. During this outbreak, what intervention(s) is/are most appropriate for all your unvaccinated, frail
nursing home patients who have no symptoms of febrile respiratory illness?

A) Antiviral prophylaxis with oseltamivir.

B) Antiviral prophylaxis with amantadine.

C) Influenza immunization.

D) A and C given together.

E) B and C given together

Answer: D

12. A 38-year-old male presents with episodic wheeze and non-productive cough which occurs
particularly at night. He has been employed in the plastics industry. Which of the following suggests a
diagnosis of occupational lung disease?

A) Absent family history of asthma

B) Commencement of symptoms on his first day in this employment

C) Elevated serum IgE concentration

D) Improved symptomatology when on holiday

E) Increased bronchial reactivity

Answer: D
13. A 68-year-old male is admitted with a two months history of difficulty raising his arms, ascending
stairs, and is also aware of a dry mouth. He smokes 15 cigarettes daily and admits to heavy alcohol
consumption. On examination he has proximal weakness affecting all four limbs with absent tendon
reflexes. His chest X-ray shows a right pleural effusion. What is the most likely diagnosis?

A) Alcohol induced myopathy

B) Eaton- Lambert syndrome

C) Myasthenia gravis

D) Polymyalgia rheumatica

E) Polymyositis

Answer: B

14. A 65-year-old woman, a heavy smoker for many years, has had worsening dyspnoea for the past five
years, without a significant cough. A chest x ray shows increased lung size along with flattening of the
diaphragm, consistent with emphysema. Over the next several years she develops worsening peripheral
oedema. Her BP 115/70 mmHg. Which of the following cardiac findings is most likely to be present?

A) Constrictive pericarditis 

B) Left ventricular aneurysm 

C) Mitral valve stenosis 

D) Non-bacterial thrombotic endocarditis 

E) Right ventricular hypertrophy

Answer: E

15. A young male developed spontaneous DVT of right lower limb and he is having a sister who had DVT
5 years back and having protein C deficiency. He was started on warfarin and the INR came to 2-3 after
which he had started following at local hospital where his warfarin dose was steadily increased as his
INR never went higher than 1.5 and is currently on warfarin dose of 20 mg/day. He was referred back to
hospital and was evaluated and found to be having warfarin level of 2.385 mg/l (therapeutic range 0.7 –
2.3 mg/l), PIVKA is > 10 

(Ref. range < 0.2). What is the most likely explanation for the subtherapeutic INR?

A) Cytochrome p450 mutation

B) VKORC 1 mutation

C) Not taking warfarin

D) Local hospital INR testing quality control is not good

Answer: D

16. Which of the following anatomical considerations is correct?

A) Optic chiasm lesions characteristically produce a bitemporal hemianopia.

B) Central scotoma occurs early in papilloedema.

C) In cortical blindness pupillary reactions are abnormal.

D) Optic tract lesions produce an ipsilateral homonymous hemianopia.

E) Opticokinetic nystagmus is found with bilateral infarction of the parieto-occipital lobes

Answer: A

17. The most common non-opportunistic protozoon parasite is AIDS patients is:

A) Cryptosporidium 

B) Gardia lamblia 

C) Blastomyces hominis 

D) Entamoeba histolytica 

E) Toxoplasma gondii
Answer: D

18. To assist in the decision whether to hospitalize a patient with community acquired pneumonia (CAP),
each of the following may be a factor in favor of hospitalization except for which one?

A) The patient is confused

B) Serum creatinine >2.0 mg/dL

C) Respiratory rate >30

D) Blood pressure < 90 mm Hg

E) Age >64 years

Answer: B

19. A 35-year-old African-American woman complains of red and irritated eyes with photophobia for
about 2 months. Visual acuity is 20/25 for each eye separately, and she says this is her “normal.” On
questioning she admits to shortness of breath with exertion, which she attributes to neglecting physical
training and advancing age. A chest x-ray shows perihilar adenopathy. A lung biopsy finds noncaseous
granuloma. Angiotensinconverting enzyme is elevated. On spirometry, FEV 1 is 80% of predicted normal
for her (percent of vital capacity expired in 1 second). Which of the following would be the best
therapeutic approach.

A) Non-steroidal anti-inflammatory drugs (NSAIDs)

B) Bronchodilators

C) Inhaled glucosteroids

D) Observation for 4–6 months while treating the eyes symptomatically

E) Systemic glucosteroids

Answer: D

20. A 30 year old lady underwent prosthetic valve replacement surgery done for rheumatic heart disease
and she is started on UFH by continuous infusion plus warfarin. Six days after she developed DVT of right
lower limb. Her CBC was normal preoperatively and now she has dropped her platelet count to
50,000/mm3. Her INR is 2.3 and  APTT is 90 (control 35). The best therapeutic option would be:

A) IVIg followed by steroids

B) Fondaparinux

C) Stop heparin and add aspirin to warfarin

D) Change to low molecular weight heparin

Answer: B

21. A 59-year-old male presents with a 1 hour history of central crushing chest pain. He is known to be
diabetic, hypertensive and is a non-smoker. On examination his pulse rate is 90 beats/min, blood
pressure 130/85 mmHg, S1 S2 are audible with no murmurs. There is no evidence of cardiac failure. An
ECG is performed. Which of the following would be an indication for thrombolysis?

A ) Right bundle branch block

B ) Supraventricular tachycardia

C ) ST elevation of 2mm in V4-V6

D ) ST depression of 2mm in leads II,III, avF

E ) Atrial fibirillation >150min-1

Answer: C

22. A 46-year-old Caucasian woman complains of increasingly severe fatigue that she believe emanates
from poor sleep quality. Her husband notes that she moves frequently during her sleep, and the patient
notes that recently as she begins to ready herself for sleep she has discomfort in her legs that is
momentarily relieved by moving them. Each of the following may be helpful in the treatment of this
condition except?

A) Two month trial of ferrous sulfate

B) Stretching exercises before bedtime

C) A glass of red wine before bedtime


D) Gabapentin

E) Oxycodone 5–10 before bedtime

Answer: C

23. A 35-year-old woman complains of tingling and numbness of the little finger on the left hand. On
examination, you corroborate hypesthesia of the 5th finger and also all of the ring finger, and you also
find left-side weakness of flexion of fingers and wrist. The patient is able to spread the fingers
adequately. Which of the following diagnoses is likely to account for these symptoms?

A) Ulnar nerve injury

B) Carpal tunnel syndrome

C) C6 nerve root compression

D) C7 nerve root compression

E) C8 nerve root compression 

Answer: E

24. Characteristic features of Mitral valve prolapse include

A) Early systolic murmur at the apex

B) Mid-systolic click

C) Valve Replacement gives better results than Valve repair

D) A poor prognosis 

E) Higher incidence in males

Answer: B
25. A known case of chronic obstructive pulmonary disease presents to A & E, distressed and cyanosed.
Arterial blood gases reveal a pH 7.2, PaO2 55 mmHg and PaCO2 60 mmHg. He is given high
concentration oxygen together with a salbutamol nebulizer. Intravenous hydrocortisone is also given.
The patient becomes even worse with poorer breathing effort although pulse oximetry showed SaO2 of
93%. What is the cause of patient deterioration?

A) Constriction of bronchioles in response to salbutamol nebulizer

B) High concentration oxygen administration

C) Pulmonary artery relaxation causing mismatch between perfusion and ventilation

D) Pulmonary vein relaxation causing mismatch between perfusion and ventilation

E) Reaction to IV hydrocortisone

Answer: B

26. Which of the following is a recognised treatment for complications of cystic fibrosis?

A) DNAase to assist in reinflating collapsed lung segments

B) Hypotonic saline drinks for hypernatraemic dehydration

C) Nebulised tobramycin for pseudomonas colonisation of the lower respiratory tract

D) Pancreatic transplant for diabetes mellitus

E) Rectal pull-through and anastamosis for rectal prolapse

Answer: C

27. A 45-year-old busy male attorney complains of headaches and generalized pruritus. He flew into an
eastern American city from Denver, Colorado, to attend a high-level legal conference. He smokes 1 pack
of cigarettes per day. He denies recent upper respiratory tract infection, foci of specific pain, and urinary
and GI symptoms. On examination, he manifests a blood pressure of 160/105, a temperature of 98.6 F, a
ruddy complexion, and splenomegaly and hepatomegaly. Blood gases are not immediately available. His
CBC showed the hemoglobin level to be 18 g/dL, the white blood cell count to be 14,000, and the
platelet count to be 7,00,000. Red blood cells are normochromic and normocytic. The BUN level is 18
mg/dL and the creatinine level is 1.1 mg/dL. Urinalysis is within normal limits with a specific gravity of
1.015. Which of the following is the diagnosis?

A) Polycythemia vera

B) Secondary polycythemia from altitude accommodation

C) Secondary polycythemia caused by chronic obstructive pulmonary disease

D) Spurious polycythemia from dehydration caused by air travel

E) Secondary polycythemia compensating for carboxyhemoglobin in a heavy smoker

Answer: A

28. A 26-year-old man has noted painless swelling in the left side of his neck. Furthermore, he has noted
afternoon chills and night sweats for several weeks. He has stopped smoking because inhaling tobacco
smoke makes him ill. Although he seldom drinks alcohol, during the past week, he took a glass of wine
and only then noted pain in the swelling about the neck. Examination is unremarkable except for the
neck,

which exhibits a firm irregular mass measuring approximately 3 cm 5 cm located in the left anterior
cervical region. The CBC and laboratory chemical battery are within normal limits. Which of the
following is the most likely significant cause of these symptoms?

A) Hodgkin disease

B) Sialoadenitis

C) Viral respiratory tract infection

D) Streptococcal pharyngitis

E) Carotid artery aneurysm

Answer: A

29. A 36-year-old male assembly line worker complains that 2 days ago, while hefting a bucket of metal
parts that slipped from his right hand while still grasping the other handle, experiences sudden pain in
the left side of his neck that radiates into the left lateral upper arm into the thumb, associated with
paresthesias in the thumb. Strength testing shows that his left wrist extension is 3/5. Which of the
following is the nerve root involved? 

A) C5 

B) C6 

C) C7 

D) C8 

E) T1

Answer: B

30. A 55-year-old man is scheduled for abdomen CT with contrast. He has type 2 diabetes; well
controlled with pioglitazone, metformin, and glimepride. On the day of the procedure he is to receive
nothing by mouth all morning except for medications until after the procedure. Which of the following
would be the most appropriate oral regimen on the day of the procedure?

A) Continue all medications

B) Hold metformin, and continue glimepride and pioglitazone

C) Hold glimepride and metformin, and continue pioglitazone

D) Hold glimepride and pioglitazone and decrease metformin dose by half

E) Hold metformin and pioglitazone, and continue glimepride

Answer: C

31. A 65-year-old woman, has smoked 50 cigarettes a day for 40 years. She has had increasing dyspnoea
for the several years, but no cough. A Chest X-ray shows increased lung size along with flattening of the
diaphragms, consistent with emphysema. Over the next several years she develops worsening
peripheral oedema. Her vital signs show T° 36.7 C, P 80, RR 15, and BP 120/80 mm Hg. Which of the
following cardiac findings is most likely to be present?

A) Mitral valve stenosis

B) Constrictive pericarditis
C) Right ventricular hypertrophy

D) Left ventricular aneurysmm

E) Non-bacterial thrombotic endocarditis

Answer: C

32. Which of the following is a typical feature of Farmer's lung?

A) basal crackles

B) Eosinophilia

C) Haemoptysis

D) Increased pCO2

E) Positive serum paraproteins

Answer: A

33. A 19-year-old female developed pleural effusions, ascites and ankle swelling. Her blood pressure was
112/76 mmHg.

Investigations revealed:

• serum alanine transferase 17 U/L (5 - 15)

• serum total bilirubin 17 umol/L (1 - 22)

• serum albumin 21 g/L (34 - 94)

• serum total cholesterol 9.8 mmol/L (<5.2)

What is the next most appropriate investigation?

A) Antinuclear antibody

B) Pregnancy test

C) Prothrombin time
D) Serum protein electrophoresis

E) Urinary protein estimation

Answer: E

34. The following are recognized features of Pancoast's tumour except:

A) ipsilateral Horner's syndrome

B) wasting of the dorsal interossei

C) pain in the arm radiating to the fourth and fifth fingers

D) erosion of the first rib

E) weakness of abduction at the shoulder

Answer: E

35.  Which of the following is a recognised cause of a phrenic nerve palsy?

A) Aortic aneurysm

B) Dermoid

C) Ganglioneuroma

D) Pericardial cyst

E) Sarcoidosis

Answer: A

36. A 58 year old woman falls and suffers a vertebral compression fracture. BMD confirms osteoporosis.
Prior history of hysterectomy/oopherectomy at age 40 and prior DVT. Pain controlled with Tylenol. Best
treatment:
A) Estrogen

B) Estrogen/progesterone

C) SERM

D) Bisphosphonate

E) Calcitonin

Answer: D

37. A 56-year-old man is admitted to the intensive care unit for acute respiratory distress syndrome
(ARDS). The patient was transferred from an outside hospital today after a 2-week hospitalization for
pneumonia. During that time, the patient's pulmonary status continued to deteriorate. One week ago he
was intubated and placed on mechanical ventilation and over the past week, his oxygenation has
worsened with a PaO2 of 66 on an inspired concentration of 100% oxygen. The patient has no other
medical history except for rheumatoid arthritis. On transfer to the ICU, the patient is intubated and
sedated on a mechanical ventilator. His chest radiograph shows patchy, bilateral, diffuse interstitial
infiltrates. The most important intervention that will most benefit this patient is to

A) keep the patient in a prone position during mechanical ventilation

B) keep tidal volumes greater than 15 cc/kg

C) limit peak inspiratory pressure to 45 cm H2O or less

D) limit PEEP levels to less than 10 cm H2O

E) limit tidal volumes to 6cc/kg

Answer: E

38. A 56-year-old woman presents with a 3-day history of fever, headache, fatigue, and myalgia, along
with nausea and vomiting. She reports cough with minimal hemoptysis but denies abdominal pain and
dysuria. About a week ago, she came back from a month-long missionary trip to a small village in Liberia.
She had received appropriate pre-travel vaccines including the yellow fever vaccine. She was also
compliant with her malaria prophylaxis as prescribed. On examination, she is tachypneic, tachycardic,
and in mild respiratory distress. She has generalized petechiae but no other rashes or lymphadenopathy.
What is the most likely infectious pathogen?
A) Plasmodium malariae

B) Salmonella typhi

C) Influenza virus

D) Ebola virus

E) Mycobacterium tuberculosis

Answer: D

39. In a 21 year old man with symptoms of chronic back pain, pain in his feet, particularly the great toe
and metatarsophalangeal joints, and bilateral sacroiliitis on plain films, the most likely diagnosis is:

A) Ankylosing spondylitis

B) Gout

C) Inflammatory bowel disease-related arthropathy

D) Reiter’s syndrome

E) Psoriatic arthritis

Answer: D

40. A 49-year-old woman comes to the office because of difficulty breathing, fevers reaching 40 C (104
F), and a productive cough with blood tinged sputum. She was recently diagnosed with ductal carcinoma
of the breast and underwent a radical mastectomy with four rounds of adjuvant chemotherapy with
vinblastine and doxorubicin. Her last infusion of chemotherapy was 5 days ago. A chest radiograph
shows focal infiltrates in both lungs. Laboratory studies show: You admit her to the hospital and start
her on vancomycin and ceftazidime. Over the next 3 days, her clinical status continues to worsen. Blood
cultures are negative. A bronchoscopy is performed and biopsy samples are obtained. The biopsy
specimen shows septated, branching hyphae that are locally invading tissue. The most appropriate
pharmacotherapy at this time is

A) amphotericin B, intravenously

B) fluconazole, intravenously

C) fluconazole, orally
D) itraconazole, intravenously

E) rifampin plus isoniazid, orally

Answer: A

41. An 82-year-old woman with a history of Hashimoto's thyroiditis is evaluated for a rapidly expanding
thyroid mass and progressive dysphagia and dyspnea. She has lost 2.2 kg over the preceding 6 weeks.
O/E: BP118/78 mm Hg, PR 86/min. Thyroid examination reveals a firm 5-cm left thyroid mass that moves
poorly with swallowing. Respiratory stridor is evident. Fine needle aspiration of the mass shows
numerous uniform lymphocytes. CT scan of the neck shows the thyroid to completely encircle the
trachea. 

Which of the following is the most likely diagnosis?

A) Medullary thyroid cancer 

B) Anaplastic thyroid cancer 

C) Thyroid hemorrhage 

D) Thyroid lymphoma 

E) Laryngeal cancer

Answer: D

42. Thiazide diuretics can contribute to all of the following metabolic effects EXCEPT:

A) Hypomagnesemia

B) Hypourecemia

C) Hypercalcemia

D)Hypercholesrolemia

E) Hyponatremia
Answer: B

43. A 55 year old women is referred for evaluation of hypercalcemia. Three years ago, she had a left
mastectomy for breast carcinoma, after which she underwent treatment with radiation and adjuvant
chemotherapy because 2 of 12 lymph nodes were found to be positive for carcinoma. She has been
clinically well since then, but on a routine postoperative check-up 3 months ago, she was found to have
a serum calcium level of 2.75 mmol/L (N:2.2-2.6 mmol/L). On repeat testing, the serum calcium level is
2.9 mmol/L

The best first diagnostic step to identify the cause of this patient’s hypercalcemia is:

A) serum alkaline phosphatase level

B) serum parathyroid hormone level

C) serum 1,25- dihydroxyvitamin D level

D) Serum 25 hydroxyvitamin D level

E) Bone Scan.

Answer: B

44. All of the following bedside maneuvers are useful to distinguish the murmur of mitral valve prolapse
from other valvular lesions EXCEPT ?

A) Isometric hangrip

B) Standing from a supine position

C) Carotid sinus massage

D) Valsalva maneuver

E) Squating from standing position

Answer: C
45. One of your 52-year-old female patients with an eGFR of 12 mL/min comes to your office with
complaints of fatigue and poor appetite due to metallic taste in the mouth for the last 2 years. She has
no chest pain or hyperkalemia. Her BP is 130/80 mmHg. Her serum [HCO3] is 20 mEq/L. During her
previous visit 3 months ago, you discussed about the choice of her future renal replacement therapies,
including transplantation. After prolonged discussion, she feels that PD is a reasonable choice in view of
her daily work in the school. Based on her clinical history, which one of the following choices is MOST
appropriate regarding the management of her symptoms and selection of dialysis modality is

CORRECT?

A) Admit to hospital and start HD with a central vein catheter (CVC), and then start PD

B) Tell her that she would receive in-center HD once acute HD treatment is over

C) Admit to hospital and place a PD catheter, and then train her for continuous ambulatory PD with
small volume exchanges initially

D) Change her diet and increase her NaHCO3 to 1350 mg every 8 h

E) Tell her that she may not need any renal replacement therapy (either HD or PD) until her eGFR falls to
7 mL/min

Answer: C

46. Each of the following combinations has the potential for significant pharmacological interaction and
drug toxicity except

A) Simvastatin and Erythromycin

B) Sildenafil and Nitroglycerin

C) Pravastatin and Ketconazole

D) Cyclosporine and St. johns wort

E) Digoxin and Verapamil

Answer: C
47. A 40 year old man with recurrent severe headaches. They appeared to occur episodically every 3-6
months, with headaches occuring daily for up to 8 weeks at a time. They often woke him from sleep at 2
am, with sever right peri-oricular pain and would typically last for 30-60 mins. He also described right
nasal congestion and lacrimation but no vomiting or photophobia. Neurological exam., MRI brain were
normal. What is the most appropriate prophylactic treatment:

A) Carbamazepine

B) Erogotamine

C) Propranolol

D) Sumatriptan

E) Verapamil

Answer: E

48. A 57-year-old female school cleaner is undergoing investigation for breathlessness. All the following
would be in keeping with a diagnosis of constrictive pericarditis except:

A) Ascites 

B) Elevated JVP with absent y descent 

C) Orthopnoea 

D) Peripheral oedema 

E) Previous cardiac surgery

Answer: B

49. A 45-year-old woman undergoes upper endoscopy for symptoms of dyspepsia. The endoscopic
examination is normal. Physical examination, routine laboratory studies, serologic tests forHelicobacter
pylori, and abdominal ultrasonography are also normal. The patient has not received any medications
for her symptoms. Which of the following is the most appropriate initial empiric therapy for this
patient? 

A) A proton pump inhibitor 


B) Alosetron 

C) Ondansetron 

D) Tegaserod 

E) Sumatriptan

Answer: A

50. By definition, an episode of pancreatitis is considered chronic pancreatitis if

A) It is a recurrent episode.

B) It occurs in the face of alcohol use.

C) There are radiographic findings of ductal irregularity and parenchymal fibrosis.

D) It is associated with steatorrhea.

E) A pseudocyst is present.

Answer: C

MULTIPLE CHOICE QUESTIONS-7

1. A 37-year-old woman with a history of intravenous drug use, hepatitis B, asthma, and acquired
immunodeficiency syndrome (AIDS) is admitted to the hospital because of fever, night sweats, and
malaise. Her last CD4 count was 1 month ago and measured 180/mm3. Vital signs are: temperature 38.5
C (101.3 F), blood pressure 145/76 mm Hg, and pulse 90/min. Physical examination is significant for a
soft diastolic murmur heard best at the lower left sternal border. Auscultation of the lungs reveals
diffuse rhonchi. The abdominal and neurologic exams are unremarkable. The next step in managing this
patient is 

A) analysis and culture of spinal fluid

B) a blood culture

C) a CT of the head
D) a urinalysis

E) an x-ray of the chest

Answer: E

2. An 18-year-old man presents with pain in the right side of the neck, veering to the right, and
numbness on the right side of the face and in the left arm and leg that came on acutely with lifting heavy
weights. His examination shows right hemiataxia, right miosis and ptosis, poor palatal elevation on the
right, and decreased sensation to pinprick of the right side of the face and the left arm and leg. To which
of the following structures does the lesion best localize?

A) Right lateral medulla

B) Left lateral medulla

C) Right cerebellum

D) Left midbrain

E) Central pons

Answer: A

3. A 78-year-old woman is admitted to the hospital because of a fever, productive cough, and a chest x-
ray demonstrating right lower lobe consolidation. Her past medical history is significant for seasonal
allergies. She has been taking estrogen/progesterone replacement since menopause 19 years ago and
occasional acetaminophen for headaches. The patient lives alone at her home and she does not drink
alcohol or smoke. Review of systems is significant for weakness attributed to "old age". On the day prior
to discharge, a repeat chest x-ray shows the pneumonia to be resolving. An incidental note is made of
severe osteoporosis involving all of the bones visualized on the film. Vital signs are temperature 38.8 C
(101.8 F), blood pressure 100/50 mm Hg, pulse 90/min, and respirations 10/min. Physical examination is
significant only for decreased breath sounds at the right lung base. The patient is neurologically intact
and wants to return home. Laboratory studies show a leukocyte count 15,000/mm3, hematocrit 28%,
and platelets 150,000 mm3. The next step in the management of this patient is to

A) discharge her and do a bone marrow biopsy as an outpatient

B) discharge her and send her for a bone scan as an outpatient


C) discharge her and order serum protein electrophoresis as an outpatient

D) do a bone marrow biopsy before discharge

E) order a bone scan and serum protein electrophoresis before discharge

Answer: C

4. Eosinophilia is frequently seen with fever of unknown origin from all but 1 of the following illnesses.
Which illness is not frequently associated with eosinophilia?

A) Systemic lupus erythematosus (SLE)

B) Drug fever

C) Tuberculosis

D) Myeloproliferative disease

E) Polyarteritis nodosa (PAN)

Answer: C

5. In a patient with a known Chiari I malformation, burning pain develops in both shoulders. On
examination, strength is normal in the limbs, but the sensation to pinprick is reduced in a capelike
distribution around the shoulders extending to the upper part of the arms. To what structure does the
lesion best localize?

A) Bilateral dorsal columns

B) Right lateral funiculus

C) Central spinal cord

D) Bilateral ventral funiculus

E) Bilateral ventral horns

Answer: C
6. A 33-year-old asymptomatic woman was referred because of abnormal ALT. History of migraine
headache for which she takes Sumatriptan. No risk factors or history of liver disease. Examination is
notable for obesity.AST 60, ALT 75, ALP , bilirubin , INR, iron studies and albumin are normal. What
would be the most appropriate next step ?

A) Observe and repeat ALT in 3 months 

B) Ultrasound abdomen

C) Check viral markers 

D) Liver biopsy

E) Stop Sumatriptan

Answer: A

7. A 63-year-old woman with 4 hour history of abdominal pain , fever and nausea. On examination ,
patient has fever, jaundice and mild epigastric tenderness . TC 18000, with shift to left, bilirubin
elevated, ALP 150,(normal upto 125) AST 745 IU, ALT 650 IU, USG multiple small stones in gall bladder,
no bile duct dilatation , normal pancreas. Started on antibiotics, but following day still has fever. Repeat
labs bilirubin elevated, AST 284 IU, ALT 200 IU, WBC 25000, blood culture positive for E.Coli. which of
the following would you advice next ?

A) Doppler of hepatic vessels 

B) Lab cholecystectomy

C) MRCP

D) Endoscopic ultrasound

E) ERCP

Answer: E

8. A 18-year-old woman presents with acute onset of jaundice and somnolence. On examination ,
jaundiced , sleepy but arousable. Labs INR 1.6, AST 240, ALT 210, total bilirubin increased , mostly in
direct, hemoglobin 9.4, ceruloplasmin 8ng/L(normal more than 22), 24 hr urine copper 563mcg/L
(normal less than 60).which of the following would you advise now ?

A) Trientine 

B) Penicillamine

C) Urgent liver transplantation 

D) Intracranial pressure monitoring 

E) Liver biopsy

Answer: C

9. Warfarin is administered to a 56-year-old man following placement of a prosthetic cardiac valve. The
warfarin dosage is adjusted to maintain an INR of 2.5. Subsequently, trimethoprim-sulfamethoxazole
therapy is begun for a recurring urinary tract infection. In addition to monitoring prothrombin time,
which of the following actions should the physician take to maintain adequate anticoagulation? 

A) Begin therapy with vitamin K 

B) Increase the dosage of warfarin 

C) Make no alterations in the dosage of warfarin 

D) Decrease the dosage of warfarin 

E) Stop the warfarin and change to low dose aspirin

Answer: D

10. You are called emergently to the medical floor where a 66-year-old man was found to be minimally
responsive. His past medical history is unclear but his arm band lists allergies to penicillin and sulfa
medications. On arrival, chest compressions are being performed and 2 operators are mask ventilating
the patient. Evaluation with an electrocardiogram reveals sinus tachycardia and the diagnosis of
pulseless electrical activity is made. Volume is infused and compressions are continued. The patient
remains apneic, so mask ventilation continues. During masking, the patient appears to regurgitate large
volumes of gastric contents. The most appropriate immediate next step in the management of this
patient is to
A) cease mask ventilation and suction the mouth

B) continue masking the patient

C) insert a nasogastric tube

D) intubate the trachea and suction the airway

E) intubate the trachea and ventilate

Answer: D

11. Which statement about evaluating a patient with fever of unknown origin (FUO) is true?

A) Bone marrow cultures have a high yield when the fever is greater than 39 ° C and other testing has
been unrevealing.

B) A temporal artery biopsy is a reasonable test to perform next for a 75-year-old man who has FUO and
no localizing complaints, an erythrocyte sedimentation rate greater than 100 mm/h, mild anemia,
normal blood chemistry results, and negative blood cultures at 3 days and who has not had any other
evaluation.

C) Lumbar puncture should be done on every patient with FUO.

D) When there are no clues to the underlying diagnosis, extensive serologic testing for unusual
infections is often helpful.

E) Liver biopsy is unlikely to be helpful in a patient with FUO and miliary tuberculosis

Answer: B

12. A 31-year-old man describes a burning sensation in the substernal area after eating chocolates,
caffeine, or alcohol. The symptoms are exacerbated at night, and he has woken on several occasions
from sleep because of coughing. He has tried over-the-counter antacids and H2 receptor antagonists
with little relief. He often takes antacids before dinner. Which of the following is the most likely
explanation for his symptoms of coughing?

A) Acid-induced bronchoconstriction 

B) Aspiration of antacids taken before bedtime 


C) Aspiration of solid food ingested with dinner 

D) H2 receptor antagonist-induced bronchoconstriction 

E) Tracheoesophageal fistula

Answer: A

13. A 24-year-old lady at 10 weeks of gestation during her first pregnancy. She has had 2 days of nausea
and mild fever. She vomited once this morning. On examination jaundiced, with tender , mild
hepatomegaly. Labs AST 56) IU, ALT 1150 IU, bilirubin increased, platelets 140. most likely diagnosis 

A) Hyperemesis gravidarum

B) Acute viral hepatitis 

C) Acute cholecystitis

D) Intra hepatic cholestasis of pregnancy

E) Fatty liver of pregnancy

Answer: B

14. A 36-year-old male with alcoholic hepatitis. Which clinical or lab parameter is least predictive of
patient survival ?

A) Bilirubin 

B) Prothrombin time 

C) AST/ALT ratio

D) Encephalopathy

E) Creatinine

Answer: C
15. A 64-year-old male executive with remote history of heavy alcohol use is noted to have cirrhosis on
liver biopsy. There are no varices detected on upper GI Endoscopy. When would you recommend that
the patient return for a repeat upper Endoscopy to screen for esophageal varices ?

A) 6 months 

B) 12 months 

C) 18 months 

D) 24 months 

E) Never

Answer: D

16. Which of the following statements about pulmonary function during pregnancy is true?

A) Total lung capacity is reduced.

B) Functional residual capacity is reduced.

C) FEV1 /FVC ratio is reduced.

D) Lung compliance is reduced.

E) The single breath diffusing capacity for carbon monoxide is reduced

Answer: B

17. Which one of the following is the commonest cause of death in SLE?

A) Infection

B) Cardiovascular disease

C) Lupus nephritis

D) Cerebral lupus

E) Thromboembolism
Answer: A

18. Each of the following is a solid indication for neuroimaging in a patient with headache EXCEPT: 

A) Onset of headaches over the age of 50 years 

B) Seizures associated 

C) Prolonged aura 

D) Nausea and vomiting 

E) Headache worsening with movement

Answer: D

19. A lady is newly diagnosed with osteoporosis. She has never been on treatment. Which one of the
following is the first-line treatment?

A) Alendronic acid

B) Etidronate

C) Risedronate

D)  Strontium ranelate

E) Zoledronate

Answer: A

20. A 48 year-old lady is referred to the rheumatology outpatient clinic with a 4-month history of
fatigue, aches and weakness of her thighs.She has a past medical history of hypercholesterolaemia and
is on simvastatin. On examination she has normal muscle bulk. Active hip flexion was
reduced. Neurological examination was otherwise normal.

Investigations:
Hb 98 g/L (115–165)

WCC 12.0 × 109 /L (4.0–11.0)

Platelets 400 × 109/L (150–400)

Plasma viscosity 2.0 (1.5–1.72)

Creatine kinase 15000 IU/L

The diagnosis is likely to be which one of the following?

A) Polymyalgia rheumatica

B) Statin-induced myopathy

C) Polymyositis

D) Dermatomyositis

E) Hip osteoarthritis

Answer: C

21. A 78-year-old man has collapsed several hours ago and been found on the floor by the paramedics.
He was found to be hypothermic at 32°C. His creatinine kinase is 10,000 IU. His serum potassium is
5mmol/L, urea is 20mmol/L and creatinine is 300μmol/L. The most important form of initial treatment
should be:

A) Intravenous infusion of warmed crystalloid.

B) Active warming.

C) Intravenous administration of mannitol 0.25-0.5g/kg.

D) Intravenous furosemide.

E) Urgent haemodialysis

Answer: A
22. A 25-year-old woman with a history of epilepsy presents to the emergency room with impaired
attention and unsteadiness of gait. Her phenytoin level is 37. She has white blood cells in her urine and
has a mildly elevated TSH. Examination of the eyes would be most likely to show which of the following?

A) Weakness of abduction of the left eye

B) Lateral beating movements of the eyes

C) Impaired convergence

D) Papilledema

E) Impaired upward gaze

Answer: B

23. A 35-year-old female is referred for evaluation of positive antinuclear antibodies (ANA). She is
asymptomatic. Her lab work reveals ANA 1:320 homogeneous pattern; extractable nuclear antigens are
negative. Hematologic and renal function values are normal and urinalysis is without sediment. Her
examination is unremarkable apart from a smooth nontender goiter. The past medical history is
significant for Hashimoto’s thyroiditis. No rash, synovitis, or serositis is observed. What further
treatment or investigations are warranted?

A) Start prednisolone 40 mg daily

B) Start prednisolone 20 mg daily

C) Repeat ANA test

D) Start hydroxychloroquine

E) No further intervention is warranted

Answer: E

24. A 41-year-old man is diagnosed with iron deficiency anemia and is found to have heme-positive
stools. Colonoscopy reveals a large ulcerated tumor in his transverse colon. He also has two smaller
polyps in his ascending colon. Pathologic examination of the tumor biopsy reveals adenocarcinoma,
while biopsies of the polyps confirm that these are adenomas. His sister has been diagnosed
with uterine cancer, and two cousins have died of colon cancer. All of the following are true statements
about this case except

A) Referral for genetic counseling is indicated

B) He is at increased risk for other epithelial-derived tumors

C) He likely has familial adenomatous polyposis (FAP), with a germ line mutation in the APC gene

D) His condition is often associated with a defect in DNA mismatch repair

Answer: C

25. A 60-year-old man presents with abdominal pain and a cupful of haematemesis. On examination he
is noted to have ascites, hepatomegaly and a very enlarged spleen extending to the right iliac fossa. His
initial blood tests reveal a leukoerythroblastic picture with a haemoglobin of 8, white cell count (WCC) of
3 and platelets of 120. A diagnosis of myelofibrosis is made. What is most likely to be seen on the
peripheral blood smear?

A) Schistocytosis

B) Sickle cells

C) Spherocytes

D) Dacrocytes

E) Target cells

Answer: D

26. All the following conditions are associated with high output heart failure EXCEPT

A) Iron overload

B) Hyperthyroidism

C) Systemic arteriovenous fistula

D) Thiamine deficiency
E) Paget disease

Answer: A

27. Primary prophylaxis to prevent acute infection in susceptible patients is recommended for all the
following HIV Infected patients EXCEPT

A) Pneumocystis jiroveci (formerly carinii) pneumonia.

B) Disseminated Mycobacterium avium complex.

C) Mucocutaneous candidiasis.

D) Malaria for persons traveling to areas where malaria is endemic.

E) Toxoplasma gondii encephalitis

Answer: C

28. A 30-year-old male with Alports and gradually worsening renal dysfunction has reached ESRF and is
on hemodialysis. His family is being evaluated for donation for renal transplant. His 57 year old mother
and one aunt have microscopic hematuria but normal renal functions. Two of his uncles also have ESRD
and are on dialysis. He has a 40 brother with normal urine analysis, a 38 year old sister with normal
urine analysis and a 25 year old sister with microscopic hematuria, all with normal renal function.

Which of the following can most safely donate kidney for this patient?

A) Brother as he has extremely low probability of having Alports.

B) Sister with normal urine analysis, as she has no probability of having Alports.

C) Sister with microscopic hematuria as she is younger.

D) Mother 

E) None of family members should be taken as donor

Answer: B
29. A 48-year old female with rheumatoid arthritis presents to the emergency department with 2-week
pain and tightness behind the left knee. Examination reveals cystic swelling over the left popliteal fossa.
Which of the following is the most appropriate next action?

A) Arthrogram of the left knee

B) Synovial biopsy of left knee

C) Ultrasound study of left knee and popliteal fossa

D) Venogram of left lower limb

E) None of the above

Answer: C

30. A 37-year-old woman is admitted to accident and emergency with severe facial burns. Despite
prompt management, she develops acute respiratory distress syndrome (ARDS). Which of the following
is not associated with the diagnostic criteria for ARDS?

A) Bilateral infiltrates on chest x-ray

B) Acute onset

C) Pulmonary capillary wedge pressure >19

D) Refractory hypoxemia (PaO2:FiO2 <200)

E) Lack of clinical congestive heart failure

Answer: C

31.  A 55-year-old man with chronic hepatitis C is being considered for liver transplantation. The patient
has cirrhosis that was documented by liver biopsy 10 years ago. For the past 3 months, he has had
ascites and edema, which are poorly controlled with diuretics. Lactulose was recently begun because of
confusion. Which of the following combinations of laboratory studies will be most helpful in estimating
his survival over the next 6 months? 

A) Serum total bilirubin and INR 


B) Serum aspartate aminotransferase and gamma globulin 

C) Serum alanine aminotransferase and hepatitis C RNA (HCV RNA) 

D) Serum alkaline phosphatase and ammonia 

E) Serum albumin and ?-glutamyltransferase

Answer: A

32.  A 67-year-old African American man with bronchogenic carcinoma returns to the office for follow-
up of confusion and lethargy that have been gradually increasing during the past 3 weeks. CT scan of the
head 4 weeks ago showed no metastases. Current medications include inhaled bronchodilator
medication and oxycodone for pain. The patient is 185 cm (6 ft 1 in) tall and weighs 61 kg (135 lb); BMI is
18 kg/m2. Vital signs are normal. Physical examination shows generalized muscle wasting. Auscultation
of the lungs discloses scattered rhonchi in all fields and expiratory wheezes. Mini- Mental State
Examination score is 24/30. Results of laboratory studies are shown:

Serum Na+ 125 mEq/L

K+ 3.2 mEq/L

Cl− 100 mEq/L

HCO3 − 25 mEq/L

Blood

Hematocrit 32.2%

Hemoglobin 11.2 g/dL

Which of the following is the most appropriate study to order at this time?

A) 24-Hour urine collection for creatinine clearance

B) Determination of AM serum cortisol concentration

C) Determination of serum iron concentration

D) Determination of serum magnesium concentration

E) Determination of urine sodium concentration


Answer: B

33. Which of the following β-adrenergic antagonists is a nonselective β1 and β2 blocker?

A) Atenolol

B) Betaxolol

C) Esmolol

D) Metoprolol

E) Nadolol

Answer: E

34. Signs and symptoms of opioid withdrawal include all of the following EXCEPT

A) Increased blood pressure (BP) and heart rate

B) Seizures

C) Abdominal cramps

D) Jerking of the legs

E) Hyperthermia

Answer: B

35. A 45-year-old man is evaluated for heartburn with water brash, usually occurring at bedtime. He
notes worsening of symptoms when lifting weights, but he is asymptomatic during his routine 5-km run.
He has not had dysphagia, bleeding, vomiting, or weight loss. He has been taking nonprescription
omeprazole once daily for the past two weeks. BMI is 32. Vital signs and physical examination are
normal. Complete blood count, serum electrolytes, and electrocardiogram are normal.

Which of the following should you recommend?

A) An increase of omeprazole dosage to twice daily


B) Exercise stress test

C) Computed tomography of the abdomen

D) Ultrasonography of the abdomen

E) Esophagogastroduodenoscopy

Answer: A

36. A pregnant woman is being followed for gestational diabetes. She walks 30 minutes per day and is
following a diabetic diet. She is not gaining weight. Fasting sugars are 5 - 6.5; postprandial, 7 - 8.5. She
now has ketonuria in the mornings. What should you advise?

A) Increase caloric intake, plus begin insulin NPH 10u QHS

B) Increase caloric intake, plus begin insulin lispro ac meals

C) Increase caloric intake and exercise before breakfast 

D) Increase calorie intake and postprandial exercise

E) Increase calorie intake, plus an oral agent

Answer: B

37. A 60-year-old man is admitted to the hospital for management of acute pancreatitis. Results of
laboratory studies are shown: Serum Blood Amylase 1000 U/L Hematocrit 42%; Calcium 8.4 mg/dL WBC
14,000/mm3; Urea nitrogen 5 mg/dL. Results of serum liver chemistry profile are within the reference
ranges. After 48 hours of fluid therapy and observation, a poor prognosis would be indicated by which of
the following laboratory study results?

A) Serum alanine aminotransferase (ALT) concentration of 106 U/L

B) Serum amylase concentration of 2000 U/L

C) Serum bilirubin concentration of 4.2 mg/dL

D) Serum calcium concentration of 6.6 mg/dL

E) Serum glucose concentration of 200 mg/dL


Answer: D

38. Linezolid has the following characteristics as compared to vancomycin:

A) Decreased nephrotoxicity at higher doses

B) Increased intrapulmonary penetration

C) Increased incidence of thrombocytopenia

D) All of the above

E) None of the above

Answer: D

39. A 19-year-old female developed pleural effusions, ascites and ankle swelling. Her blood pressure was
112/76 mmHg.

Investigations revealed:

• serum alanine transferase 17 U/L (5 - 15)

• serum total bilirubin 17 umol/L (1 - 22)

• serum albumin 21 g/L (34 - 94)

• serum total cholesterol 9.8 mmol/L (<5.2)

What is the next most appropriate investigation?

A) Antinuclear antibody

B) Pregnancy test

C) Prothrombin time

D) Serum protein electrophoresis

E) Urinary protein estimation


Answer: E

40. The following are recognized features of Pancoast's tumour except:

A) ipsilateral Horner's syndrome

B) wasting of the dorsal interossei

C) pain in the arm radiating to the fourth and fifth fingers

D) erosion of the first rib

E) weakness of abduction at the shoulder

Answe: E

41. A 43-year-old woman has the following laboratory test results: serum bilirubin, total 4.6 mg/dL and
direct 0.3 mg/dL; serum alkaline phosphatase, 108 U/L; serum aminotransaminase, aspartate
aminotransferase (AST) 18 U/L and alanine aminotransferase (ALT) 22 U/L. What is the most likely

diagnosis?

A) Chronic hepatitis C virus infection

B) Dubin-Johnson syndrome

C) Hemolytic anemia

D) Primary biliary cirrhosis

E) Primary sclerosing cholangitis

Answer: C

42. A 52 year old woman referred because of palpitation, weight loss and increased sweating. 2 years
back she was admitted to CCU because of Acute MI complicated by ventricular tachycardia and was
discharged on amiodarone. Her TFT revealed TSH 0.01, FT4 > 100. RAIU showed increased uptake and
U/S thyroid showed increased vascularity. What is the next immediate step:
A) Stop amiodarone

B) Start carbimazole

C) Start steroids

D) Give appointment with cardiologist to change amiodarone

E) Observe and reassure the patient

Answer: B

43. You are asked to see a woman recently admitted to the ICU with hypotension. She is unable to give a
history, and no family members or friends are available. Witnesses report that she was sitting at the
airport waiting for a connecting flight when she passed out. The person sitting next to her noticed that
she became quite agitated when it was announced that the flight would be delayed for 6 hours, just
before she passed out. There is no evidence of a pulmonary embolism. During your examination, you
note that she is very thin, lacks axillary hair, has sparse pubic hair, and has extra pigmentation on her
gums and buccal mucosa. The blood pressure was 40/palp with a heart rate of 130 when she first
arrived, but is now 100/64 with a heart rate of 78 with saline running. Her temperature is 98.2° F; the
cardiac rhythm is sinus; the routine chemistry tests show a hyperkalemic metabolic acidosis; and the
renal status is compatible with prerenal azotemia.

Which of the following should you do now?

A) Begin hydrocortisone 100 mg IV every 8 hours and order an ACTH stimulation test.

B) Measure cortisol and FT4 levels before beginning thyroxine and methylprednisolone.

C) Begin hydrocortisone 100 mg IV every 8 hours.

D) Begin dexamethasone 4 mg IV and order an ACTH stimulation test.

E) Measure ACTH and TSH before beginning prednisone 60 mg and thyroxine 100 μg every day.

Answer: D

44. A 45-year-old woman is hospitalized for a cholecystectomy. She appears to be anxious and worried
about the surgery. Which type or types of information provided to the patient will be most effective in
reducing stress?
A) Sensory information

B) Procedural information

C) Coping information

D) Sensory and procedural information

E) Sensory and coping information

Answer: E

45. A 60-year-old woman was hospitalized with a severe respiratory infection for several weeks.
Afterward, she displayed symptoms of myalgia and weakness of the lower limbs. In addition, she also
showed loss of muscle tone and some flaccidity with loss of tendon reflexes. Examination also revealed a
weakness of facial muscles. This constellation of symptoms

progressed for approximately 2 weeks and persisted for more than a year, at which time, recovery took
place at a slow rate. There was also some demyelination coupled with lymphatic inflammation at the
site of demyelination. The most likely cause of this patient’s condition is

A) Myasthenia gravis

B) Muscular dystrophy (MD)

C) Multiple sclerosis (MS)

D) Guillain-Barré syndrome

E) Lumbar disk prolapse

Answer: D

46. A 40-year-old patient with a recent viral infection presents with a significantly tender gland, low
radioiodine uptake, and signs and symptoms of thyrotoxicosis. This presentation is most likely

A) Graves’ disease

B) Subacute thyroiditis

C) Toxic multinodular goiter


D) Hashimoto’s thyroiditis

E) Toxic adenoma

Answer: B

47. A 30-year-old male presents with multiple soft, raised, beefy-red superficial ulcers in his left groin.
Physical examination reveals several enlarged left inguinal lymph nodes. A histologic section from an
enlarged lymph node that is stained with a silver stain reveals characteristic Donovan bodies within
macrophages. What is the most likely diagnosis?

A) Chancroid

B) Gonorrhea

C) Granuloma inguinale

D) Lymphogranuloma venereum

E) Syphilis

48. A 55-year-old woman comes to the emergency department because of abdominal pain. She had just
finished eating a steak dinner with her family when she suddenly experienced sharp, crampy pain in the
upper right and middle of her abdomen. The pain has lasted for the past 3 hours and she is starting to
feel nauseous. On physical examination, she is obese and in obvious discomfort. Her temperature is 38.8
C (101.8 F), blood pressure is 140/87 mm Hg, pulse is 90/min, and respirations are 16/min. Abdominal
examination is significant for focal tenderness and guarding in her right upper quadrant. She is
particularly tender when you palpate her right upper quadrant as she takes in a deep breath. The most
appropriate next step in the evaluation of her abdominal pain is

A) an abdominal x-ray

B) a CT of the abdomen

C) an endoscopic retrograde cholangiopancreatography (ERCP)

D) serum liver function tests including bilirubin

E) an ultrasonography of the abdomen


Answer: E

49. A 32-year-old female smoker presents with acute severe asthma. The SaO2 are 80% on 15 L of
oxygen and the pO2 is 8.2kPa (10.5-13). There is widespread expiratory wheeze throughout the chest.
She is given IV hydrocortisone, 100% oxygen and 5 mg of nebulised salbutamol.

What is the next step in your management

A) IV augmentin 

B) IV magnesium 

C) IV potassium 

D) IV theophylline 

E) Oral prednisolone

Answer: B

50. A previously healthy16 year old girl brought by her parents to ER because of repeated vomiting,
lethargy, breathlessness and confusion of 1 day duration. She developed 2 attacks of tonic clonic
seizures up on arrival to ER. Physical examination was remarkable for confusion, tachypnea and temp. of
38.5 C. CT brain, CXR and CSF analysis were normal. Blood biochemistry revealed Na 132 mmol/L, K 3.0
mmol/l, BUN 12 mmol/l, Creatinine 74 mmol/l and HCO3 12 mmol/L. ABG showed ph 7.5 , PaCO2 16
mmHg, PaO2 120 mmHg, HCO3 13 mmol/L. What is the best management option:

A) start dexamethasone, ceftriaxone and vancomycin

B) Start dexametahsone, ceftriaxone, vancomycin and oseltamivir

C) Start N-acetylcysteine infusion

D) Urinary alkalinization followed by hemodialysis 

E) Treat with septrin and request HIV testing

Answer: D
 

Answer: B

50. A previously healthy16 year old girl brought by her parents to ER because of repeated vomiting,
lethargy, breathlessness and confusion of 1 day duration. She developed 2 attacks of tonic clonic
seizures up on arrival to ER. Physical examination was remarkable for confusion, tachypnea and temp. of
38.5 C. CT brain, CXR and CSF analysis were normal. Blood biochemistry revealed Na 132 mmol/L, K 3.0
mmol/l, BUN 12 mmol/l, Creatinine 74 mmol/l and HCO3 12 mmol/L. ABG showed ph 7.5 , PaCO2 16
mmHg, PaO2 120 mmHg, HCO3 13 mmol/L. What is the best management option:

A) start dexamethasone, ceftriaxone and vancomycin

B) Start dexametahsone, ceftriaxone, vancomycin and oseltamivir

C) Start N-acetylcysteine infusion

D) Urinary alkalinization followed by hemodialysis 

E) Treat with septrin and request HIV testing

Answer: D

MULTIPLE CHOICE QUESTIONS-8

1. A 24-year-old Asian man presents to his family physician with one episode of painless gross
hematuria. He is otherwise healthy with no significant family history. On physical examination, his blood
pressure is normal and there are no other abnormalities. Laboratory examination shows a
normal metabolic profile, hemogram, liver enzymes, and renal function tests, as well as a negative urine
culture. Serologic tests including complement evels, antinuclear antibody, antineutrophil cytoplasmic
antibody, and a hepatitis panel are all negative. Urine examinations show 3+ blood, 3+ protein, and
many RBCs and RBC casts on microscopic examination. His renal ultrasound is normal. A repeat physical
examination and laboratory tests after 1 week show 1+ heme, 5 to 10 RBCs, and trace urinary protein,
which quantitates to 150 mg/day. Your recommendation at this point should be:

A) Continue observation.
B) Renal biopsy.

C) Empiric therapy with alternate day steroids.

D) Genetic testing for patient and his family.

E) Urology referral

Answer: A

2. A 69-year-old man presents with confusion. His carers state that over the last month he has become
increasingly lethargic, irritable and confused. Despite maintaining a good appetite, he has lost 10 kg in
the last month. Blood results are as follows:

Sodium 125 mmol/L

Potassium 4 mmol/L

Urea 3

Glucose (fasting) 6 mmol/L

Urine osmolality 343 mmol/L

The most likely diagnosis is:

A) Hypothyroidism

B) Dilutional hyponatraemia

C) Addison’s disease

D) Acute tubulointerstitial nephritis

E) Syndrome of inappropriate anti-diuretic hormone (SIADH)

Answer: E

3. You see a 19-year-old Caucasian man in your clinic who presents with a history of transient jaundice.
On direct questioning, you ascertain that the jaundice is noticeable after periods of increased physical
activity and subsides after a few days. The patient has no other symptoms and physical examination is
unremarkable. Full blood count is normal (with a normal reticulocyte count) and liver function tests
reveal a bilirubin of 37 μmol/L. The most appropriate management is:

A) Reassure and discharge

B) Start on a course of oral steroids

C) Request abdominal ultrasound

D) Request MRCP

E) Refer to Hematology

Answer: A

4. A 60-year-old man presented with epigastric and right hypochondrial pain associated with nausea,
vomiting and fever. He consumed alcohol almost daily for the last 30 years. He recalled that a surgeon
had previously suggested a gall bladder surgery but he did refused to have it. On exam: temp. 38°C, B.P.
120/70, pulse 100/min. Sclerae were yellow. He had eight spider nevi on his upper body. Chest and
heart exam were normal. Abdomen was tender in the right upper quadrant but no mass was palpable.
There was no detectable ascites and rectal examination was unremarkable. Hemoglobin 14.2, WBC
count 17,000, Platelets 110,000, Serum albumin 30 g/L, total bilirubin 70 (1-22), ALT 70 (< 40), alkaline
phosphatase 500 U/L (45-105), amylase 100 U/L (60-180), INR 1.5 What is the likely diagnosis?

A) Cholangitis

B) Acute on chronic pancreatitis

C) Alcoholic hepatitis

D) Acute cholecystitis

E) Empyema of the gall bladder

Answer: A

5. A 45-year-old man collapses at home and is brought to accident and emergency. He has a fever at
39.5°C and blood pressure is 90/60 mmHg, although he is in a lucid state. Bruises can be seen on his skin
which he remembers being present before he fell. Blood tests show the patient to have a normocytic
anaemia with a low platelet count and increased fibrin split products. The most likely diagnosis is:
A) Warm autoimmune haemolytic anaemia

B) Cold autoimmune haemolytic anaemia

C) Paroxysmal nocturnal haemoglobinuria

D) Disseminated intravascular coagulation

E) Thalassaemia minor

Answer: D

6. A 40-year-old man with cryptogenic liver cirrhosis presents with acute onset of confusion. His wife
reports that over the last few days his abdomen has become increasingly distended, however she is
more concerned about him being disoriented to time and place. He takes spironolactone 100 mg daily,
furosemide 40 mg bid and lactulose 10 mg bid. Exam: He is confused to time and place and is afebrile.
Icteric sclerae , spider nevi and asterixis . There is tense ascites with distended abdominal veins is also
noted. WBC count 10,200, Hemoglobin 10.5, Platelets 35,000, INR 1.7, Serum sodium 128, Serum
creatinine 90, ALT 60 What should be the next step?

A) Abdominal paracentesis 

B) CT-scan of the brain

C) Start IV cefotaxime 

D) Start oral neomycin 

E) Increase dose of diuretics

Answer: A

7. A 62-year-old woman presents to accident and emergency with a 1-day history of sudden onset back
pain and difficulty walking. She has not opened her bowels or passed urine for the previous day. She has
a past medical history of breast cancer, diagnosed two years earlier and staged as T2N1M0 disease with
oestrogen receptor positive status. She has been treated for her cancer with a wide local excision and
axillary node clearance, followed by radiotherapy, chemotherapy and tamoxifen. On examination, there
is reduced tone in the lower limbs. Power is diminished throughout the lower limbs, but especially on
hip flexion. There is reduced sensation below the L1 dermatome. What is the most appropriate
diagnostic investigation?
A) A full set of bloods, including bone profile

B) Computed tomography (CT) thorax, abdomen and pelvis

C) Magnetic resonance imaging (MRI) spine

D) Bone scan

E) Positron emission tomography (PET) CT

Answer: C

8. A 35-year-old woman comes to your clinic complaining of shortness of breath. It is immediately
apparent that she has a bluish tinge of her face, trunk, extremities, and mucus membranes. Which of the
following is most likely?

A) Atrial septal defect

B) Myocarditis

C) Raynaud’s phenomenon

D) Vasospasm due to cold temperature

E) Connective tissue disease

Answer: A

9. A 50-year-old woman, who has received a recent diagnosis of rheumatoid arthritis, presents to her GP
with ongoing pain and stiffness in her hands and feet. Which joints are usually spared at onset of
rheumatoid arthritis?

A) Proximal interphalangeal joints

B) Distal interphalangeal joints

C) Metacarpophalangeal joints

D) Wrists

E) Metatarsophalangeal joints
Answer: B

10. A 45-year-old woman with unexpected weight loss, loss of appetite and shortness of breath presents
to you in clinic. On examination, there is reduced air entry and dullness to percussion in the right lung. A
pleural tap is performed and the aspirate samples sent for analysis. You are told that the results reveal a
protein content of >30 g/L. From the list below, select the most likely diagnosis:

A) Bronchogenic carcinoma

B) Congestive cardiac failure

C) Liver cirrhosis

D) Nephrotic syndrome

E) Meig’s syndrome

Answer: A

11. A 79-year-old woman presents to her GP with pain in the left knee. This is particularly bad in the
evenings and is stopping her from sleeping. The GP explains that her discomfort is most likely due to
osteoarthritis and arranges for her to have

an x-ray of the knee. Which of the following descriptions are most likely to describe the x-ray?

A) Reduced joint space, subchondral sclerosis, bone cysts and osteophytes

B) Increased joint space, subchondral sclerosis, bone cysts and osteophytes

C) Reduced joint space, soft tissue swelling and peri-articular osteopenia

D) Increased joint space, soft tissue swelling and peri-articular osteopenia

E) Normal x-ray

Answer: A
12. A 52-year-old man is referred for newly diagnosed diabetes.  He reports a two month history of
polyuria and polydipsia. He has no past history of note and was not taking any regular medications,
although he reports recent weak erections. On examination, liver edge is palpable 2 cm below the right
costal margin and his testes are found to be small. HbA1c 10.8, Hemoglobin15.1 g/dL, Platelets 341
x109/L, Serum iron 50 µmol/L (12-30), Transferrin saturation 85% (<50%). Which investigation is most
likely to assist in making a diagnosis?

A) Genetic analysis for C282Y and H63D mutations 

B) Red cell transketolase

C) Serum ferritin

D) Total iron binding capacity (TIBC)

E) Ultrasound of the abdomen

Answer: A

13. A 49-year-old man complains of sudden onset, painless unilateral visual loss lasting about a minute.
He describes ‘a black curtain coming down’. His blood pressure is 158/90, heart rate 73 bpm. There is an
audible bruit on auscultation of his neck. His past medical history is insignificant other than deep vein
thrombosis of his right leg ten years ago. The most likely diagnosis is:

A) Retinal vein thrombosis

B) Retinal artery occlusion

C) Amaurosis fugax

D) Optic neuritis

E) Acute angle glaucoma

Answer: C

14. A 69-year-old man present with a 2-week history of abdominal pain which has worsened over the
last few days. On examination, the patient is jaundiced and the abdomen is distended with tenderness
in the epigastric region. In addition, there is a smooth hepatomegaly and shifting dullness. Which of the
following is a cause of hepatomegaly?
A) Iron deficiency anaemia

B) Budd–Chiari syndrome

C) Ulcerative colitis

D) Crohn’s disease

E) Left-sided heart failure

Answer: B

15. A 79-year-old woman is admitted to the coronary care unit (CCU) with unstable angina. She is
started on appropriate medication to reduce her cardiac risk. She is hypertensive, fasting glucose is
normal and cholesterol is 5.2. She is found to be in atrial fibrillation. What is the most appropriate
treatment?

A) Aspirin and clopidogrel

B) Digoxin

C) Cardioversion

D) Aspirin alone

E) Warfarin

Answer: E

16. A 53-year-old healthy man was found to have a fasting plasma glucose of 7.3 mmol. Further testing
showed HbA1c of 6.2. What should be done next?

A) Repeat HbA1c

B) Repeat fasting glucose

C) Advise home glucose monitoring 

D) lifestyle changes, repeat all tests in 3 months

E) Measure 2-hour postprandial glucose


Answer: B

17. A 66-year-old woman presents to accident and emergency with a 2-day history of shortness of
breath. The patient notes becoming progressively short of breath as well as a sharp pain in the right side
of the chest which is most painful when taking a deep breath. The patient also complains of mild pain in
the right leg, though there is nothing significant on full cardiovascular and respiratory examination.
Heart rate is 96 and respiratory rate is 12. The patient denies any weight loss or long haul flights but
mentions undergoing a nasal polypectomy 3 weeks ago. The most likely diagnosis is:

A) Muscular strain

B) Heart failure

C) Pneumothorax

D) Angina

E) Pulmonary embolism

Answer: E

18. A 78-year-old woman is admitted with heart failure. The underlying cause is determined to be aortic
stenosis. Which sign is most likely to be present?

A) Pleural effusion on chest x-ray

B) Raised jugular venous pressure (JVP)

C) Bilateral pedal oedema

D) Bibasal crepitations

E) Atrial fibrillation

Answer: D
19. A 22-year-old woman complains of dizziness and feeling light-headed. She works in an office and
most frequently experiences this when standing up to visit the toilet. She has never fainted. The patient
has lost 5 kg, but attributes this to eating more healthily. She has noticed a recent scar on the back of
her hand which has started to turn very dark. The most appropriate investigation is:

A) Synacthen test

B) Low-dose dexamethasone test

C) Cortisol measurement

D) Urinary free cortisol measurement

E) Abdominal ultrasound (US) scan

Answer: A

20. A 71-year-old man with atrial fibrillation is seen in clinic following an episode of syncope. He
describes getting a poor night’s sleep and, as he got out of bed in the morning, feeling dizzy for a couple
of seconds before the lights dimmed around him. He was woken a couple of seconds later by his wife
who had witnessed the event. She says he went pale and fell to the floor and his arms and legs jerked.
After waking, he was shaken but was ‘back to normal’ a few minutes after the event. His medication
includes aspirin, atenolol and frusemide. What is the most likely diagnosis?

A) Vasovagal syncope

B) Orthostatic hypotension

C) Cardiogenic syncope

D) Transient ischaemic attack (TIA)

E) Seizure

Answer: B

21. A 29-year-old man presents with a 4-week history of polyuria and extreme thirst. The patient denies
difficulty voiding, hesitancy or haematuria, although the urine is very dilute. The patient does not
believe he has lost any weight and maintains a good diet. No findings are found on urine dipstick. The
most appropriate investigation is:
A) Serum osmolality

B) Fasting plasma glucose

C) Urinary electrolytes

D) Magnetic resonance imaging (MRI) scan of the head

E) Water deprivation test

Answer: E

22. You are discussing a patient with your registrar who has become acutely short of breath on the
ward. After performing an arterial blood gas, you have high clinical suspicion that the patient has a
pulmonary embolism. Which of the following is the investigation of choice for detecting pulmonary
embolism?

A) Magnetic resonance imaging (MRI) of the chest

B) High-resolution CT chest (HRCT)

C) Chest x-ray

D) Ventilation/perfusion scan (V/Q scan)

E) CT pulmonary angiogram (CT-Pa)

Answer: E

23. A patient is admitted with pneumonia. A murmur is heard on examination. What finding points to
mitral regurgitation?

A) Murmur louder on inspiration

B) Murmur louder with patient in left lateral position

C) Murmur louder over the right 2nd intercostal space midclavicular line

D) Corrigan’s sign

E) Narrow pulse pressure


Answer: B

24. A 43-year-old woman suffers from Crohn’s disease. A blood test shows the following results:

Haemoglobin 10.5 g/dL

MCV 120 fL

Platelet count 300× 109/L

The most likely diagnosis is:

A) Vitamin B12 deficiency

B) Iron deficiency

C) Hypothyroidism

D) Folic acid deficiency

E) Anaemia of chronic disease

Answer: A

25. A 27-year-old woman develops left leg swelling during week 20 of her pregnancy. Left lower
extremity ultrasonogram reveals a left iliac vein deep vein thrombosis. What is the best next step?

A) bed rest

B) catheter-directed thrombolysis

C) enoxaparin

D) inferior vena cava filter placement

E) Repeat doppler U/S after 1 week

Answer: E
26. A 67-year-old woman is admitted to accident and emergency with pyrexia (38.1°C) and a cough
productive of green sputum. The observations show a pulse rate of 101, BP 80/60 and respiratory rate of
32. She is alert and orientated in space and time. Blood results reveal a WCC of 21, urea of 8.5 and chest
x-ray shows a patch of consolidation in the lower zone of the right lung. She is treated for
severe community-acquired pneumonia. Which of the following is the correct calculated CURB-65
score?

A) 6

B) 8

C) 4

D) 0

E) 1

Answer: C

27. A 69-year-old man presents to clinic with a six-month history of progressive lower back pain which
radiates down to his buttock. He found the pain was exacerbated while taking his daily morning walk
and noticed that it eased going uphill but worsened downhill. He stopped his daily walks as a result and
he now walks only slowly to the shops when he needs to, taking breaks to sit down and ease the pain.
He has a history of hypertension, diabetes and prostatic hyperplasia. What is the diagnosis?

A) Peripheral vascular disease

B) Osteoporotic fracture

C) Spinal stenosis

D) Sciatica

E) Metastatic bone disease

Answer: C

28. A 47-year-old woman is evaluated for difficult-to-control hypertension. She was previously treated
for hypokalemia. On physical examination, temperature is 36.0 °C (96.8°F), blood pressure is 178/100
mm Hg, pulse rate is 58/min, respiration rate is 16/min, and BMI is 29. No abdominal bruit is detected.
Funduscopic examination shows mild arteriolar narrowing. Laboratory studies:

Electrolytes

Sodium 143 meq/L (143 mmol/L)

Potassium 3.5 meq/L (3.5 mmol/L) (after replacement

therapy)

Chloride 101 meq/L (101 mmol/L)

Bicarbonate 33 meq/L (33 mmol/L)

Aldosterone 

Baseline 23 ng/dL (635 pmol/L)

3 Days after high salt intake 15 ng/dL (414 pmol/L)

Renin activity

Baseline <0.1 ng/mL/h (0.1 μg/L/h)

3 Days after high salt intake <0.1 ng/mL/h (0.1 μg/L/h)

Aldosterone to renin activity ratio >50

Which of the following is the most appropriate next step in management?

A) Adrenalectomy

B) Bilateral adrenal vein catheterization

C) CT of the adrenal glands

D) Duplex ultrasonography of the renal arteries

Answer: C

29. Decreased or absent haptoglobin levels are seen in the following conditions EXCEPT

A) Hemolytic anemia 

B) Genetic disorders
C) Acute hepatitis

D) Pregnancy

E) Burns

Answer: C

30. A 41-year-old man is diagnosed with iron deficiency anemia and is found to have heme-positive
stools. Colonoscopy reveals a large ulcerated tumor in his transverse colon. He also has two smaller
polyps in his ascending colon. Pathologic examination of the tumor biopsy reveals adenocarcinoma,
while biopsies of the polyps confirm that these are adenomas. His sister has been diagnosed with

uterine cancer, and two cousins have died of colon cancer. All of the following are true statements
about this case except

A. Referral for genetic counseling is indicated

B. He is at increased risk for other epithelial-derived tumors

C. He likely has familial adenomatous polyposis (FAP), with a germ line mutation in the APC gene

D. His condition is often associated with a defect in DNA mismatch repair

Answer: B

31. Concerning adrenal physiology, which of the following hormones is secreted by the zona
glomerulosa?

A) Adrenaline

B) Noradrenaline

C) Cortisol

D) Aldosterone

E) Angiotensin II
Answer: D

32. A 76-year-old gentleman is referred with an international normalized ratio (INR) of 8.4. He normally
takes warfarin for a prosthetic mitral valve. He had an epistaxis lasting 15 minutes this morning. He is
otherwise asymptomatic and haemodynamically stable. Having stopped his warfarin for tonight, Which
of the following is the most appropriate course of action?

A) 5 mg iv vitamin K

B) 0.5 mg orally (po) vitamin K

C) 4 units of fresh frozen plasma iv

D) 50 units/kg of prothrombin complex concentrate

E) Recheck INR in 24 hours

Answer: E

33. A 64-year-old who suffers from haemochromatosis is seen in A&E with a 2-day history of pain and
swelling in his right knee. On examination it is swollen and he has decreased range of movement. You
aspirate his knee. What will the aspirate be most likely to show?

A) Gram-positive cocci in clusters

B) Positively bifringent crystals

C) Gram-positive cocci in chains

D) Inflammatory cells with haemosiderin deposition

E) Negatively bifringent crystals

Answer: B

34. A 50-year-old male with a history of alcohol dependence and chronic hepatitis C presents to your
clinic with three months of fatigue, weakness and weight loss. He has also noted some “yellowing of my
eyes.” You suspect cirrhosis. All the following are clinical signs of cirrhosis except
A) Arthralgia

B) Asterixis

C) Dupuytren’s contracture

D) Haemorrhoids

E) Testicular atrophy

Answer: A

35. A 75-year-old lady presents with persistent diarrhoea and weight loss. Twenty years previously she
had radiotherapy for carcinoma of the ovary. Thirty years ago she lived in Hong Kong for 18 months. Her
father died of cancer of the colon aged 62 years.

Hb 9.4 g/dL (11.5-16 g/dL)

WCC 9.6 x 109/L (4-11 x 109/L)

Platelets 234 x 10/L (150-400 x 109/L)

B12 153 pmol/L (160-900pmol/L)

Red cell folate 21 nmol/L (130-630 nmol/L)

Ferritin 5 ug/L (20-250 ug/L)

Erythrocyte sedimentation rate (ESR) 36 mm/hour 

Which of the following is the most likely diagnosis?

A) Radiation enteritis

B) Carcinoma of the colon

C) Crohn's disease 

D) Tropical sprue

E) Acquired lactose intolerance

Answer: A
36. A phlebotomist sustains a needlestick injury whilst taking blood from a HIV-positive patient. Which
one of the following has been shown to most reduce the risk of HIV transmission?

A) Thorough immediate washing of the injury site with water

B) Oral lamivudine therapy for 1 month

C) Oral zidovudine therapy for 1 month

D) Oral ritonavir therapy for 1 month

E) Oral triple therapy for 1 month

Answer: E

37. A 33-year-old female is seen in the maternity department.she suffers from SLE. Her child was born
with profound bradycardia. Which of the following antibodies is most likely to be present?

A) Anti-Ro

B) Anti-La

C) Anti-Jo

D) Anti-Scl-70

E) Antiphospholipid antibodies

Answer: A

38. A 67-year-old man presents with nausea and pain in his right upper quadrant. On examination, he
has tender hepatomegaly with an irregular edge. His blood tests are as follows:

Hb 8 g/dL (11.5-16 g/dL)

WCC 17.8 x 109/L (4-11 x 109/L)

Platelets 104 x 109/L (150-400x109/L)

Neutrophils 68%
Normoblasts 8%

Myeloblasts 8%

Myelocytes 5%

Metamyelocytes 5%

Lymphocytes 15%

What is the most likely cause of his anaemia?

A) Gaucher's disease

B) Leukaemoid reaction secondary to acute infection

C) Myelosclerosis

D) Malignant infiltration of bone marrow 

E) Osteopetrosis

Answer: D

39. A 32-year-old woman presents with left loin pain and haematuria. She is known to suffer with
Crohn's disease.

Which of the following is likely to be the aetiology of this presentation?

A) Hypercalciuria

B) Hyperbilirubinaemia

C) Hyperuricaemia

D) Hyperoxaluria

E) Type 1 renal tubular acidosis

Answer: D

40. Regarding glucocorticoid-induced hyperglycemia all the following statements are correct EXCEPT.
A) The risk of new onset diabetes mellitus is related to the total glucocorticoid dose and d)ration of
therapy.

B) Raised fasting plasma glucose is the earliest abnormality. 

C) Reduced insulin sensitivity is the predominant mechanism.

D) Insulin is the most effective and safety therapy

E) Long term use of thiazolidinediones in combination with glucocorticoid is contraindicated.

Answer: B

41. A 37-year-old asymptomatic woman presents for a routine physical examination. Physical
examination reveals a 2-cm right-sided thyroid nodule that is firm, nontender and moves with
swallowing. The rest of the physical examination is unremarkable. Lab Work: TSH 1.8 μU/mL. 

She is referred for fine-needle aspiration biopsy of the thyroid nodule. Which of the following is true
regarding interpreting the cytologic results of thyroid fine-needle aspiration biopsy? 

A) Follicular carcinoma can be accurately diagnosed. 

B) Most fine-needle aspiration biopsy specimens reveal malignant cells. 

C) Follicular adenoma can be accurately diagnosed. 

D) Papillary carcinoma can be accurately diagnosed. 

E) Cystic nodules are more likely than solid nodules to provide diagnostic material.

Answer: D

42. A 40 year old construction laborer admitted to the hospital with sudden onset of dysphagia and
inability to open his mouth. During examination he was conscious but unable to speak, in pain and
pointing to his back. His abdomen was rigid. His upper and lower limb examination was normal. The
most important initial step in the management of this patient is:

A) Perform an urgent CT scan of the neck to exclude pharyngeal abscess

B) Start immediate Ceftriaxone and Vancomycin


C) Start I.V Penicillin,Tetanus Ig and Lorazepam

D) Start I.V Metronidazole,Tetanus Ig and Lorazepam

E) Search for a wound, perform wound cleaning or debridement followed by I.V penicillin and Tetanus Ig

Answer: D

43. 74-year-old man 70 Kg enters the ICU with fever and hypotension. He has a history of kidney stones
with multiple previous bouts of urinary tract infection. 1.5 L of Normal saline bolus started, Urine and
blood cultures are obtained and he was started on Tazocin and Vancmycin, his BP remain 70/40 and
additional 500 ml of Normal saline were infused, CVC placed and norepinephrine started 10 mcg/min,
He was intubated to decrease work of breathing and his hemodynamic data as follow:

BP: 80/45 (55), CVP: 6, Urine output 20 ml/hr, Lactate: 6, ScVO2: 50%.

What is the next step in managing this patient?

A) Start stress dose steroids for refractory septic shock.

B) Add vasopressin to the current regimen for refractory septic shock.

C) Start Dobutamine to increase cardiac output and oxygen delivery.

D) Bolus IV normal saline to keep CVP 8-12.

E) Start furosamide to keep urine output more than 35 ml/hr.

Answer: D

44. A 59-year-old lady is admitted with a 30 minute history of heavy central chest pain associated with
nausea and sweating. Her ECG shows ST elevation in leads V1, V2, V3 and V4. Which of the following
coronary arteries is most likely to be occluded?

A) Circumflex artery 

B) Left anterior descending artery 

C) Obtuse marginal artery 

D) Posterior descending artery 


E) Right coronary artery

Answer: B

45. A 68-year-old man presents with a one-month history of dyspnoea and a 3kg weight loss. On
examination there were signs of a large left pleural effusion, confirmed on Chest X-ray.

Pleural fluid analysis:

Protein 38 g/L

Cytology a few lymphocytes and red blood cells.

Which one of the following investigations should be considered next?

A) Bronchoscopy 

B) CT scan of thorax Correct 

C) Repeat pleural aspiration with biopsy 

D) Thoracoscopic pleural biopsy 

E) Tuberculin test

Answer: B

46. A 65-year-old male is admitted with a two hour history of central chest pain associated with
sweating and nausea.

A myocardial infarction is suspected. Which of the following is an indication for thrombolytic therapy?

A) 1mm ST elevation in leads II, III and aVF. 

B) 1mm ST elevation in leads V2 and V6 

C) 2mm ST depression in leads V2-4 

D) Q waves in leads V2-4 

E) T wave inversion in lead V3-5


Answer: A

47. A 32-year-old female presents with 3-day history of colicky abdominal pain and fatigue. 

Lab: HB 2.9 g/dl, Haematocrit 22%, MCV 78, platelets 60000, retic count 9%, direct and indirect Coomb’s
test (-), ferritin 10, abdominal US shoes portal vein thrombosis. 

Urine analysis: hemosiderin 

The most likely diagnosis is:

A) Factor V Leiden mutation

B) Paroxysmal cold hemoglobinuria

C) Warm autoimmune hemolytic anemia

D) Aplastic anemia

E) Paroxysmal nocturnal hemoglobinuria

Answer: E

48. An 18-year-old attending the A+E department is noted to have central cyanosis. She is perfectly well
but was told to go to A+E by her friends who said she looked blue. What is the most likely cause? 

A) Carbon Monoxide Poisoning 

B) Lead Poisoning 

C) Drinking water contaminated with nitrates Correct 

D) Anorexia Nervosa 

E) Severe Anaemia

Answer: C
49. A 58-year-old woman comes to the emergency department complaining of crampy left upper
quadrant pain that is exacerbated by fatty foods. She has a history of diabetes, hyperlipidemia, and
gallstones and her medications include glyburide, simvastatin, and aspirin. She denies any alcohol or
drug use. She is morbidly obese and her temperature is 37.9 C (100.2 F), blood pressure is 102/87 mm
Hg, pulse is 105/min, and respirations are 23/min. On examination, her lungs are clear to auscultation
bilaterally. Her cardiac sounds are muffled, although her cardiac rhythm is regular. No murmurs are
audible. She has definite left upper quadrant tenderness to palpation, without rebound or guarding.
Rectal examination shows guaiac-negative brown stool. Her amylase and lipase levels are elevated. The
most appropriate next step is to order a

A) an abdominal ultrasound

B) a chest radiograph

C) an electrocardiogram

D) a HIDA scan

E) an upper endoscopy

Answer: A

50. Which of the following statements is NOT true of primary pulmonary tuberculosis:

A) It is characteristically asymptomatic 

B) Miliary spread is commoner in a younger age group 

C) The initial immunological response causes hilar lympadenopathy 

D) Pleural effusion occurs before tuberculin skin testing is positive 

E) A positive tuberculin skin test develops within two weeks of infection

Answer: E

MULTIPLE CHOICE QUESTIONS-9


1. A 78-year-old woman is treated for depression with nortriptyline. Which of the following is the most
common side effect of nortriptyline?

A) impaired cardiac contractility

B) heart block

C) weight loss

D) anticholinergic side effects

E) diarrhea

Answer: D

2. During a neurologic examination, you ask a patient to stand with both arms fully extended and
parallel to the ground with his eyes closed for 10 seconds. What is the name of this test?

A) Babinski sign

B) Dysdiadochokinesis

C) Lhermitte symptom

D) Pronator drift

E) Romberg sign

Answer: D

3. A person with liver disease caused by Schistosoma mansoni would be most likely to have what
condition?

A) Ascites

B) Esophageal varices

C) Gynecomastia

D) Jaundice

E) Spider nevi
Answer: B

4. All of the following hormones is produced by the anterior pituitary EXCEPT:

A) Adrenocorticotropic hormone

B) Growth hormone

C) Oxytocin

D) Prolactin

E) Thyroid-stimulating hormone

Answer: C

5. The following statements concerning pulmonary Stenosis are correct

A) It is the commonest cardiac abnormality in Turner's syndrome

B) The chest x-ray plethoric lung fields

C) There is a recognized association with carcinoid syndrome

D) An ejection click indicates that the Stenosis is sub-valvular

E) The pulmonary component of the second sound is accentuated when the Stenosis is severe

Answer: C

6. A 36-year-old woman with acquired immunodeficiency syndrome (AIDS) and a CD4 count of 35/μL
presents with odynophagia and progressive dysphagia. The patient reports daily fevers and a 20-lb
weight loss. The patient has been treated with clotrimazole troches without relief. On physical
examination, the patient is cachectic with a body mass index (BMI) of 16 and a weight of 86 lb. The
patient has a temperature of 38.2°C (100.8°F). She is noted to be orthostatic by blood pressure and
pulse. Examination of the oropharynx reveals no evidence of thrush. The patient undergoes
esophagogastroduodenoscopy (EGD), which reveals serpiginous ulcers in the distal esophagus without
vesicles. No yellow plaques are noted. Multiple biopsies are taken that show intranuclear and
intracytoplasmic inclusions in large endothelial cells and fibroblasts. What is the best treatment for this
patient’s esophagitis?

A) Ganciclovir

B) Glucocorticoids

C) Fluconazole

D) Foscarnet

E) Thalidomide

Answer: A

7. A 45-year-old man reports to his primary care physician that his wife has noted coarsening of his facial
features over several years. In addition, he reports low libido and decreased energy. Physical
examination shows frontal bossing and enlarged hands. An MRI confirms that he has a pituitary mass.
Which of the following screening tests should be ordered to diagnose the cause of the mass?

A) 24-Hour urinary free cortisol

B) Adrenocorticotropic hormone (ACTH) assay

C) Growth hormone level

D) Serum insulin-like growth factor-1 (IGF-1) level

E) Serum prolactin level

Answer: D

8. Bronchopulmonary aspergillosis is characterized by all of the following EXCEPT: 

A) Underlying asthma 

B) Central bronchiectasis 

C) Elevated serum immunoglobulin E level 


D) Positive serum precipitins for Aspergillus 

E) Positive delayed hypersensitivity skin test to Aspergillus antigens

Answer: B

9. The following are complications of nephrotic syndrome with the exception of 

A) Acute renal failure 

B) Accelerated hypertension 

C) Hypocalcaemia 

D) Pneumococcal infection 

E) Venous thrombosis

Answer: B

10. You have a febrile 37-year-old male with a very high WBC count, most of which are blasts. Most
likely, he has an acute leukemia. A bone marrow has not yet been done, and it is unknown if this is an
acute myeloid leukemia or an acute lymphoblastic leukemia. Which of the following statements is NOT
TRUE?

A) The patient’s fever is likely due to the leukemic cells, and antibiotics should be started only if you
identify a specific infection.

B) The patient is at risk for tumor lysis syndrome and should be started on allopurinol.

C) Aggressive inpatient chemotherapy is required for both the treatment of acute myeloid leukemia and
acute lymphoblastic leukemia.

D) You should consult a hematologist/oncologist as soon as possible.

Answer: A
11. A 48-year old female with rheumatoid arthritis presents to the emergency department with 2-week
pain and tightness behind the left knee. Examination reveals cystic swelling over the left popliteal fossa.
Which of the following is the most appropriate next action? 

A) Arthrogram of the left knee

B) Synovial biopsy of left knee

C) Ultrasound study of left knee and popliteal fossa

D) Venogram of left lower limb

E) None of the above

Answer: C

B) The patient is at risk for tumor lysis syndrome and should be started on allopurinol.

C) Aggressive inpatient chemotherapy is required for both the treatment of acute myeloid leukemia and
acute lymphoblastic leukemia.

D) You should consult a hematologist/oncologist as soon as possible.

Answer: A

11. A 48-year old female with rheumatoid arthritis presents to the emergency department with 2-week
pain and tightness behind the left knee. Examination reveals cystic swelling over the left popliteal fossa.
Which of the following is the most appropriate next action? 

A) Arthrogram of the left knee

B) Synovial biopsy of left knee

C) Ultrasound study of left knee and popliteal fossa

D) Venogram of left lower limb

E) None of the above


Answer: C

12. Concerning pseudomyxoma peritonei: All are true EXCEPT

A) Begins with a mucinous cystadenocarcinoma, most commonly involving either the ovary or appendix.

B) Rupture of the lesion results in diffuse metastatic spread with implantation of the mucin secreting
lesions on the omentum as well as the peritoneum. 

C) Average age of presentation of this disease process is 45-55, with men making up 80% of cases.

D) Presenting symptoms are non-specific, generally including pain, but also usually with a palpable mass.

E) A 5-year survival of these patients is only approximately 50%.

Answer: C

13. A 60-year-old man with end stage renal failure from chronic glomerunephritis presents with acute
onset of gross hematuria and mild flank pain. He has been on hemodialysis for 4 years, and his course
otherwise been uneventful. He was afebrile and the hematuria resolved without intervention. Which of
the following is the most appropriate action at this time?

A) Angiography

B) Computed tomography (to rule out acquired cystic kidney disease)

C) Renal ultrasound

D) Intravenous pyelography

E) None of the above

Answer: B

14. A 30-year-old female with systemic lupus erythematosus is recovering from a fracture of the right
femur and right radius following a motor vehicle accident. She has been in the hospital for 5 days. She
has a temperature spike of 39.0 °C (102.2 °F). Blood cultures are drawn which grow yeast, species to be
identifi ed. She does not appear ill or toxic. No obvious source of infection is found. The patient has an
indwelling central catheter. In addition to changing the patient’s central line, which of the following do
you recommend?

A) Continued observation.

B) Computed tomography of the chest and abdomen.

C) Start fluconazole.

D) Start liposomal amphotericin B.

E) Repeat blood cultures and treat if positive

Answer: C

15. A 78-year-old woman is admitted to the medical intensive care unit (ICU) with multilobar
pneumonia. On initial presentation to the emergency department, her initial oxygen saturation was 60%
on room air and only increased to 82% on a non-rebreather face mask. She was in marked respiratory
distress and intubated in the emergency department. Upon admission to the ICU, she was sedated and
paralyzed. The ventilator is set in the assist-control mode with a respiratory rate of 24 breaths/min, tidal
volume of 6 mL/kg, FiO2 of 1.0, and positive end-expiratory pressure of 12 cmH2O. An arterial blood gas
measurement is performed on these settings; the results are pH of 7.20, PCO2 of 32 mmHg, and PO2 of
54 mmHg. What is the cause of the hypoxemia?

A) Hypoventilation alone

B) Hypoventilation and ventilation-perfusion mismatch

C) Shunt

D) Ventilation-perfusion mismatch

Answer: C

16. Which of the following may be responsible for a hypokalaemic hypertension? 

A) Non-classical congenital adrenal hyperplasia 

B) Barter's syndrome 

C) Diabetic nephropathy 
D) Liddle's syndrome 

E) Type IV renal tubular acidosis

Answer: D

17. A 63-year-old male recently admitted with sepsis is noted to have a urine output of approximetely
20 mls per hour. The oliguria is more likely to be due to prerenal failure than intrinsic renal failure if:

A) A urine free of red blood cells or casts Correct 

B) A urine:plasma urea ratio < 3 

C) Urine osmolality <350 mOsm/l 

D) A blood pressure of 150/90 and good tissue perfusion. 

E) Urinary sodium >10mmol/l

Answer: A

18. A 50-year-old female presents with acute chest pain and dyspnea. Examination reveals bilateral
ankle oedema with 24 hr urine protein assessment showing 8g/d (<0.2). Which is the most likely
explanation for these findings?

A) Factor V Leiden Incorrect answer selected 

B) Reduced antithrombin III activity This is the correct answer 

C) Reduced concentration of Von Willebrand’s factor 

D) Reduced fibrinogen concentration 

E) Reduced factor VIII

Answer: B
19. A 19-year-old woman became breathless while travelling on an aeroplane. Which one of the
following features most strongly supports a diagnosis of acute hyperventilation related to a panic
disorder? 

A) Carpal spasm. 

B) Finger paraesthesiae. 

C) Hypotension. 

D) Light-headedness. 

E) Loss of conciousness

Answer: A

20. Which ONE of the following is a recognised feature of achondroplasia? 

A) Autosomal recessive inheritance 

B) May be diagnosed radiologically at birth 

C) Increased liability to pathological fractures 

D) Shortened spine 

E) Subfertility

Answer: B

21. You are asked to see a 67-year-old woman who was recently discharged from the hospital following
internal fixation of fragility fracture of left femoral neck. She had hypertension and primary
hypothyroidism. Her medications include lisinopril 10 mg and L-thyroxine 100ug per day. Family history
was unremarkable. On physical examination BMI 23.9 kg/m2 BP 130/84 mmHg. Laboratory studies
showed: Serum creatinine 1.2 mg/dl Serum calcium 9.2 mg/dl: Serum PTH 69 pg/ml (N 10-58): Serum 25
hydroxy vitamin D 42 ng/mg (N 30-70): Serum TSH 1.9 mU/l (N 0.4-4.0 u): Bone density measured by
DXA revealed: T score : Hip-2.9 Lumbar spine -2.7. In addition to vitamin D and calcium supplement,
which of the following you recommend to reduce the incidence of another hip fracture?

A) Zoledronate 5mg IV yearly


B) Combined estrogen and progestin.

C) Raloxifene 60 mg per day.

D) Alphacalcidol 1 µg per day

Answer: A

22. A 29-year-old man with episodic abdominal pain and stress-induced edema of the lips, tongue, and
occasionally larynx is likely to have low functional or absolute levels of which of the following proteins?

A. C1 esterase inhibitor

B. C5A (complement cascade)

C. Cyclooxygenase

D. Immunoglobulin (Ig) E

E. T-cell receptor, α chain

Answer: A

23. Pseudopseudohypoparathyroidism is characterized by all of the following EXCEPT:

A) Hypocalcemia 

B) G5 mutation

C) Albright’ s hereditary osteodystrophy 

D) inheritance of the genetic defect from the father

E) Normal urinary c-AMP response to parathyroid hormone

Answer: A
24. Most of the manifestations of acute rheumatic fever present approximately 3 weeks after the
precipitating group A streptococcal infection. Which manifestation may present several months after the
precipitating infection?

A) Chorea

B) Erythema marginatum

C) Fever

D) Polyarthritis

E) Subcutaneous nodules

Answer: A

25. A 57-year-old woman with depression and chronic migraine headaches reports several years of dry
mouth and dry eyes. Her primary complaint is that she can no longer eat her favorite crackers, although
she does report photosensitivity and eye burning on further questioning. She has no other associated
symptoms. Examination shows dry, erythematous, sticky oral mucosa. All of the following tests are likely
to be positive in this patient EXCEPT:

A) La/SS-B antibody

B) Ro/SS-A antibody

C) Schirmer I test

D) Scl-70 antibody

E) Sialometry

Answer: D

26. Which of the following has a well-established association with gastroesophageal reflux?

A) Chronic sinusitis

B) Dental erosion

C) Pulmonary fibrosis
D) Recurrent aspiration pneumonia

E) Sleep apnea

Answer: B

27. A 68-year-old woman presents with a left arm tremor. She has had the tremor for about 3 years and
recently was noticed to have developed a side-to-side head tremor as well. Her tremor worsens when
she performs movements that involve her bringing her hand to her face, such as smoking or drinking
coffee. Of note is that if she drinks beer, the tremor seems to improve.

Which of the following is the likely etiology?

A) Gilles de la Tourette syndrome 

B) Parkinson disease 

C) Myasthenia gravis 

D) Benign essential tremor 

E) Huntington disease

Answer: D

28. The following are recognized features of Pancoast's tumour except:

A) ipsilateral Horner's syndrome

B) wasting of the dorsal interossei

C) pain in the arm radiating to the fourth and fifth fingers

D) erosion of the first rib

E) weakness of abduction at the shoulder

Answer: E
29. Major risk factor for osteoporosis includes all of the following EXCEPT.

A) Tobacco smoking 

B) Glucocorticoid therapy.

C) Primary hyperparathyroidism.

D) History of previous fragility fracture after the age 40 year.

E) Hypogonadism

Answer: D

30. A 20-year-old male student is assessed for shortness of breath that occurs whilst running. He has no
other symptoms and does not smoke. Examination, full blood count, and chest X-ray are normal. Which
of the following is most likely to be helpful in confirming the suspected diagnosis?

A) Arterial blood gas studies before and after exercise

B) Determination of lung volumes and diffusing capacity

C) Measurement of venous blood lactate before and after exercise

D) Spirometry before and after administration of bronchodilators

E) Spirometry before and after exercise

Answer: E

31. A 65-year-old man has the following blood results on admission to hospital:

Na 132 mmol/L (135-145 mmol/L)

K 8.6 mmol/L (3.5-5 mmol/L)

Urea 42.4 mmol/L (2.5-6.7 mmol/L)

Creatinine 1178 um0l/L (70-150 umol/L)

The patient appears unwell and the ECG shows tented T waves, absent P waves and widened QRS.
Which of the following therapeutic interventions should be performed first?
A) Immediate haemodialysis

B) Intravenous insulin and dextrose

C) Intravenous calcium gluconate

D) Nebulized salbutamol

E) Oral calcium resonium

Answer: C

32. A 55-year-old man with chronic hepatitis C is being considered for liver transplantation. The patient
has cirrhosis that was documented by liver biopsy 10 years ago. For the past 3 months, he has had
ascites and edema, which are poorly controlled with diuretics. Lactulose was recently begun because of
confusion. Which of the following combinations of laboratory studies will be most helpful in estimating
his survival over the next 6 months? 

A) Serum total bilirubin and INR 

B) Serum aspartate aminotransferase and gamma globulin 

C) Serum alanine aminotransferase and hepatitis C RNA (HCV RNA) 

D) Serum alkaline phosphatase and ammonia 

E) Serum albumin and ?-glutamyltransferase

Answer: A

33. Signs and symptoms of opioid withdrawal include all of the following EXCEPT

A) Increased blood pressure (BP) and heart rate

B) Seizures

C) Abdominal cramps

D) Jerking of the legs

E) Hyperthermia
Answer: B

34. Which of the following β-adrenergic antagonists is a nonselective β1 and β2 blocker?

A) Atenolol

B) Betaxolol

C) Esmolol

D) Metoprolol

E) Nadolol

Answer: E

35. In primary hypothyroidism increased requirements for L thyroxine occur in the following conditions
EXCEPT:

A) Nephrotic syndrome with massive proteinuria.

B) Pregnancy.

C) Achalasia

D) Helicobacter pylori associated chronic gastritis.

E) Use of estrogen replacement therapy.

Answer: C

36. A 70-year-old man is admitted with new-onset atrial fibrillation with an apical rate of 120/minute
and a BP of 100/70 mm Hg. Shortly after admission he develops moderate lower abdominal pain and
diarrhea with dark red blood.

Which is the most appropriate course of action?

A) Cardioversion 
B) Unfractionated heparin

C) Mesenteric angiography

D) Laparotomy

E) Digoxin

Answer: A

37. A 73-year-old male presents to the clinic with 3 months of increasing back pain. He localizes the pain
to the lumbar spine and states that the pain is worst at night while he is lying in bed. It is improved
during the day with mobilization. Past history is notable only for hypertension and remote cigarette
smoking. Physical examination is normal. Laboratory

studies are notable for an elevated alkaline phosphatase. A lumbar radiogram shows a lytic lesion in the
L3 vertebra. Which of the following malignancies is most likely?

A) Gastric carcinoma

B) Non-small cell lung cancer

C) Osteosarcoma

D) Pancreatic carcinoma

E) Thyroid carcinoma

Answer: B

38. A 36-year-old African-American woman with systemic lupus erythematosus presents with the acute
onset of lethargy and jaundice. On initial evaluation, she is tachycardic, hypotensive, appears pale, is
dyspneic, and is somewhat difficult to arouse. Physical examination reveals splenomegaly. Her initial
hemoglobin is 6 g/dL, white blood cell count is 6300/μL, and platelets are 294,000/ μL. Her total bilirubin
is 4 g/dL, reticulocyte count is 18%, and haptoglobin is not detectable. Renal function is normal, as is
urinalysis. What would you expect on her peripheral blood smear?

A) Macrocytosis and PMN’s with hypersegmented nuclei

B) Microspherocytes
C) Schistocytes

D) Sickle cells

E) Target cells

Answer: B

39. A 22 year-old lady presents to her GP with facial swelling, most marked in the morning, and
shortness of breath. Urinalysis shows protein +++. Results were as follows: 

24-hour urinary protein 4.2 g; 

Haemoglobin(Hb) 10.8 g/l (11.5-16.0 g/L); 

Urea 22.8 mmol/L (2.5-6.5 mmol/L); 

Creatinine 374 µmol/L (70-150 umol/L)

C3 0.24 g/L (0.55-1.2g/L)

C4 0.29g/L (0.2-0.5 g/L)

Which of the following is the most likely diagnosis?

A) Minimal change nephropathy

B) Systemic lupus erythematosus

C) Rheumatoid arthritis

D) Cryoglobulinaemia

E) Diabetes mellitus

Answer: B

40. A 31-year-old male with hemophilia A is admitted with persistent gross hematuria. He denies recent
trauma or any history of genitourinary pathology. The examination is unremarkable. Hematocrit is 28%.
All the following are treatments for hemophilia A except

A) desmopressin (DDAVP)
B) fresh-frozen plasma (FFP)

C) cryoprecipitate

D) recombinant factor VIII

E) plasmapheresis

Answer: E

41. A 50-year-old male is taken to the General Practitioner by his long suffering wife. His snoring (which
has been steadily increasing in loudness over the past 18 months) is troublesome at home. She says that
he makes noises and moves around whilst asleep. He reports no problems with sleeping. He does admit
to gaining 20 kg in weight over the past one year, and to falling asleep during the day. A sleep study is
performed. Which of the following findings would be most compatible with this man's clinical
presentation?

A) Fragmented sleep, cessations of airflow measured at the nose accompanied by an increase in


oesophageal pressure swings and episodic oxygen desaturation. 

B) Normal sleep quality, bradycardic episodes, oxygen desaturation but normal airflow. 

C) Normal sleep quality but cessations of airflow measured at the nose with decreased abdominal wall
motion during these flow cessations. 

D) Progressive oxygen desaturation during the night and alternating periods of hyperventilation and
hypoventilation. 

E) Tachycardia, sleep fragmentation, episodes of hypoventilation with minimal oxygen desaturation.

Answer: A

42. A 50-year-old female presents with acute chest pain and dyspnoea. Examination reveals bilateral
ankle oedema with 24 hr urine protein assessment showing 8g/d (<0.2). Which is the most likely
explanation for these findings? 

A) Factor V Leiden 

B) Reduced antithrombin III activity 


C) Reduced concentration of Von Willebrand’s factor 

D) Reduced fibrinogen concentration 

E) Reduced factor VIII

Answer: B 

43. A 60-year-old woman is establishing care in your clinic. She has occasional numbness and tingling in
her fingers and toes and has noted some numbness around her mouth, especially when she is stressed
or anxious. She had thyroid surgery for Graves disease about 2 years ago and takes 100 µg of
levothyroxine and 1 tablet of calcium daily. On exam, her blood pressure is 130/80, pulse 80, and she
has cramping in her right forearm and fingers when the blood pressure cuff is attached. Based on this
history and exam, which of the following is most likely?

A) Calcium 8.5, PTH 65, PO4 4.5

B) Calcium 9.5, PTH 35, PO4 4.0

C) Calcium 6.0 (8.5-10.5), PTH 2 (10-65), PO4 6.0 (2.7-4.5)

D) Calcium 10.8, PTH 108, PO4 2.3

E) Calcium 8.0, PTH 98, PO4 2.1

Answer: C

44. A 70-year-old man was referred by his GP with difficult to treat hypertension. He had long-standing
hypertension which had been well controlled over many years but recently he was found to have a
blood pressure of 190/110 mmHg which proved resistant to additional treatment. He was generally
asymptomatic and complied with medication. Investigations showed normal U+Es. Which one of the
following is the most likely cause? 

A) Chronic pyelonephritis 

B) Conn’s syndrome (primary hyperaldosteronism) 

C) Phaeochromocytoma 

D) Polycystic kidney disease 


E) Renovascular disease

Answer: E

45. Regarding differentiated thyroid carcinoma the following statements are correct EXCEPT.

A) Radiation is a recognized risk factor.

B) Palpillary carcinoma is the most common type.

C) Fine needle aspiration biopsy is very sensitive for the diagnosis of minimally invasive follicular
carcinoma.

D) Serum thyroglobulin is a sensitive tumor marker for patients who underwent total thyroidectomy and
radioiodine remnant ablation

Answer: C

46. Risk factors for foot ulcers in patient with diabetes includes all the following EXCEPT:

A) Foot deformity.

B) Peripheral neouropathy.

C) Tobacco smoking.

D) Female sex.

E) Poor glycemic control.

Answer: D

47. A 51-year-old man is post-procedure day number 2 from an upper endoscopy and banding for
bleeding esophageal varices. The patient has a 7-year history of chronic active hepatitis and over the
past few years has developed stigmata associated with cirrhosis and worsening portal hypertension.
Three days ago, he presented to the emergency department with bright red blood per mouth and
rectum and a nasogastric tube evacuated bright red blood and coffee grounds from the patient's
stomach. He was admitted to the hospital, transfused with 2 units of red blood cells and underwent an
endoscopy. On preparation for the patient's discharge, you have a long discussion with your patient
about the course of events. In counseling this patient on his future risks and course of therapy, you
should advise him that:

A) The risk of rebleeding is between 50% and 80% and medical therapy is indicated

B) The risk of rebleeding is between 50% and 80% and surgical therapy is indicated

C) The risk of rebleeding is between 50% and 80%, but no therapy is indicated given the nature of the
problem

D) There is no concern for rebleeding

E) There is no concern for rebleeding, he is an imminent candidate for transplant

MULTIPLE CHOICE QUESTIONS-10

1. Which one of the following is NOT a contraindication to intravenous (IV) tPA in patient with ischemic
stroke?

A) Age >75

B) INR >1.7

C) Platelets <100,000

D) Stroke within last 3 months

E) Glucose <50

Answer: A

2. A 28-year-old female presents to the emergency department with a severe headache and lethargy.
The headache came on fairly quickly whilst she was watching television. She is otherwise fit and well,
has no medical problems and takes no regular medications. She lives at home with her husband and
two-year-old daughter. Whilst in the emergency department she is given two co-codamol for her
persisting headache but then vomits. On examination, there is no focal neurology but she is slightly
drowsy and her GCS is 14/15. Hb 126 g/l; Platelets 274 * 109/l; WBC 10.9 * 109/l; Na+ 124 mmol/l; K+
5.0 mmol/l; Urea 4.1 mmol/l; Creatinine 124 µmol/l. What is the most likely diagnosis?

A) Tension headache 

B) Addison's disease 

C) Pituitary apoplexy 

D) Sub-arachnoid haemorrhage 

E) Cranial diabetes insipidus

Answer: C

3. An 18-year-old woman with a microcytic anemia is diagnosed with alpha-thalassemia. Formation of
which of the following hemoglobins is increased in alpha-thalassemia?

A) H

B) A

C) F

D) A2

E) C

Answer: A

4. Which of the following is most characteristic of diabetic neuropathy?

A) it is usually bilateral

B) pain is not a feature

C) it most commonly affects the brain

D) it spares the autonomic system

E) it responds to meticulous control of blood glucose


Answer: A

5. A 19 year-old female is referred following a visit to the dentist where marked erosion of her teeth was
noted. She was entirely asymptomatic and her only medication was the oral contraceptive pill. On
examination her blood pressure was 110/70 mmHg and her body mass index was 21.5 kg/m2 (18 - 25).
Investigations sodium 135 mmol/l, potassium 2.1 mmol/l, bicarbonate 42 mmol/l, urea 2.6 mmol/L
corrected calcium 2.08 mmol/ alkaline phosphatase 201 iu/l (50-110) What is the most likely diagnosis? 

A) Bulimia nervosa 

B) Conn's syndrome 

C) Laxative abuse 

D) Pregnancy 

E) Primary hypoparathyroidism

Answer: E

6. An 88-year-old man is taken to A&E by his daughter as he had been feeling unwell that day with chest
pains. An ECG reveals that he is suffering an inferior myocardial infarction. He has a heart rate of 33 bpm
and blood pressure of 90/60 mmHg. He is given 600mcg of atropine to no effect, this is repeated up to
3mg. You commence an infusion of TPA but his rate is not immediately improving. Which of the
following treatments would be most suitable to use for his arrhythmia?

A) Adrenaline

B) Glucagon

C) Temporary pacing

D) Dopamine

E) Dobutamine

Answer: C
7. A 55-year-old politician with no previous medical history is admitted to A&E. He has been feeling
unwell since Saturday morning and an ECG taken on admission to the Emergency Department at 1am on
the Monday morning shows atrial fibrillation at rate of 130 bpm with BP 150/80 mmHg. He is Echoed
(trans-thoracic) and there is no evidence of the left atrial thrombus, no abnormal ventricular wall
movement and his ejection fraction is normal. What would be the most appropriate initial treatment for
this patient?

A) Anticoagulate with IV heparin and carry out DC cardioversion

B) Arrange a TOE next morning and if negative give 300 mg flecainide IV per day

C) Anticoagulate with warfarin and give 300 mg amiodarone IV

D) Start 300 mg amiodarone IV with no anticoagulation

E) Anticoagulate with LMWH and give 300 mg oral flecainide per day

Answer: E

8. A74-year-old woman has metastatic bone disease on x-ray. Which of the following mediators is least
likely to be involved?

A) interleukin-6 (IL-6)

B) ectopic parathyroid hormone (PTH)

C) tumor necrosis factor (TNF)

D) interleukin-1 (IL-1)

E) prostaglandins

Answer: B

9. The medication of choice to treat a patient in torsades de pointes is:

A) Epinephrine.

B) Flecainide.

C) Calcium gluconate
D) Magnesium sulfate.

E) Procainamide

Answer: D

10. A 43-year-old white male presents to the emergency room for intractable nausea and vomiting.
During inpatient admission paperwork, you complete his VTE risk assessment and realize he is a high VTE
risk. What would you choose for DVT prophylaxis (BMI = 45, CrCl = 75 ml/min)?

A) Early ambulation

B) Lovenox 40 mg SQ daily

C) Lovenox 40 mg SQ BID

D) Sequential compression device

E) Lovenox 100 mg SQ daily

Answer: C

11. A 61-year-old man has a non-ST-elevation MI and is admitted to the coronary care unit. The
following day, he develops bradycardia but no symptoms. His blood pressure is 126/84 mm Hg, pulse
50/min, and on examination, the heart sounds are normal, with no extra sounds or rubs. His ECG has
changed. Which of the following ECG findings is the best indication for this patient to receive a
pacemaker?

A) persistent bradycardia

B) second-degree AV block Mobitz type I

C) first-degree AV block

D) new right bundle branch block

E) left bundle branch block (LBBB) and second-degree AV block Mobitz type II

Answer: E
12. A79-year-old man on quinidine for paroxysmal atrial fibrillation develops thrombocytopenia. Which
of the following is the most likely mechanism for this syndrome?

A) it is due to bone marrow suppression

B) it is due to sequestration in the spleen

C) it is due to intravascular destruction of platelets

D) it is immunologically mediated

E) there is cross-reactivity with penicillin

Answer: D

13. A 28-year-old man develops viridans group streptococci septicemia. Which of the following cardiac
lesions has the highest risk of developing endocarditis?

A) ventricular septal defect

B) atrial septal defect, secundum type

C) mitral valve prolapse with regurgitation

D) pure mitral stenosis

E) asymmetric septal hypertrophy

Answer: A

14. A 61-year-old man is taking over-the-counter pseudoephedrine for cold and flulike symptoms. Over
the course of the next few days, he experiences improvement in his rhinitis but should be concerned
about the possibility of which of the following problems?

A) Contraction of the iris causing visual changes

B) Constriction of the bronchioles causing increased pulmonary secretions

C) Erectile function improvement


D) Thinning of his salivary glandular secretions

E) Urinary retention

Answer: E

15. A 45-year-old woman develops symptoms of shortness of breath on exertion, easy fatigue, and
jaundice. On examination she is pale, and there is a palpable spleen, but no lymphadenopathy. Her
hemoglobin is 9.0 g/dL, the reticulocyte count 9%, and the direct antibody test (Coombs’) is positive.
Which of the following bone marrow findings is most likely

to be seen in this patient?

A) megaloblastic changes

B) giant metamyelocytes

C) increased erythroid-to-myeloid ratio

D) increased lymphocytes

E) shift to left of the myeloid series

Answer: C

16. A 68-year-old man presents with symptoms and signs of CHF. Which of the following is a
contraindication to use furosemide?

A) has hypoalbuminemia

B) is oliguric

C) has acidosis

D) had a rash with trimethoprimsulfamethoxazole

E) is on anticoagulants

Answer: D
17. Which of the following features of barbiturate induced coma is most likely correct?

A) requires at least 20–30 times the full sedative dose

B) is increasing in frequency

C) is characterized by an initial period of hyperventilation

D) causes death by depression of the cardiovascular system

E) causes death by pulmonary complications

Answer: E

18. A 62-year-old man presents with dark tarry stools and light-headedness. Upper endoscopy finds an
ulcerating lesion in his stomach and biopsies confirm gastric cancer. Which of the following is a risk
factor for carcinoma of the stomach?

A) Helicobacter pylori infection

B) high socioeconomic status

C) high protein diet

D) high alcohol consumption

E) high fat diet

Answer: A

19. An agitated and nervous 24-year-old woman has had severe wheezing and shortness of breath for 2
days. After receiving oxygen, steroids, and salbutamol (Ventolin) in the emergency room, her breathing
improves. She is still wheezing and now feels tremulous and anxious with a pulse of 110/min and
respirations 30/min. Arterial blood gases on oxygen reveal a pH of 7.40, PO2 340 mm Hg, PCO2 40 mm
Hg, and bicarbonate of 24 mEq/L. She is hospitalized for further treatment. Which of the following
treatments or medications should be avoided in her?

A) theophylline
B) sedatives

C) corticosteroids

D) sympathomimetic amines

E) intravenous (IV) fluids

Answer: B

20. A 63-year-old man developed a transient episode of vertigo, slurred speech, diplopia, and
paresthesias. He is symptom-free now, and clinical examination is entirely normal. His past

medical history is significant for hypertension and dyslipidemia. Which of the following is the most likely
cause for symptoms?

A) posterior circulation transient ischemic attack (TIA)

B) anterior communicating artery aneurysm

C) hypertensive encephalopathy

D) pseudobulbar palsy

E) occlusion of the middle cerebral artery

Answer: A

21. A 19-year-old man is recently diagnosed with type 1 diabetes and attends your clinic to ask about
possible complications in the future. He mentions an uncle who has end-stage renal disease due to
poorly controlled diabetes and specifically inquires about testing for early signs of renal impairment. The
most appropriate investigation is:

A) Blood pressure

B) Microalbuminuria

C) Serum creatinine

D) Serum electrolytes
E) Urine dipstick for glucose

Answer: B

22. Which of the following features is least likely to be found in a patient with tuberous sclerosis? 

A) Adenoma sebaceum 

B) Cafe-au-lait spots 

C) Retinal hamartomas 

D) 'Ash-leaf' spots 

E) Lisch nodules

Answer: E

23. Which of the following statements is true about gastric lavage?

A) Except in extraordinary circumstances it should be done only in the first 1.5 hours after an overdose.

B) Patients who have had gastric lavage have higher incidence of pulmonary aspiration than patients
who have not.

C) The maximum volume that should be used is 5 liters.

D) It can push pill fragments beyond the pylorus.

E) All of the above are true

Answer: E

24. Which ONE of the following is true regarding acute pulmonary embolism?

A) a normal ECG excludes the diagnosis

B) embolectomy is more effective than thrombolysis in improving survival


C) Heparin is as effective as thrombolytic therapy

D) the presence of hypoxaemia is an indication for thrombolysis

E) thrombolysis administered through a peripheral vein is as effective as through a pulmonary artery


catheter

Answer: E

25. A 32-year-old woman is admitted to the intensive care unit (ICU) for sepsis due to pyelonephritis.
Her serum creatinine is 0.5 mg/dL at time of admission. 24-h later her urine output started to decline,
but her creatinine remains stable. The intensivist feels that the patient is in the process of developing
acute kidney injury (AKI). Which one of the following has been clinically validated as a biomarker for the
early diagnosis of AKI?

A) Neutrophil gelatinase-associated lipocalin (NGAL)

B) N-acetyl-β-D-galactosaminidase (NAG)

C) Kidney injury molecule-1 (KIM-1)

D) Interleukin-18 (IL-18)

E) None of the above

Answer: E

26. A 51-year-old man with severe, recurrent, and extensive peptic ulceration is given a diagnosis of
Zollinger–Ellison syndrome. Which of the following is true with respect to this syndrome? 

A) Common cause of peptic ulcer disease 

B) Never malignant 

C) Should be treated by gastrectomy 

D) Diarrhoea can be the presenting feature

E) Octreotide is contraindicated
Answer: D

27. For which of these overdoses is charcoal NOT indicated?

A) Acetaminophen

B) Aspirin

C) Iron

D) Digoxin

E) Opiates

Answer: C

28. Whichof the following antimicrobialsis associated with prolongation of the QT interval?

A) Coamoxiclav

B) Gentamicin

C) Cefuroxime

D) Erythromycin

E) Isoniazid

Answer: D

29. Whichof the following is associated with Hyperuricaemia?

A) is usually due to an excess purine consumption

B) occurs in association with acute lymphoblastic leukaemia

C) in primary gout is inherited in an autosomal dominant manner

D) can be reduced with low dose aspirin therapy


E) can be treated with uricosuric drugs even in renal failure

Answer: B

30. In an investigation for lung malignancy, all of the following may produce a false positive result on a
PET-CT except:

A) Pulmonary hamartoma

B) Intralobar sequestration

C) Tuberculosis

D) Pneumonia

E) Scarring

Answer: B

31. A 38-year-old woman presents with bilateral pitting ankle oedema. The jugular venous pressure is
not raised and the hepatojugular reflux was negative. The woman's ankle oedema would not be
explained by which of the following conditions? 

A) Pelvic tumour 

B) Amlodipine therapy 

C) Cyclic oedema 

D) Hypoalbuminaemia 

E) Right heart failure

Answer: E

32. A 35-year-old healthy man was seen for an annual medical review as part of his company
employment policy. Examination reveals oral thrush in an otherwise fit and healthy man. Further tests
confirm oral candidiasis. He denies any illness and he is receiving no drugs. Which one of the following is
the most likely underlying disease? 

A) Lymphoma 

B) Alcoholism 

C) HIV infection 

D) Chronic renal failure 

E) Heroin addiction

Answer: C

33. A 40-year-old Egyptian tourist is admitted with haematemesis. He is known to have chronic liver
disease caused by Schistosoma mansoni infection. Physical examination would reveal which one of the
following clinical signs? 

A) Splenomegaly 

B) Jaundice 

C) Ascites 

D) Spider nevi 

E) Gynaecomastia

Answer: A

34. A 68-year-old man presents with recent onset headache and vomiting. Ophthalmoscopy reveals
optic atrophy in the right eye and papilloedema in the left one. The most likely diagnosis is? 

A) Pinealoma 

B) Medulloblastoma 

C) Acoustic neuroma 

D) Sphenoid wing meningioma 


E) Pituitary adenoma

Answer: D

35. A 62-year-old man presented with several non-healing ulcerated lesions on the lower extremities
and hand, present for approximately three years. He relates that each lesion began as a tender
erythematous papule that subsequently ulcerated. The lesions have failed to respond to topical steroids
and topical antifungal agents. He received skin grafts over the lower extremity lesions one year ago but
the grafts ulcerated. There are multiple ulcerate lesions bilaterally on the lower extremities and one on
the right hand. The lesions have dusky purple margins and a halo of erythema. They range in size from
2.5 to 6 cm in diameter. The bases of the ulcers are verrucous with regions of necrosis and granulation
tissue. The most likely diagnosis is 

A) Venous insufficiency 

B) Pyoderma gangrenosum 

C) Tropical ulcer 

D) Bacterial skin infection 

E) Ischaemic ulcers

Answer: B

36. A 50-year-old accountant is referred for investigation of fever and general ill health. His initial renal
function tests show: urea 45 mmol/I, creatinine 480 umol/l. Which one of the following medication dose
interval should be most prolonged when used in this patient? 

A) Gentamicin 

B) Amoxicillin 

C) Vancomycin 

D) Penicillin 

E) Cefuroxime
Answer: C

37. A 58-year-old chronic alcoholic was admitted with increasing shortness of breath and a distended
abdomen. Physical examination revealed evidence for ascites and pitting oedema in both legs. The pulse
was 94 bpm, blood pressure 110/60 mmHg and the jugular venous pressure was not raised. The
condition least likely to be responsible for this clinical presentation is 

A) Alcoholic cardiomyopathy 

B) Liver cirrhosis 

C) Bowel cancer with liver metastasis 

D) Tuberculous peritonitis 

E) Portal vein thrombosis

Answer: A

38. A 24-year-old male inpatient appears confused and is behaving in an unusual manner on the ward.
Nursing staff report a disrupted sleep pattern. The patient has no psychiatric history. You wish to
exclude an organic cause for his behaviour and order an electroencephalogram (EEG). In which of the
following disorders are EEG abnormalities least expected? 

A) Herpes simplex encephalitis 

B) Interictal psychosis 

C) Metabolic encephalopathy 

D) Hebephrenic schizophrenia 

E) Subacute sclerosing panencephalitis

Answer: D

39. A 72-year-old woman is admitted to the Coronary Care Unit with an acute inferior myocardial
infarction. During ECG recording it is noticed that her heart rate is 40 bpm with sinus rhythm and blood
pressure is 87/55 mmHg. The most appropriate immediate action is 
A) Keep monitoring the pulse and the blood pressure for a further 24 hours 

B) Insert temporary pacemaker 

C) Organise emergency percutaneous coronary angioplasty 

D) Give 24-hour isoprenaline infusion 

E) Give intravenous atropine sulphate (0.6 mg)

Answer: E

40. An 80-year-old man is referred from a nursing home with fever, rigors, nausea and vomiting. He is
bedridden and has sacral decubitus ulcers. Blood cultures grow Streptococcus milleri. The most likely
site of infection is? 

A) Sacral bedsores 

B) Urinary tract infection 

C) Liver abscess 

D) Endocarditis 

E) Osteomyelitis

Answer: C

41. A 75-year-old lady presents with persistent diarrhoea and weight loss. Twenty years previously she
had radiotherapy for carcinoma of the ovary. Thirty years ago she lived in Hong Kong for 18 months. Her
father died of cancer of the colon aged 62 years.

Hb 9.4 g/dL (11.5-16 g/dL)

WCC 9.6 x 109/L (4-11 x 109/L)

Platelets 234 x 10/L (150-400 x 109/L)

B12 153 pmol/L (160-900pmol/L)

Red cell folate 21 nmol/L (130-630 nmol/L)


Ferritin 5 ug/L (20-250 ug/L)

Erythrocyte sedimentation rate (ESR) 36 mm/hour 

Which of the following is the most likely diagnosis?

A) Radiation enteritis

B) Carcinoma of the colon

C) Crohn's disease 

D) Tropical sprue

E) Acquired lactose intolerance

Answer: A

42. A 50-year-old male with a history of alcohol dependence and chronic hepatitis C presents to your
clinic with three months of fatigue, weakness and weight loss. He has also noted some “yellowing of my
eyes.” You suspect cirrhosis. All the following are clinical signs of cirrhosis except

A) Arthralgia

B) Asterixis

C) Dupuytren’s contracture

D) Haemorrhoids

E) Testicular atrophy

Answer: A

43. A 64-year-old who suffers from haemochromatosis is seen in A&E with a 2-day history of pain and
swelling in his right knee. On examination it is swollen and he has decreased range of movement. You
aspirate his knee. What will the aspirate be most likely to show?

A) Gram-positive cocci in clusters

B) Positively bifringent crystals


C) Gram-positive cocci in chains

D) Inflammatory cells with haemosiderin deposition

E) Negatively bifringent crystals

Answer: B

44. A 76-year-old gentleman is referred with an international normalized ratio (INR) of 8.4. He normally
takes warfarin for a prosthetic mitral valve. He had an epistaxis lasting 15 minutes this morning. He is
otherwise asymptomatic and haemodynamically stable. Having stopped his warfarin for tonight, Which
of the following is the most appropriate course of action?

A) 5 mg iv vitamin K

B) 0.5 mg orally (po) vitamin K

C) 4 units of fresh frozen plasma iv

D) 50 units/kg of prothrombin complex concentrate

E) Recheck INR in 24 hours

Answer: E

45. Concerning adrenal physiology, which of the following hormones is secreted by the zona
glomerulosa?

A) Adrenaline

B) Noradrenaline

C) Cortisol

D) Aldosterone

E) Angiotensin II

Answer: D
46. A 33-year-old female is seen in the maternity department.she suffers from SLE. Her child was born
with profound bradycardia. Which of the following antibodies is most likely to be present?

A) Anti-Ro

B) Anti-La

C) Anti-Jo

D) Anti-Scl-70

E) Antiphospholipid antibodies

Answer: A

47. A 67-year-old man presents with nausea and pain in his right upper quadrant. On examination, he
has tender hepatomegaly with an irregular edge. His blood tests are as follows:

Hb 8 g/dL (11.5-16 g/dL)

WCC 17.8 x 109/L (4-11 x 109/L)

Platelets 104 x 109/L (150-400x109/L)

Neutrophils 68%

Normoblasts 8%

Myeloblasts 8%

Myelocytes 5%

Metamyelocytes 5%

Lymphocytes 15%

What is the most likely cause of his anaemia?

A) Gaucher's disease

B) Leukaemoid reaction secondary to acute infection

C) Myelosclerosis
D) Malignant infiltration of bone marrow 

E) Osteopetrosis

Answer: D

48. A 32-year-old woman presents with left loin pain and haematuria. She is known to suffer with
Crohn's disease.

Which of the following is likely to be the aetiology of this presentation?

A) Hypercalciuria

B) Hyperbilirubinaemia

C) Hyperuricaemia

D) Hyperoxaluria

E) Type 1 renal tubular acidosis

Answer: D

49. Regarding glucocorticoid-induced hyperglycemia all the following statements are correct EXCEPT.

A) The risk of new onset diabetes mellitus is related to the total glucocorticoid dose and duration of
therapy.

B) Raised fasting plasma glucose is the earliest abnormality. 

C) Reduced insulin sensitivity is the predominant mechanism.

D) Insulin is the most effective and safety therapy

E) Long term use of thiazolidinediones in combination with glucocorticoid is contraindicated.

Answer: B
50. A 65-year-old man has the following blood results on admission to hospital:

Na 132 mmol/L (135-145 mmol/L)

K 8.6 mmol/L (3.5-5 mmol/L)

Urea 42.4 mmol/L (2.5-6.7 mmol/L)

Creatinine 1178 um0l/L (70-150 umol/L)

The patient appears unwell and the ECG shows tented T waves, absent P waves and widened QRS.
Which of the following therapeutic interventions should be performed first?

A) Immediate haemodialysis

B) Intravenous insulin and dextrose

C) Intravenous calcium gluconate

D) Nebulized salbutamol

E) Oral calcium resonium

Answer: C

MULTIPLE CHOICE QUESTIONS-11

1. A 72-year-old male had the onset, within the last 60 minutes, of right hemiparesis and expressive
aphasia. Which of the following is the most important variable to monitor on this patient while a
decision is being made regarding whether to employ thrombolytic therapy? 

A) WBC 

B) Hemoglobin 

C) Blood sugar 

D) Serum electrolytes 

E) Oxygenation status

Answer: E
2. A 29-year-old woman with a history of asthma presents with progressive worsening of respiratory
function. She reports that her symptoms have developed gradually over the past few months, with
occasional fevers and episodes of mild hemoptysis. She denies any weight loss, skin changes, or
diarrhea. Her medications include inhaled albuterol as needed and OCPs. She has no family history of
cancer, no recent travel, and no recent sick contacts. She works as an accountant and does not smoke or
use any illicit drugs. On examination, she has a low-grade fever with scattered wheezes and rhonchi over
bilateral lung fields. After coughing vigorously, a brown mucus plug is expectorated. Her laboratory
values show eosinophilia with an elevated total serum IgE. A chest x-ray shows interstitial infiltrates in
the upper lobes with some areas of atelectasis bilaterally, and a CT scan shows enlarged airways
primarily in the upper lobes with bronchial wall thickening. Which of the following is the most likely
diagnosis?

A) Tuberculosis

B) Worsening asthma

C) Strongyloides stercoralis

D) Allergic bronchopulmonary aspergillosis

E) Bronchial carcinoid tumor

Answer: D

3. All of the following are associated with increased levels of total T4 in the plasma with a normal free T4
EXCEPT:

A) Cirrhosis

B) Pregnancy

C) Sick-euthyroid syndrome

D) Familial dysalbuminemic hyperthyroxinemia

E) Familial excess thyroid-binding globulin

Answer: C
4. A 49-year-old female with a 5-year history of diabetes mellitus type 2 presents for an initial visit. She
has no known complications of diabetes. She takes metformin, glyburide, andaspirin. On examination,
you find a pleasant, obese female in no distress. Her blood pressure is 136/86 mm Hg, pulse 86,
respirations 14, and temperature 37°C. As you discuss monitoring her diabetes, you recommend
screening for early kidney disease. Which of the following approaches is the recommended way to
screen for diabetic kidney disease?

A) Obtain a 24-hour urine collection for albumin now and again in 3 years.

B) Obtain a spot urine microalbumin every year.

C) Obtain a spot urine microalbumin/creatinine ratio every year.

D) Obtain a urinalysis every year.

E) Obtain a serum creatinine every year.

Answer: C

5. Risk factors for foot ulcers in patient with diabetes include all the following EXCEPT:

A) Foot deformity.

B) Peripheral neouropathy.

C) Tobacco smoking.

D) Female sex.

E) Poor glycemic control.

Answer: D

6. A patient with ascites is suspected to have secondary hyperaldosteronism. Which one of the following
would be typical levels of electrolytes in an aliquot specimen of urine?

A) Sodium 2 mEq/L, potassium 40 mEq/L

B) Sodium 5 mEq/L, potassium 0 mEq/L

C) Sodium 40 mEq/L, potassium 40 mEq/L


D) Sodium 80 mEq/L, potassium 2 mEq/L

E) Sodium 100 mEq/L, potassium 20 mEq/L

Answer: A

7. Which of the following conditions should be considered if both the PT and PTT are prolonged in a
patient noted to be oozing from a surgical incision?

A) Severe liver disease, DIC, factor X deficiency.

B) Heparin effect, von Willebrand disease, factor XII deficiency.

C) Warfarin effect, factor VII deficiency, vitamin K deficiency.

D) All of the above.

Answer: A

8. A 27-year-old African-American woman with diabetes presents to the outpatient Neurology clinic with
subacute onset of bilateral facial weakness. She explains that the symptoms have developed over the
course of the last few days. She also reports having a slightly raised and tender rash over the anterior
aspects of both shins. On examination you find bilateral lower motor neuron facial palsy as well as
tender erythematous nodules over both shins. Which of the following is the most likely diagnosis?

A) Guillain-Barré syndrome

B) Lyme disease

C) Neurosarcoidosis

D) Diabetes

E) Tuberculosis

Answer: C
9. Which of the following are the cardinal features of idiopathic Parkinson disease?

A) Tremor, bradykinesia, rigidity, and postural instability

B) Bradykinesia, dementia, tremor, and rigidity

C) Rigidity, hallucinations, tremor, and postural instability

D) Tremor, rigidity, bradykinesia, and gaze palsy

E) Tremor, autonomic dysfunction, bradykinesia, and rigidity

Answer: A

10. According to the Eighth Joint National Committee (JNC-8) guideline on managing hypertension in
adults, Initial Drugs of Choice for Hypertension include all of the following EXCEPT: 

A) ACE inhibitor 

B) Angiotensin receptor blocker 

C) Thiazide diuretic 

D) Calcium channel blocker 

E) β-blockers

Answer: E

11. A 42-year-old male who works in a hog confinement area presents to your office complaining of
cough, fever, wheeze, and dyspnea. He and some other workers were cleaning the confinement area
with high-pressure hoses (which aerosolized hog waste), and they all with the same symptoms, which
started between 4 and 8 hours after work. On examination, he is febrile with a respiratory rate of 28. He
is able to talk in complete sentences. There are slight crackles when you auscultate the lungs. His chest
x-ray is normal. The most likely diagnosis is:

A) “Farmer's lung” (hypersensitivity pneumonitis).

B) Organic dust toxicity syndrome.

C) Reactive airway disease.


D) Hydrogen sulfide poisoning.

E) Bronchiolitis obliterans.

Answer: B

12. Which of the following statements best describes Graves’ ophthalmopathy?

A) Although a cosmetic problem, Graves’ ophthalmopathy is rarely associated with major ocular
complications.

B) Diplopia may occur from periorbital muscle swelling.

C) It is never found without concomitant hyperthyroidism.

D) The most serious complication is corneal abrasion.

E) Unilateral disease is not found.

Answer: B

13. A 20-year-old woman with no significant past medical history presents with a 2-month history of
episodic shortness of breath. These symptoms began with an upper respiratory tract infection. She has
fits of coughing and trouble catching her breath with exertion. She states that her breath “sounds like
whistles” at times. She tried a friend's albuterol inhaler with some improvement and wonders if she has
asthma. On exam, she is breathing comfortably at 16 times per minute and her oxygen saturation is 96%
on room air. Her lungs are clear to auscultation, and the remainder of her exam is unremarkable. You
want to better categorize this patient's disease. Which of the following tests is most appropriate to
order now?

A) Spirometry.

B) Chest x-ray.

C) Arterial blood gas (ABG).

D) Methacholine challenge.

E) Chest CT.
Answer: A

14. Ataxia may be a manifestation of which vitamin deficiency?

A) Vitamin A

B) Vitamin B

C) Vitamin C

D) Vitamin D

E) Vitamin E

Answer: E

and trouble catching her breath with exertion. She states that her breath “sounds like whistles” at times.
She tried a friend's albuterol inhaler with some improvement and wonders if she has asthma. On exam,
she is breathing comfortably at 16 times per minute and her oxygen saturation is 96% on room air. Her
lungs are clear to auscultation, and the remainder of her exam is unremarkable. You want to better
categorize this patient's disease. Which of the following tests is most appropriate to order now?

A) Spirometry.

B) Chest x-ray.

C) Arterial blood gas (ABG).

D) Methacholine challenge.

E) Chest CT.

Answer: A

14. Ataxia may be a manifestation of which vitamin deficiency?

A) Vitamin A

B) Vitamin B
C) Vitamin C

D) Vitamin D

E) Vitamin E

Answer: E

15. A 34-year-old woman with diabetes mellitus, hypertension, dyslipidemia, a family history of
premature coronary artery disease, and ischemic cardiomyopathy with an ejection fraction of 38% by
recent radionuclide angiography is contemplating pregnancy and seeks your advice. She is completely
asymptomatic and physical examination demonstrates no evidence of hypervolemia. She is taking low-
dose aspirin, a beta-blocker, an ACE inhibitor, and a hydroxymethylglutaryl-coenzyme A reductase
inhibitor (statin). Which of the following is the most appropriate statement?

A) Advise the patient to discontinue the ACE inhibitor and pursue pregnancy

B) Advise the patient to discontinue the ACE inhibitor and statin, and pursue pregnancy

C) Advise the patient to discontinue the aspirin and ACE inhibitor and pursue pregnancy

D) Advise the patient to discontinue the beta-blocker and statin and pursue pregnancy

E) Advise the patient not to become pregnant.

Answer: E

16. An asymptomatic 55-year-old male visits a health fair, where he has a panel of blood tests done. He
brings the results to you because he is concerned about the TSH level of 12.0 µU/mL (N 0.45-4.5). His
free T4 level is normal. Which one of the following is most likely to be associated with this finding?

A) Atrial fibrillation 

B) Reduced bone density 

C) Systolic heart failure 

D) Elevated LDL cholesterol 

E) Type 2 diabetes mellitus.


Answer: D

17. An 18-year-old man with a 12-year history of type 1 diabetes mellitus comes to the physician for a
follow-up examination. Medications include 25 U of NPH insulin and 10 U of regular insulin in the
morning and 10 U of NPH insulin and 10 U of regular insulin before dinner. His hemoglobin A1c was
14.5% 12 weeks ago. His current pulse is 80/min, respirations are 20/min, and blood pressure is 145/95
mm Hg. Examination shows scattered retinal microaneurysms bilaterally. The remainder of the
examination shows no other abnormalities. Laboratory studies show:

Hemoglobin A1c 13%

Serum Na+ 130 mEq/L

K+ 3.2 mEq/L

Cl− 101 mEq/L

HCO3− 23 mEq/L

Glucose 325 mg/dL

Creatinine 1.5 mg/dL

Cholesterol 350 mg/dL

Urine Blood negative

Glucose 4+

Protein 1+

Ketones Negative

Which of the following is the most likely renal diagnosis?

A) Cholesterol renal emboli

B) Diabetic nephropathy

C) Hypertensive glomerulosclerosis

D) Hypokalemic nephropathy

E) Sodium-losing nephropathy.
Answer: B

18. Which of the following is increased in patients with emphysema- dominant COPD?

A) DLCO

B) FVC

C) FEV1

D) RV

E) Alpha-1 antitrypsin

Answer: D

19. An 18-year-old man with a childhood diagnosis of small bowel Crohn’s disease attended the eye
hospital Emergency Department with a painless, erythematous right eye. His vision was reported to be
unchanged. What is the most likely diagnosis?

A) Anterior uveitis

B) Episcleritis

C) Intermediate uveitis

D) Posterior uveitis

E) Scleritis.

Answer: B

Which of the following is the most appropriate statement?

A) Advise the patient to discontinue the ACE inhibitor and pursue pregnancy

B) Advise the patient to discontinue the ACE inhibitor and statin, and pursue pregnancy

C) Advise the patient to discontinue the aspirin and ACE inhibitor and pursue pregnancy
D) Advise the patient to discontinue the beta-blocker and statin and pursue pregnancy

E) Advise the patient not to become pregnant.

Answer: E

16. An asymptomatic 55-year-old male visits a health fair, where he has a panel of blood tests done. He
brings the results to you because he is concerned about the TSH level of 12.0 µU/mL (N 0.45-4.5). His
free T4 level is normal. Which one of the following is most likely to be associated with this finding?

A) Atrial fibrillation 

B) Reduced bone density 

C) Systolic heart failure 

D) Elevated LDL cholesterol 

E) Type 2 diabetes mellitus.

Answer: D

17. An 18-year-old man with a 12-year history of type 1 diabetes mellitus comes to the physician for a
follow-up examination. Medications include 25 U of NPH insulin and 10 U of regular insulin in the
morning and 10 U of NPH insulin and 10 U of regular insulin before dinner. His hemoglobin A1c was
14.5% 12 weeks ago. His current pulse is 80/min, respirations are 20/min, and blood pressure is 145/95
mm Hg. Examination shows scattered retinal microaneurysms bilaterally. The remainder of the
examination shows no other abnormalities. Laboratory studies show:

Hemoglobin A1c 13%

Serum Na+ 130 mEq/L

K+ 3.2 mEq/L

Cl− 101 mEq/L

HCO3− 23 mEq/L

Glucose 325 mg/dL


Creatinine 1.5 mg/dL

Cholesterol 350 mg/dL

Urine Blood negative

Glucose 4+

Protein 1+

Ketones Negative

Which of the following is the most likely renal diagnosis?

A) Cholesterol renal emboli

B) Diabetic nephropathy

C) Hypertensive glomerulosclerosis

D) Hypokalemic nephropathy

E) Sodium-losing nephropathy.

Answer: B

18. Which of the following is increased in patients with emphysema- dominant COPD?

A) DLCO

B) FVC

C) FEV1

D) RV

E) Alpha-1 antitrypsin

Answer: D
19. An 18-year-old man with a childhood diagnosis of small bowel Crohn’s disease attended the eye
hospital Emergency Department with a painless, erythematous right eye. His vision was reported to be
unchanged. What is the most likely diagnosis?

A) Anterior uveitis

B) Episcleritis

C) Intermediate uveitis

D) Posterior uveitis

E) Scleritis.

Answer: B

20. A 44-year-old woman undergoes preoperative evaluation prior to surgery to repair a congenital
defect of her pelvis. Her expected blood loss is 2.0 l. She has a prior history of severe anaphylactic
reaction to a prior erythrocyte transfusion that she received for postpartum hemorrhage at age of 27
years. In addition she has a history of rheumatoid arthritis. On physical examination, the temperature is
36.8 °C (98.5 °F), blood pressure is 140/70 mm Hg, and heart rate is 76 bpm. Laboratory studies indicate
a hemoglobin level of 12.0 g/dL, a leukocyte count of 6500 μL, and a platelet count of 150,000 μL.
Previous laboratory studies indicate an IgG level of 800 mg/dL and an IgM level of 65 mg/dL. Which of
the following is the most appropriate erythrocyte transfusion product for this patient?

A) Leuko-reduced blood

B) Cytomegalovirus-negative blood

C) Irradiated blood

D) Phenotypically matched blood

E) Washed blood

Answer: E

21. A 75-year-old man underwent surgery to correct a large abdominal aortic aneurysm. The procedure
appeared to go well, but you are called a few hours later to evaluate the patient who states that he
cannot move or feel his legs. On the way to the ICU, you consider the possible causes of his symptoms
and plan your physical examination. What is the most important test to help localize the lesion?

A) MRI of the spine

B) Sensory level

C) Reflexes in lower extremities

D) Plantar flexion reflex

E) Toe position sense.

Answer: E

22. A 35-year-old man is suspected of having a small right-sided pleural effusion. What imaging modality
is most sensitive to detect a small amount of pleural fluid?

A) chest CT

B) lateral chest film

C) left lateral decubitus chest film

D) standard upright chest film

Answer: A

23. A 70-year-old man with history of HTN and DM presents with a 2-month history of increasing
paroxysmal nocturnal dyspnea and SOB with minimal exertion. An echocardiogram shows an ejection
fraction of 25%. Which one of the patients current medications should be discontinued?

A) Lisinopril 

B) Pioglitazone 

C) Glipizide 

D) Metoprolol
Answer: B

24. A 40-year-old male is diagnosed with active TB. In educating the pt, he is told to avoid tyramine
containing foods and the possibility of the development of hepatitis and peripheral neuropathy. Which
of the following medication is the pt being educated about?

A) rifampin

B) Isoniazid

C) Pyrazinamide

D) ethambutol

E) streptomycin

Answer: B

25. A 32-year-old man with family history of HTN measures his own BP daily and keeps in his diary. He
visits his primary care physician, and his BP is found to be 124/72 mm Hg, which is much lower than his
home BP. He insists that his BP be measured 2 h later, which was found to be 125/73 mm Hg. He asks
you to clarify about this discrepancy between office and home BP measurements, and you would say
that he has:

A) Essential HTN

B) Prehypertension

C) White coat HTN

D) Masked HTN

E) Secondary HTN

Answer: D

26. An 88-year-old man is taken to A&E by his daughter as he had been feeling unwell that day with
chest pains. An ECG reveals that he is suffering an inferior myocardial infarction. He has a heart rate of
33 bpm and blood pressure of 90/60 mmHg. He is given 600mcg of atropine to no effect, this is repeated
up to 3mg. You commence an infusion of TPA but his rate is not immediately improving. Which of the
following treatments would be most suitable to use for his arrhythmia?

A) Adrenaline

B) Glucagon

C) Temporary pacing

D) Dopamine

C) Dobutamine

Answer: C

27. All of the following are common manifestations of cytomegalovirus (CMV) infection following lung
transplantation EXCEPT:

A) bronchiolitis obliterans

B) CMV esophagitis

C) CMV pneumonia

D) CMV retinitis

E) CMV syndrome (fever, malaise, cytopenias, transaminitis, and CMV viremia)

Answer: D

28. A 25-year-old man presents to the ED with a broad complex tachycardia that is irregularly irregular.
The patient is haemodynamically uncompromised. An anaesthetist is not available to assist with
immediate DC cardioversion. What is the best initial treatment option?

A) IV adenosine

B) IV verapamil

C) Oral beta-blocker

D) IV beta-blocker
E) IV flecainide

Answer: E

29. A young patient is diagnosed with multiple endocrine neoplasia (MEN) type 3 (also known as type
2b) after an episode of bowel obstruction. Which one of the following features would he be unlikely to
have or develop in the future with this diagnosis?

A) Medullary carcinoma of the thyroid

B) Marfanoid appearance

C) Mucosal neuromas of the small bowel

D) Facial angiofibromas

E) Prognathism

Answer: D

30. All the following types of cancer commonly metastasize to the central nervous system (CNS) EXCEPT:

A) ovarian

B) breast

C) hypernephroma

D) melanoma

E) acute lymphoblastic leukemia (ALL)

Answer: A

31. A 32-year-old woman presents with diarrhea, with mucus and blood in the stool, and recurrent
abdominal pain. Colonoscopy is performed and a mucosal biopsy is obtained. Which one of the
following pathological findings would make ulcerative colitis a more likely diagnosis than Crohn's
disease? 

A) Ileal involvement 

B) Crypt abscesses 

C) Transmural involvement 

D) Granulomas 

E) Skip lesions

Answer: B

32. A 61-year-old man is admitted because of increasing confusion and inadequately controlled lower
backache. He is found to have multiple myeloma. Which of the following statements regarding this
patient's disease is correct? 

A) Osteoporotic vertebral collapse fracture may be responsible for his backache 

B) Radiotherapy is ineffective in treating bone pain 

C) Isotope bone scanning is more sensitive for identifying early disease 

D) Plain radiographs of the spine would show typical widespread osteoblastic bone lesions 

E) The hypercalcaemia is usually steroid-resistant

Answer: A

33. A 64-year-old builder who had a surgical operation for inguinal hernia a develops a wound infection
a week later and methicillin-resistant Staphylococcus aureus (MRSA) is recovered from the wound.
Which one of the following isolation techniques do you think is appropriate in this condition? 

A) Enteric precautions 

B) Blood and body fluid precautions 

C) Respiratory isolation only 


D) Strict isolation 

E) Contact isolation

Answer: E

34. A teacher has brought his son to you because he is very worried that his son might be suffering with
haemophilia. Which one of the following best excludes a diagnosis of haemophilia? 

A) Normal activated partial thromboplastin time 

B) Normal prothrombin time 

C) Absence of excessive bleeding after tooth extraction 

D) Absence of spontaneous bleeding episodes 

E) Absence of family history

Answer: C

35. A 20-year-old car mechanic who has been abusing inhalants for many years was admitted with
sudden collapse. The ambulance crew said he was inhaling solvent by rebreathing from a plastic bag. He
is transferred to the Intensive Care Unit. A few minutes later he stops breathing and the cardiac monitor
records ventricular fibrillation rhythm. In resuscitating this patient which one of the following should be
avoided? 

A) Direct current shock 

B) Intubation 

C) Oxygen therapy 

D) Intravenous ß-blocker 

E) Intravenous adrenaline (epinephrine)

Answer: E
36. An 88-year-old woman is referred because of a tense bullous eruption on the inner sides of the
upper and lower limbs as well as the abdomen. The most likely diagnosis is 

A) Pemphigus 

B) Bullous pemphigoid 

C) Herpes zoster 

D) Dermatitis herpetiformis 

E) Stevens-Johnson syndrome

Answer: B

37. A 40-year-old male dancer presents with thrombophlebitis of the lower limb. Examination reveals
five painless, punched-out ulcers on the scrotum with concomitant oral ulcers. Which of the following is
the most likely diagnosis? 

A) Granuloma inguinale 

B) Syphilis 

C) Herpes simplex genitalis 

D) Belicet's disease 

E) Systemic lupus erythematosus

Answer: D

38. A 50-year-old woman is admitted with high fever and a generalised headache. Examination reveals
chemosis and cyanosis of the upper face. There was proptosis and ophthalmoplegia of the left eye. Pin
prick sensation was lost from the left forehead. The most likely diagnosis is 

A) Periorbital cellulitis 

B) Osteomyelitis of the left maxillary sinus 


C) Left retro-orbital tumour 

D) Cavernous sinus thrombosis 

E) Superior sagittal sinus thrombosis

Answer: D

39. A 59-year-old lady is being investigated for haematuria. A diagnosis of glomerulonephritis is


suspected. Which one of the following tests is not indicated?

A) X-ray kidney, ureter, bladder (KUB).

B) Urine microscopy.

C) Serum and urine immunoelectrophoresis.

D) Complement levels.

E) Antineutrophil cytoplasmic (ANCA) and antiglomerular basement membrane (anti-GM) antibodies.

Answer: A

40. Which of the following is incorrect regarding specific treatment of overdose?

A) Naloxone for opiate overdose is often given as an infusion due to its short half-life.

B) Flumazenil can be given for benzodiazepine overdose.

C) N-acetylcysteine for paracetamol overdose can only be given intravenously.

D) Oral methionine is effective in aspirin overdose.

E) ’Digibind’ is a fragment of digoxin-specific antibody given in severe overdose.

Answer: D
41. A 55-year-old man with chronic hepatitis C is being considered for liver transplantation. The patient
has cirrhosis that was documented by liver biopsy 10 years ago. For the past 3 months, he has had
ascites and edema, which are poorly controlled with diuretics. Lactulose was recently begun because of
confusion. Which of the following combinations of laboratory studies will be most helpful in estimating
his survival over the next 6 months? 

A) Serum total bilirubin and INR 

B) Serum aspartate aminotransferase and gamma globulin 

C) Serum alanine aminotransferase and hepatitis C RNA (HCV RNA) 

D) Serum alkaline phosphatase and ammonia 

E) Serum albumin and ?-glutamyltransferase

Answer: A

42. Signs and symptoms of opioid withdrawal include all of the following EXCEPT

A) Increased blood pressure (BP) and heart rate

B) Seizures

C) Abdominal cramps

D) Jerking of the legs

E) Hyperthermia

Answer: B

43. Which of the following β-adrenergic antagonists is a nonselective β1 and β2 blocker?

A) Atenolol

B) Betaxolol

C) Esmolol

D) Metoprolol
E) Nadolol

Answer: E

44. In primary hypothyroidism increased requirements for L thyroxine occur in the following conditions
EXCEPT:

A) Nephrotic syndrome with massive proteinuria.

B) Pregnancy.

C) Achalasia

D) Helicobacter pylori associated chronic gastritis.

E) Use of estrogen replacement therapy.

Answer: C

45. A 70-year-old man is admitted with new-onset atrial fibrillation with an apical rate of 120/minute
and a BP of 100/70 mm Hg. Shortly after admission he develops moderate lower abdominal pain and
diarrhea with dark red blood.

Which is the most appropriate course of action?

A) Cardioversion 

B) Unfractionated heparin

C) Mesenteric angiography

D) Laparotomy

E) Digoxin

Answer: A
46. An 18-year-old insulin-dependent diabetic is seen in A&E. Over the last 4 days she has suffered from
diarrhoea and vomitting, and has cut down on her insulin injections. Her mother has brought her in
because she has become drowsy and confused. Her blood results are as follows:

WCC 16.4 X109/L (4-11X109/L)

Hb 17.8 * g/dL (12-16 g/dL)

Platelets 597 X10/L (150-400X109/L)

Na 146 mmpl/L (135-145 mmol/L)

K 6.6 mmol/L (3.5-5 mmol/L)

Urea 8.4 mmol/L (2.5-6.7 mmol/L)

Creatinine 145 umol/L (60-120 umol/L)

Random glucose 26 mmol/L (4.5-5.6 mmol/L)

You are with one nurse in the resuscitation room. What is the first thing you are going to ask her to do? 

A) Start an insulin sliding scale

B) Give 1 L of normal saline over 30 minutes 

C) Run to the lab with blood gases

D) Draw up 10 mL of 10% calcium gluconate

E) Give 100 mmol bicarbonate intravenously (iv)

Answer: B

47. A 73-year-old male presents to the clinic with 3 months

of increasing back pain. He localizes the pain to the lumbar

spine and states that the pain is worst at night while he is lying

in bed. It is improved during the day with mobilization.

Past history is notable only for hypertension and remote cigarette smoking. Physical examination is
normal. Laboratory

studies are notable for an elevated alkaline phosphatase. A


lumbar radiogram shows a lytic lesion in the L3 vertebra.

Which of the following malignancies is most likely?

A) Gastric carcinoma

B) Non-small cell lung cancer

C) Osteosarcoma

D) Pancreatic carcinoma

E) Thyroid carcinoma

Answer: B

48. A 36-year-old African-American woman with systemic lupus erythematosus presents with the acute
onset of lethargy and jaundice. On initial evaluation, she is tachycardic, hypotensive, appears pale, is
dyspneic, and is somewhat difficult to arouse. Physical examination reveals splenomegaly.

Her initial hemoglobin is 6 g/dL, white blood cell count is 6300/μL, and platelets are 294,000/ μL. Her
total bilirubin is 4 g/dL, reticulocyte count is 18%, and haptoglobin is not detectable. Renal function is
normal, as is urinalysis. What would you expect on her peripheral blood smear?

A) Macrocytosis and PMN’s with hypersegmented nuclei

B) Microspherocytes

C) Schistocytes

D) Sickle cells

E) Target cells

Answer: B

49. A 22 year-old lady presents to her GP with facial swelling, most marked in the morning, and
shortness of breath. Urinalysis shows protein +++. Results were as follows: 

24-hour urinary protein 4.2 g; 


Haemoglobin(Hb) 10.8 g/l (11.5-16.0 g/L); 

Urea 22.8 mmol/L (2.5-6.5 mmol/L); 

Creatinine 374 µmol/L (70-150 umol/L)

C3 0.24 g/L (0.55-1.2g/L)

C4 0.29g/L (0.2-0.5 g/L)

Which of the following is the most likely diagnosis?

A) Minimal change nephropathy

B) Systemic lupus erythematosus

C) Rheumatoid arthritis

D) Cryoglobulinaemia

E) Diabetes mellitus

Answer: B

50. A 31-year-old male with hemophilia A is admitted with persistent gross hematuria. He denies recent
trauma or any history of genitourinary pathology. The examination is unremarkable. Hematocrit is 28%.
All the following are treatments for hemophilia A except

A) desmopressin (DDAVP)

B) fresh-frozen plasma (FFP)

C) cryoprecipitate

D) recombinant factor VIII

E) plasmapheresis

Answer: E

You might also like